Вы находитесь на странице: 1из 166

Albania

Team Selection Test


2013

1 Find the 3-digit number whose ratio with the sum of its digits its minimal.
2 Let a, b, c, d be positive real numbers such that abcd = 1.Find with proof that x = 3 is the
minimal value for which the following inequality holds :
1 1 1 1
ax + bx + cx + dx + + +
a b
c d
3 Solve the function f : < <
f (x3 ) + f (y 3 ) = (x + y)(f (x2 ) + f (y 2 ) f (x*y))
4 It is given a triangle ABC whose circumcenter is O and orthocenter H. If AO = AH find
of that triangle.
the angle BAC
5 Let k be a natural number.Find all the couples of natural numbers (n, m) such that : (2k )! =
2n m

This file was downloaded from the AoPS Math Olympiad Resources Page
http://www.artofproblemsolving.com/

Page 1

Argentina
National Olympiad
2013

Day 1
1 2013 cards, each one of them having a different number from 1 to 2013, are placed upside
down on a table (i.e. the numbers cant be seen). One may ask whether the arithmetic mean
of any set of cards is an integer. The answer will always be true. a) Find all numbers that
can be determined without a doubt asking these questions. b) Some groups are formed using
cards such that we know what numbers are in each group, but we dont necessarily know what
number is written in each card (for example, find a group of 3 cards that have the numbers
1, 2, and 3, without knowing what number each card has). What is the maximum amount of
groups that can be formed?
2 In a convex quadrilateral ABCD, BAD = BCD, and the angle bisector of ABC passes
AB
through the midpoint of CD. If CD = 3AD, find BC
.
EDIT: Its supposed to be BAD = BCD, not BAC = BCD (thank you Vo Duc Dien
for pointing it out!)
3 Find out how many 2013-digit numbers d1 d2 . . . d2013 , all of its digits being odd, exist such
that d1 d2 + d2 d3 + . . . + d1809 d1810 has remainder 1 after dividing it by 4, and d1810
d1811 + d1811 d1812 + . . . + d2012 d2013 has remainder 1 after dividing it by 4.

This file was downloaded from the AoPS Math Olympiad Resources Page
http://www.artofproblemsolving.com/

Page 1

Argentina
National Olympiad
2013

Day 2
4 Let x 5, y 6, z 7 such that x2 + y 2 + z 2 125. Find the minimum value of x + y + z.
5 The numbers 1 through 2013 are written on a board. A valid operation is to pick a positive
integer a from the board, and replace each number b a with b a (if there is any number a
on the board other than the original one, then it is replaced with a 0). After some operations,
the sum of the numbers of the board is equal to 10. Determine which numbers may be on
the board at this moment. Give all possibilites.
6 A positive integer n is called pretty if there exists two divisors d1 , d2 of n (1 d1 , d2 n)
such that d2 d1 = d for each divisor d of n (where 1 < d < n). Find the smallest pretty
number larger than 401 that is a multiple of 401.

This file was downloaded from the AoPS Math Olympiad Resources Page
http://www.artofproblemsolving.com/

Page 2

Austria
Federal Competition For Advanced Students, Part 1
2013

1 Show that if for non-negative integers m, n, N , k the equation


k

(n2 + 1)2 (44n3 + 11n2 + 10n + 2) = N m


holds, then m = 1.
2 Solve the following system of equations in rational numbers:
(x2 + 1)3 = y + 1, (y 2 + 1)3 = z + 1, (z 2 + 1)3 = x + 1.
3 Arrange the positive integers into two lines as follows:
1

11

19

32

53 . . . 2

4 5

7 8 9 10

We start with writing 1 in the upper line, 2 in the lower line and 3 again in the upper line.
Afterwards, we alternately write one single integer in the upper line and a block of integers
in the lower line. The number of consecutive integers in a block is determined by the first
number in the previous block. Let a1 , a2 , a3 , . . . be the numbers in the upper line. Give an
explicit formula for an .
4 Let A, B and C be three points on a line (in this order). For each circle k through the points
B and C, let D be one point of intersection of the perpendicular bisector of BC with the
circle k. Further, let E be the second point of intersection of the line AD with k. Show that
for each circle k, the ratio of lengths BE : CE is the same.

This file was downloaded from the AoPS Math Olympiad Resources Page
http://www.artofproblemsolving.com/

Page 1

12 13 14 15 1

Austria
Federal Competition For Advanced Students, Part 2
2013

Day 1
1 For each pair (a, b) of positive integers, determine all non-negative integers n such that
jnk n + b
b+
.
=
a
a
2 Let k be an integer. Determine all functions f : R R with f (0) = 0 and
f (xk y k ) = xyf (x)f (y)

for x, y 6= 0.

3 A square and an equilateral triangle are inscribed in a same circle. The seven vertices form
a convex heptagon S inscribed in the circle (S might be a hexagon if two vertices coincide).
For which positions of the triangle relative to the square does S have the largest and smallest
area, respectively?

This file was downloaded from the AoPS Math Olympiad Resources Page
http://www.artofproblemsolving.com/

Page 1

Austria
Federal Competition For Advanced Students, Part 2
2013

Day 2
4 For a positive integer n, let a1 , a2 , . . . an be nonnegative real numbers such that
P for all real
numbers x1 > x2 > . . . > xn > 0 with x1 + x2 + . . . + xn < 1, the inequality nk=1 ak x3k < 1
holds. Show that
na1 + (n 1)a2 + . . . + (n j + 1)aj + . . . + an 6

n2 (n + 1)2
.
4

5 Let n > 3 be an integer. Let A1 A2 . . . An be a convex n-gon. Consider a line g through


A1 that does not contain a further vertice of the n-gon. Let h be the perpendicular to g
through A1 . Project the n-gon orthogonally on h. For j = 1, . . . , n, let Bj be the image of Aj
under this projection. The line g is called admissible if the points Bj are pairwise distinct.
Consider all convex n-gons and all admissible lines g. How many different orders of the points
B1 , . . . , Bn are possible?
6 Consider a regular octahedron ABCDEF with lower vertex E, upper vertex F , middle crosssection ABCD, midpoint M and circumscribed sphere k. Further, let X be an arbitrary
point inside the face ABF . Let the line EX intersect k in E and Z, and the plane ABCD
in Y . Show that ^EM Z = ^EY F .

This file was downloaded from the AoPS Math Olympiad Resources Page
http://www.artofproblemsolving.com/

Page 2

Bosnia Herzegovina
Team Selection Test
2013

Day 1
1 Triangle ABC is right angled at C. Lines AM and BN are internal angle bisectors. AM and
BN intersect altitude CH at points P and Q respectively. Prove that the line which passes
through the midpoints of segments QN and P M is parallel to AB.
2 The sequence an is defined by a0 = a1 = 1 and an+1 = 14an an1 4,for all positive integers
n. Prove that all terms of this sequence are perfect squares.

3 Prove that in the set consisting of 2n
n people we can find a group of n + 1 people in which
everyone knows everyone or noone knows noone.

This file was downloaded from the AoPS Math Olympiad Resources Page
http://www.artofproblemsolving.com/

Page 1

Bosnia Herzegovina
Team Selection Test
2013

Day 2
4 Find all primes p, q such that p divides 30q 1 and q divides 30p 1.
5 Let x1 , x2 , . . . , xn be nonnegative real numbers of sum equal to 1. Let Fn = x21 + x22 + +
x2n 2(x1 x2 + x2 x3 + + xn x1 ). Find: a) min F3 ; b) min F4 ; c) min F5 .
6 In triangle ABC, I is the incenter. We have chosen points P, Q, R on segments IA, IB, IC
respectively such that IP IA = IQ IB = IR IC. Prove that the points I and O belong to
Euler line of triangle P QR where O is circumcenter of ABC.

This file was downloaded from the AoPS Math Olympiad Resources Page
http://www.artofproblemsolving.com/

Page 2

Brazil
National Olympiad
2013

Day 1 - 19 October 2013


1 Let be a circle and A a point outside . The tangent lines to through A touch at B
and C. Let M be the midpoint of AB. The segment M C meets again at D and the line
AD meets again at E. Given that AB = a, BC = b, compute CE in terms of a and b.
2 Arnaldo and Bernaldo play the following game: given a fixed finite set of positive integers A
known by both players, Arnaldo picks a number a A but doesnt tell it to anyone. Bernaldo
thens pick an arbitrary positive integer b (not necessarily in A). Then Arnaldo tells the
number of divisors of ab. Show that Bernaldo can choose b in a way that he can find out the
number a chosen by Arnaldo.
3 Find all injective functions f : R R from the non-zero reals to the non-zero reals, such
that
f (x + y) (f (x) + f (y)) = f (xy)
for all non-zero reals x, y such that x + y 6= 0.

This file was downloaded from the AoPS Math Olympiad Resources Page
http://www.artofproblemsolving.com/

Page 1

Brazil
National Olympiad
2013

Day 2 - 20 October 2013


4 Find the largest n for which there exists a sequence (a0 , a1 , . . . , an ) of non-zero digits such that,
for each k, 1 k n, the k-digit number ak1 ak2 . . . a0 = ak1 10k1 + ak2 10k2 + + a0
divides the (k + 1)-digit number ak ak1 ak2 . . . a0 .

P.S.: This is basically the same problem as ! m a class=postlink href=http://www.artofproblemsolving


... 7t=548550/a! m .
5 Let x be an irrational number between 0 and 1 and x = 0.a1 a2 a3 its decimal representation.
For each k 1, let p(k) denote the number of distinct sequences aj+1 aj+2 aj+k of k
consecutive digits in the decimal representation of x. Prove that p(k) k + 1 for every
positive integer k.
6 The incircle of triangle ABC touches sides BC, CA and AB at points D, E and F , respectively.
Let P be the intersection of lines AD and BE. The reflections of P with respect to EF, F D
and DE are X, Y and Z, respectively. Prove that lines AX, BY and CZ are concurrent at a
point on line IO, where I and O are the incenter and circumcenter of triangle ABC.

10 This file was downloaded from the AoPS Math Olympiad Resources Page
http://www.artofproblemsolving.com/

Page 2

Brazil
Olympic Revenge
2013

1 Let n to be a positive integer. A family of intervals [i, j] with 0 i < j n and i, j


integers is considered happy if, for any I1 = [i1 , j1 ] and I2 = [i2 , j2 ] such that I1 I2 ,
we have i1 = i2 or j1 = j2 .
Determine the maximum number of elements of a happy family.
2 Let ABC to be an acute triangle. Also, let K and L to be the two intersections of the
perpendicular from B with respect to side AC with the circle of diameter AC, with K closer
to B than L. Analogously, X and Y are the two intersections of the perpendicular from C
with respect to side AB with the circle of diamter AB, with X closer to C than Y . Prove
that the intersection of XL and KY lies on BC.
3 Let a, b, c, d to be non negative real numbers satisfying ab + ac + ad + bc + bd + cd = 6. Prove
that

a2

1
1
1
1
+ 2
+ 2
+ 2
2
+1 b +1 c +1 d +1

4 Find all triples (p, n, k) of positive integers, where p is a Fermats Prime, satisfying
pn + n = (n + 1)k
.
Observation: a Fermats Prime is a prime number of the form 2 + 1, for positive integer.
5 Consider n lamps clockwise numbered from 1 to n on a circle.
Let to be a configuration where 0 ` n random lamps are turned on. A cool procedure
consists in perform, simultaneously, the following operations: for each one of the ` lamps
which are turned on, we verify the number of the lamp; if i is turned on, a signal of range i
is sent by this lamp, and it will be received only by the next i lamps which follow i, turned
on or turned off, also considered clockwise. At the end of the operations we verify, for each
lamp, turned on or turned off, how many signals it has received. If it was reached by an even
number of signals, it remains on the same state(that is, if it was turned on, it will be turned
on; if it was turned off, it will be turned off). Otherwise, its state will be changed.
The example in attachment, for n = 4, ilustrates a configuration where lamps 2 and 4 are
initially turned on. Lamp 2 sends signal only for the lamps 3 e 4, while lamp 4 sends signal
for lamps 1, 2, 3 e 4. Therefore, we verify that lamps 1 e 2 received only one signal, while
lamps 3 e 4 received two signals. Therefore, in the next configuration, lamps 1 e 4 will be
turned on, while lamps 2 e 3 will be turned off.
Let to be the set of all 2n possible configurations, where 0 ` n random lamps are
turned on. We define a function f : where, if is a configuration of lamps, then f ()
is the configurations obtained after we perform the cool procedure described above.

This file was downloaded from the AoPS Math Olympiad Resources Page
http://www.artofproblemsolving.com/

11

Page 1

Brazil
Olympic Revenge
2013

Determine all values of n for which f is bijective.

12 This file was downloaded from the AoPS Math Olympiad Resources Page
http://www.artofproblemsolving.com/

Page 2

Canada
Canadian Mathematical Olympiad Qualification Repechage
2013

1 Determine all real solutions to the following equation:


x

2(2 ) 3 2(2

x1 +1)

+ 8 = 0.

2 In triangle ABC, A = 90 and C = 70 . F is point on AB such that ACF = 30 , and


E is a point on CA such that CF E = 20 . Prove that BE bisects B.
3 A positive integer n has the property that there are three positive integers x, y, z such that
lcm(x, y) = 180, lcm(x, z) = 900, and lcm(y, z) = n, where lcm denotes the lowest common
multiple. Determine the number of positive integers n with this property.
4 Four boys and four girls each bring one gift to a Christmas gift exchange. On a sheet of
paper, each boy randomly writes down the name of one girl, and each girl randomly writes
down the name of one boy. At the same time, each person passes their gift to the person
whose name is written on their sheet. Determine the probability that both of these events
occur:
(i) Each person receives exactly one gift; [/*:m] (ii) No two people exchanged presents with
each other (i.e., if A gave his gift to B, then B did not give her gift to A).[/*:m]
5 For each positive integer k, let S(k) be the sum of its digits. For example, S(21) = 3 and
S(105) = 6. Let n be the smallest integer for which S(n) S(5n) = 2013. Determine the
number of digits in n.
6 Let x, y, z be real numbers that are greater than or equal to 0 and less than or equal to

1
2

(a) Determine the minimum possible value of


x + y + z xy yz zx
and determine all triples (x, y, z) for which this minimum is obtained. [/*:m] (b) Determine
the maximum possible value of
x + y + z xy yz zx
and determine all triples (x, y, z) for which this maximum is obtained.[/*:m]
7 Consider the following layouts of nine triangles with the letters A, B, C, D, E, F, G, H, I in its
interior.

This file was downloaded from the AoPS Math Olympiad Resources Page
http://www.artofproblemsolving.com/

13

Page 1

Canada
Canadian Mathematical Olympiad Qualification Repechage
2013

A
B
E

D
H

A sequence of letters, each letter chosen fromA, B, C, D, E, F, G, H, I is said to be trianglefriendly if the rst and last letter of the sequence is C, and for every letter except the rst letter,
the triangle containing this letter shares an edge with the triangle containing the previous
letter in the sequence. For example, the letter after C must be either A, B, or D. For example,
CBF BC is triangle-friendly, but CBF GH and CBBHC are not.
(a) Determine the number of triangle-friendly sequences with 2012 letters. [/*:m] (b) Determine the number of triangle-friendly sequences with exactly 2013 letters.[/*:m]
8 Let ABC be an acute-angled triangle with orthocentre H and circumcentre O. Let R be the
radius of the circumcircle.
Let A be the point on AO (extended if necessary) for which HA AO.
Let B be the point on BO (extended if necessary) for which HB BO.
Let C be the point on CO (extended if necessary) for which HC CO.

Prove that HA0 + HB 0 + HC 0 < 2R


(Note: The orthocentre of a triangle is the intersection of the three altitudes of the triangle. The circumcircle of a triangle is the circle passing through the triangles three vertices. The
circummcentre is the centre of the circumcircle.)

14 This file was downloaded from the AoPS Math Olympiad Resources Page
http://www.artofproblemsolving.com/

Page 2

Canada
National Olympiad
2013

1 Determine all polynomials P (x) with real coefficients such that


(x + 1)P (x 1) (x 1)P (x)
is a constant polynomial.
2 The sequence a1 , a2 , . . . , an consists of the numbers 1, 2, . . . , n in some order. For which
positive integers n is it possible that the n + 1 numbers 0, a1 , a1 + a2 , a1 + a2 + a3 , . . . , a1 +
a2 + + an all have different remainders when divided by n + 1?
3 Let G be the centroid of a right-angled triangle ABC with BCA = 90 . Let P be the
point on ray AG such that CP A = CAB, and let Q be the point on ray BG such that
CQB = ABC. Prove that the circumcircles of triangles AQG and BP G meet at a point
on side AB.
4 Let n be a positive integer. For any positive integer j and positive real number r, define fj (r)
and gj (r) by

  

j
j
, n , and gj (r) = min(djre, n) + min
,n ,
fj (r) = min(jr, n) + min
r
r
where dxe denotes the smallest integer greater than or equal to x. Prove that
n
X
j=1

fj (r) n + n

n
X

gj (r)

j=1

for all positive real numbers r.


5 Let O denote the circumcentre of an acute-angled triangle ABC. Let point P on side AB
be such that BOP = ABC, and let point Q on side AC be such that COQ = ACB.
Prove that the reflection of BC in the line P Q is tangent to the circumcircle of triangle AP Q.

This file was downloaded from the AoPS Math Olympiad Resources Page
http://www.artofproblemsolving.com/

15

Page 1

China
China Girls Math Olympiad
2013

Day 1
1 Let A be the closed region bounded by the following three lines in the xy plane: x = 1, y = 0
and y = t(2x t), where 0 < t < 1. Prove that the area of any triangle inside the region A,
with two vertices P (t, t2 ) and Q(1, 0), does not exceed 14 .
2 As shown in the figure below, ABCD is a trapezoid, AB k CD. The sides DA, AB, BC are
tangent to O1 and AB touches O1 at P . The sides BC, CD, DA are tangent to O2 ,
and CD touches O2 at Q. Prove that the lines AC, BD, P Q meet at the same point.

D
A
Q

O1
P

O2

3 In a group of m girls and n boys, any two persons either know each other or do not know
each other. For any two boys and any two girls, there are at least one boy and one girl among
them,who do not know each other. Prove that the number of unordered pairs of (boy, girl)
who know each other does not exceed m + n(n1)
.
2
4 Find the number of polynomials f (x) = ax3 + bx satisfying both following conditions: (i)
a, b {1, 2, . . . , 2013}; (ii) the difference between any two of f (1), f (2), . . . , f (2013) is not a
multiple of 2013.

This file was downloaded from the AoPS Math Olympiad Resources Page
http://www.artofproblemsolving.com/

16

Page 1

China
China Girls Math Olympiad
2013

Day 2
5 For any given
P positive numbers a1 , a2 , . . . , an , prove that there exist positive numbers
Pn x1 , x2 , . . . , xn
satisfying ni=1Pxi = 1, such that
for
any
positive
numbers
y
,
y
,
.
.
.
,
y
with
1 2
n
i=1 yi = 1,
n
ai xi
1 Pn
the inequality i=1 xi +yi 2 i=1 ai holds.
6 Let S be a subset of {0, 1, 2, . . . , 98} with exactly m 3 (distinct) elements, such that for
any x, y S there exists z S satisfying x + y 2z (mod 99). Determine all possible values
of m.
7 As shown in the figure, O1 and O2 touches each other externally at a point T , quadrilateral
ABCD is inscribed in O1 , and the lines DA, CB are tangent to O2 at points E and F
respectively. Line BN bisects ABF and meets segment EF at N . Line F T meets the arc
d (not passing through the point B) at another point M different from A. Prove that M
AT
is the circumcenter of 4BCN .
8 Let n ( 4) be an even integer. We label n pairwise distinct real numbers arbitrarily on the
n vertices of a regular n-gon, and label the n sides clockwise as e1 , e2 , . . . , en . A side is called
positive if the numbers on both endpoints are increasing in clockwise direction. An unordered
pair of distinct sides {ei , ej } is called alternating if it satisfies both conditions:
(i) 2 | (i + j); and
(ii) if one rearranges the four numbers on the vertices of these two sides ei and ej in increasing
order a < b < c < d, then a and c are the numbers on the two endpoints of one of sides ei or
ej .
Prove that the number of alternating pairs of sides and the number of positive sides are of
different parity.

17 This file was downloaded from the AoPS Math Olympiad Resources Page
http://www.artofproblemsolving.com/

Page 2

China
National Olympiad
2013

Day 1
1 Two circles K1 and K2 of different radii intersect at two points A and B, let C and D be
two points on K1 and K2 , respectively, such that A is the midpoint of the segment CD. The
extension of DB meets K1 at another point E, the extension of CB meets K2 at another
point F . Let l1 and l2 be the perpendicular bisectors of CD and EF , respectively. i) Show
that l1 and l2 have a unique common point (denoted by P ). ii) Prove that the lengths of CA,
AP and P E are the side lengths of a right triangle.
2 Find all nonempty sets S of integers such that 3m 2n S for all (not necessarily distinct)
m, n S.
3 Find all positive real numbers t with the following property: there exists an infinite set X of
real numbers such that the inequality
max{|x (a d)|, |y a|, |z (a + d)|} > td
holds for all (not necessarily distinct) x, y, z X, all real numbers a and all positive real
numbers d.

This file was downloaded from the AoPS Math Olympiad Resources Page
http://www.artofproblemsolving.com/

18

Page 1

China
National Olympiad
2013

Day 2
1 Let n > 2 be an integer. There are n finite sets A1 , A2 , . . . , An which satisfy the condition
|Ai Aj | = |i j|
Find the minimum of

n
P

i, j {1, 2, ..., n} .

|Ai |.

i=1

2 For any positive integer n and 0 6 i 6 n, denote Cni c(n, i) (mod 2), where c(n, i) {0, 1}.
Define
n
X
f (n, q) =
c(n, i)q i
i=0

where m, n, q are positive integers and q +1 6= 2 for any N. Prove that if f (m, q) |f (n, q) ,
then f (m, r) |f (n, r) for any positive integer r.
3 Let m, n be positive integers. Find the minimum positive integer N which satisfies the
following condition. If there exists a set S of integers that contains a complete residue
P system
module m such that |S| = N , then there exists a nonempty set A S so that n |
x.
xA

19 This file was downloaded from the AoPS Math Olympiad Resources Page
http://www.artofproblemsolving.com/

Page 2

China
National Olympiad
2013

Day 3 - 21 December 2013


1 Let ABC be a triangle with AB > AC. Let D be foot of internal angle bisector of A. F and
E on AC, AB such that BCF E concyclic. Prove that circumcentre of DEF is incentre of
ABC if and only if BE + CF = BC
2 For the integer n > 1 , define D(n) = {a b | ab = n, a > b > 0, a, b N}. Prove
that: For any integer k > 1 , there exists pairwise distinct positive integersn1 , n2 , . . . , nk and
n1 > 1, n2 > 1, , nk > 1, such that |D(n1 ) D(n2 ) D(nk )| 2. (Edited)
3 Prove that: there exists only one function f : N N satisfying: i) f (1) = f (2) = 1;
ii)f (n) = f (f (n 1)) + f (n f (n 1)) for n 3. For each integer m 2, find the value of
f (2m ).

20 This file was downloaded from the AoPS Math Olympiad Resources Page
http://www.artofproblemsolving.com/

Page 3

China
National Olympiad
2013

Day 4 - 22 December 2013


1 Let n = pa11 pa22 pat t be the standard factorisation of n. Define (n) = t and (n) =
a1 + a2 + . . . + at . Prove or disprove: For any fixed positive integer k and positive reals ,
(n+k)
there exists a positive integer n > 1 such that 1) (n+k)
(n) > 2) (n) <
2 Let f : X X, where X = {1, 2, . . . , 100}, be a function satisfying: 1) f (x) 6= x for all
x = 1, 2, . . . , 100; 2) for any subset A of X such that |A| = 40, we have A f (A) 6= . Find
the minimum k such that for any such function f , there exist a subset B of X, where |B| = k,
such that B f (B) = X.
3 For non-empty number sets S, T , define the sets S + T = {s + t | s S, t T } and
2S = {2s | s S}. Let n be a positive integer, and A, B be two non-empty subsets of
{1, 2 . . . , n}. Show that there exists a subset D of A + B such that 1) D + D 2(A + B), 2)
|D| |A||B|
2n , where |X| is the number of elements of the finite set X.

21 This file was downloaded from the AoPS Math Olympiad Resources Page
http://www.artofproblemsolving.com/

Page 4

China
Northern Mathematical Olympiad
2013

2 If a1 , a2 , , a2013 [2, 2] and a1 + a2 + + a2013 = 0 , find the maximum of a31 + a32 +


+ a32013 .
5 Find all non-integers x such that x +
where n x.

13
x

= [x] +

13
[x] .where[x]

mean the greatest integer n ,

7 Suppose that {an } is a sequence such that an+1 = (1 + nk )an + 1 with a1 = 1.Find all positive
integers k such that any an be integer.

This file was downloaded from the AoPS Math Olympiad Resources Page
http://www.artofproblemsolving.com/

22

Page 1

China
South East Mathematical Olympiad
2013

Day 1 - 27 July 2013


1 Let a, b be real numbers such that the equation x3 ax2 + bx a = 0 has three positive real
3
.
roots . Find the minimum of 2a 3ab+3a
b+1
2 4ABC, AB > AC. the incircle I of 4ABC meet BC at point D, AD meet I again at E.
EP is a tangent of I, and EP meet the extension line of BC at P . CF k P E, CF AD = F .
the line BF meet I at M, N , point M is on the line segment BF , the line segment P M meet
I again at Q. Show that EN P = EN Q
3 A sequence {an } , a1 = 1, a2 = 2, an+1 =

a2n + (1)n
. Show that a2m + a2m+1 {an }, m N
an1

4 There are 12 acrobats who are assigned a distinct number (1, 2, , 12) respectively. Half of
them stand around forming a circle (called circle A); the rest form another circle (called circle
B) by standing on the shoulders of every two adjacent acrobats in circle A respectively. Then
circle A and circle B make up a formation. We call a formation a tower if the number of any
acrobat in circle B is equal to the sum of the numbers of the two acrobats whom he stands
on. How many heterogeneous towers are there? (Note: two towers are homogeneous if either
they are symmetrical or one may become the other one by rotation. We present an example
of 8 acrobats (see attachment). Numbers inside the circle represent the circle A; numbers
outside the circle represent the circle B. All these three formations are towers, however they
are homogeneous towers.)

This file was downloaded from the AoPS Math Olympiad Resources Page
http://www.artofproblemsolving.com/

23

Page 1

China
South East Mathematical Olympiad
2013

Day 2 - 28 July 2013

5 f (x) =

2013
P hxi

. A integer n is called good if f (x) = n has real root. How many good
i!
numbers are in {1, 3, 5, . . . , 2013}?
i=1

6 n > 1 is an integer. The first n primes are p1 = 2, p2 = 3, . . . , pn . Set A = pp11 pp22 ...ppnn . Find
A
A
all positive integers x, such that
is even, and
has exactly x divisors
x
x
7 Given a 3 3 grid, we call the remainder of the grid an angle when a 2 2 grid is cut out
from the grid. Now we place some angles on a 10 10 grid such that the borders of those
angles must lie on the grid lines or its borders, moreover there is no overlap among the angles.
Determine the maximal value of k, such that no matter how we place k angles on the grid,
we can always place another angle on the grid.
8 n 3 is a integer. , , (0, 1). For every ak , bk , ck 0(k = 1, 2, . . . , n) with
,

n
P

(k+)bk ,

k=1

n
P

(k+)ck , we always have

k=1

n
P

n
P

(k+)ak

k=1

(k+)ak bk ck . Find the minimum

k=1

of

24 This file was downloaded from the AoPS Math Olympiad Resources Page
http://www.artofproblemsolving.com/

Page 2

China
Team Selection Test
2013

Day 1 - 13 March 2013


1 The quadrilateral ABCD is inscribed in circle . F is the intersection point of AC and BD.
BA and CD meet at E. Let the projection of F on AB and CD be G and H, respectively.
Let M and N be the midpoints of BC and EF , respectively. If the circumcircle of 4M N G
only meets segment BF at P , and the circumcircle of 4M N H only meets segment CF at
Q, prove that P Q is parallel to BC.



2 For the positive integer n, define f (n) = min 2 m
n . Let {ni } be a strictly increasing
mZ

C
for all i {1, 2, . . .}.
n2i
for all i {1, 2, . . .}.

sequence of positive integers. C is a constant such that f (ni ) <


Show that there exists a real number q > 1 such that ni > q i1

3 There aren balls numbered 1, 2, , n, respectively. They are painted with 4 colours, red,
yellow, blue, and green, according to the following rules: First, randomly line them on a circle.
Then let any three clockwise consecutive balls numbered i, j, k, in order. 1) If i > j > k,
then the ball j is painted in red; 2) If i < j < k, then the ball j is painted in yellow; 3) If
i < j, k < j, then the ball j is painted in blue; 4) If i > j, k > j, then the ball j is painted
in green. And now each permutation of the balls determine a painting method. We call two
painting methods distinct, if there exists a ball, which is painted with two different colours
in that two methods.
Find out the number of all distinct painting methods.

This file was downloaded from the AoPS Math Olympiad Resources Page
http://www.artofproblemsolving.com/

25

Page 1

China
Team Selection Test
2013

Day 2 - 14 March 2013


1 Let n and k be two integers which are greater than 1. Let a1 , a2 , . . . , an , c1 , c2 , . . . , cm be
non-negative real numbers such that i) a1 a2 . . . an and a1 + a2 + . . . + an = 1; ii) For
any integer m {1, 2, . . . , n}, we have that c1 + c2 + . . . + cm mk . Find the maximum of
c1 ak1 + c2 ak2 + . . . + cn akn .
2 Let P be a given point inside the triangle ABC. Suppose L, M, N are the midpoints of
BC, CA, AB respectively and
P L : P M : P N = BC : CA : AB.
The extensions of AP, BP, CP meet the circumcircle of ABC at D, E, F respectively. Prove
that the circumcentres of AP F, AP E, BP F, BP D, CP D, CP E are concyclic.
3 Find all positive real numbers r < 1 such that there exists a set S with the given properties:
i) For any real number t, exactly one of t, t + r and t + 1 belongs to S; ii) For any real number
t, exactly one of t, t r and t 1 belongs to S.

26 This file was downloaded from the AoPS Math Olympiad Resources Page
http://www.artofproblemsolving.com/

Page 2

China
Team Selection Test
2013

Day 3 - 18 March 2013


1 For a positive integer k 2 define Tk = {(x, y) | x, y = 0, 1, . . . , k 1} to be a collection of k 2
lattice points on the cartesian coordinate plane. Let d1 (k) > d2 (k) > be the decreasing
sequence of the distinct distances between any two points in Tk . Suppose Si (k) be the number
of distances equal to di (k). Prove that for any three positive integers m > n > i we have
Si (m) = Si (n).
2 Prove that: there exists a positive constant K, and an integer series {an }, satisfying: (1)
0 < a1 < a2 < < an < ; (2) For any positive integer n, an < 1.01n K; (3) For any finite
number of distinct terms in {an }, their sum is not a perfect square.
3 Let A be a set consisting of 6 points in the plane. denoted n(A) as the number of the unit
circles which meet at least three points of A. Find the maximum of n(A)

27 This file was downloaded from the AoPS Math Olympiad Resources Page
http://www.artofproblemsolving.com/

Page 3

China
Team Selection Test
2013

Day 4 - 19 March 2013


1 For a positive integer N > 1 with unique factorization N = p1 1 p2 2 pk k , we define
(N ) = 1 + 2 + + k .
Let a1 , a2 , . . . , an be positive integers and p(x) = (x + a1 )(x + a2 ) (x + an ) such that for
all positive integers k, (P (k)) is even. Show that n is an even number.
2 Find the greatest positive integer m with the following property: For every permutation
a1 , a2 , , an , of the set of positive integers, there exists positive integers i1 < i2 < <
im such that ai1 , ai2 , , aim is an arithmetic progression with an odd common difference.
3 Let n > 1 be an integer and let a0 , a1 , . . . , an be non-negative real numbers. Definite Sk =

Pk
k
i=0 i ai for k = 0, 1, . . . , n. Prove that
n1

1X 2
1
Sk 2
n
n
k=0

n
X
k=0

!2
Sk

4
(Sn S0 )2 .
45

28 This file was downloaded from the AoPS Math Olympiad Resources Page
http://www.artofproblemsolving.com/

Page 4

China
Team Selection Test
2013

Day 5 - 24 March 2013


1 Let n 2 be an integer. a1 , a2 , . . . , an are arbitrarily chosen positive integers with (a1 , a2 , . . . , an ) =
1. Let A = a1 + a2 + + an and (A, ai ) = di . Let (a2 , a3 , . . . , an ) = D1 , (a1 , a3 , . . . , an ) =
n Aa
Q
i
D2 , . . . , (a1 , a2 , . . . , an1 ) = Dn . Find the minimum of
d
D
i
i
i=1
\ and QP is the
2 The circumcircle of triangle ABC has centre O. P is the midpoint of BAC
diameter. Let I be the incentre of 4ABC and let D be the intersection of P I and BC.
The circumcircle of 4AID and the extension of P A meet at F . The point E lies on the
line segment P D such that DE = DQ. Let R, r be the radius of the inscribed circle and
2r
circumcircle of 4ABC, respectively. Show that if AEF = AP E, then sin2 BAC =
R
3 101 people, sitting at a round table in any order, had 1, 2, ..., 101 cards, respectively. A
transfer is someone give one card to one of the two people adjacent to him. Find the smallest
positive integer k such that there always can through no more than k times transfer, each
person hold cards of the same number, regardless of the sitting order.

29 This file was downloaded from the AoPS Math Olympiad Resources Page
http://www.artofproblemsolving.com/

Page 5

China
Team Selection Test
2013

Day 6 - 25 March 2013


1 Let p be a prime number and a, k be positive integers such that pa < k < 2pa . Prove that
there exists a positive integer n such that
n < p2a , Cnk n k

(mod pa ).

2 Let k 2 be an integer and let a1 , a2 , , an , b1 , b2 , , bn be non-negative real numbers.


Prove that
!
!2

n1
n
n
n
Y
1
n
1X 2
1X
ai +
bi

(a2i + b2i ) n .
n1
n
n
i=1

i=1

i=1

3 A point (x, y) is a lattice point if x, y Z. Let E = {(x, y) : x, y Z}. In the coordinate


plane, P and Q are both sets of points in and on the boundary of a convex polygon with
vertices on lattice points. Let T = P Q. Prove that if T 6= and T E = , then T is a
non-degenerate convex quadrilateral region.

30 This file was downloaded from the AoPS Math Olympiad Resources Page
http://www.artofproblemsolving.com/

Page 6

China
Western Mathematical Olympiad
2013

1 Does there exist any integer a, b, c such that a2 bc + 2, ab2 c + 2, abc2 + 2 are perfect squares?
2 Let the integer n 2, and the real numbers x1 , x2 , , xn [0, 1].Prove that
n

X
1k<jn

kxk xj

n1X
kxk .
3
k=1

4 There are n coins in a row, n 2. If one of the coins is head, select an odd number of
consecutive coins (or even 1 coin) with the one in head on the leftmost, and then flip all the
selected coins upside down simultaneously. This is a move. No move is allowed if all n coins
are tails. Suppose m 1 coins are heads at the initial stage, determine if there is a way to
m
carry out b 23 c moves
5 A nonempty set A is called an n-level-good set if A {1, 2, 3, . . . , n} and |A| minxA x
(where |A| denotes the number of elements in A and minxA x denotes the minimum of the
elements in A). Let an be the number of n-level-good sets. Prove that for all positive integers
n we have an+2 = an+1 + an + 1.
7 Label sides of a regular n-gon in clockwise direction in order 1,2,..,n. Determine all integers n
(n 4) satisfying the following conditions: (1) n 3 non-intersecting diagonals in the n-gon
are selected, which subdivide the n-gon into n 2 non-overlapping triangles; (2) each of the
chosen n 3 diagonals are labeled with an integer, such that the sum of labeled numbers on
three sides of each triangles in (1) is equal to the others;
8 Find all positive integers a such that for any positive integer n 5 we have 2n n2 | an na .

This file was downloaded from the AoPS Math Olympiad Resources Page
http://www.artofproblemsolving.com/

31

Page 1

Finland
Finnish National High School Mathematics Competition
2013

1 The coefficients a, b, c of a polynomial f : R R, f (x) = x3 + ax2 + bx + c are mutually


distinct integers and different from zero. Furthermore, f (a) = a3 and f (b) = b3 . Determine
a, b and c.
2 In a particular European city, there are only 7 day tickets and 30 day tickets to the public
transport. The former costs 7.03 euro and the latter costs 30 euro. Aina the Algebraist
decides to buy at once those tickets that she can travel by the public transport the whole
three year (2014-2016, 1096 days) visiting in the city. What is the cheapest solution?
3 The points A, B, and C lies on the circumference of the unit circle. Furthermore, it is known
that AB is a diameter of the circle and
3
|AC|
= .
|CB|
4
The bisector of ABC intersects the circumference at the point D. Determine the length of
the AD.
4 A subset E of the set {1, 2, 3, . . . , 50} is said to be special if it does not contain any pair of
the form {x, 3x}. A special set E is superspecial if it contains as many elements as possible.
How many element there are in a superspecial set and how many superspecial sets there are?
5 Find all integer triples (m, p, q) satisfying
2m p2 + 1 = q 5
where m > 0 and both p and q are prime numbers.

This file was downloaded from the AoPS Math Olympiad Resources Page
http://www.artofproblemsolving.com/

32

Page 1

Germany
Bundeswettbewerb Mathematik
2013

Round 1

1 Is it possible to partition the set S = {1, 2, . . . , 21} into subsets that in each of these subsets
the largest number is equal to the sum of the other numbers?
2 Is it possible to partition a triangle, with line segments, into exactly five isosceles triangles?
All the triangles in concern are assumed to be nondegenerated triangles.
3 In the interior of the square ABCD, the point P lies in such a way that DCP = CAP =
25 . Find all possible values of P BA.
4 Two players A and B play the following game taking alternate moves. In each move, a player
writes one digit on the blackboard. Each new digit is written either to the right or left of the
sequence of digits already written on the blackboard. Suppose that A begins the game and
initially the blackboard was empty. B wins the game if ,after some move of B, the sequence
of digits written in the blackboard represents a perfect square. Prove that A can prevent B
from winning.

This file was downloaded from the AoPS Math Olympiad Resources Page
http://www.artofproblemsolving.com/

33

Page 1

Germany
Bundeswettbewerb Mathematik
2013

Round 2

1 Suppose m and n are positive integers such that m2 + n2 + m is divisible by mn. Prove that
m is a square number.
2 A parallelogram of paper with sides 25 and 10 is given. The distance between the longer sides
is 6. The paper should be cut into exactly two parts in such a way that one can stick both
the pieces together and fold it in a suitable manner to form a cube of suitable edge length
without any further cuts and overlaps. Show that it is really possible and describe such a
fragmentation.
3 Let ABCDEF be a convex hexagon whose vertices lie on a circle. Suppose that AB CD
EF = BC DE F A. Show that the diagonals AD, BE and CF are concurrent.
4 Consider the Pascals triangle in the figure where the binomial coefficients are arranged in
the usual manner. Select any binomial coefficient from anywhere except the right edge of the
triangle and labet it C. To the right of C, in the horizontal line, there are t numbers, we
denote them as a1 , a2 , , at , where at = 1 is the last number of the series. Consider the line
parallel to the left edge of the triangle containing C, there will only be t numbers diagonally
above C in that line. We successively name them as b1 , b2 , , bt , where bt = 1. Show that
bt a1 bt1 a2 + bt2 a3 + (1)t1 b1 at = 1

. For example, Suppose you choose 41 = 4 (see figure), then t = 3, a1 = 6, a2 = 4, a3 = 1
and b1 = 3, b2 = 2, b3 = 1.
1
1
1
1
1
...

b3

b1

4
...

b2

a1

...

1
1

a2

...

a3

...

...

34 This file was downloaded from the AoPS Math Olympiad Resources Page
http://www.artofproblemsolving.com/

Page 2

Greece
National Olympiad
2013

1 Let the sequence of real numbers (an ), n = 1, 2, 3... with a1 = 2 and an =


2. Find the term a2013 .

n+1
n1

(a1 + a2 + ... + an1 ) , n

2 Solve in integers the following equation:


y = 2x2 + 5xy + 3y 2
3 We define the sets A1 , A2 , ..., A160 such that |Ai | = i for all i = 1, 2, ..., 160. With the elements
of these sets we create new sets M1 , M2 , ...Mn by the following procedure: in the first step we
choose some of the sets A1 , A2 , ..., A160 and we remove from each of them the same number
of elements. These elements that we removed are the elements of M1 . In the second step we
repeat the same procedure in the sets that came of the implementation of the first step and
so we define M2 . We continue similarly until there are no more elements in A1 , A2 , ..., A160 ,
thus defining the sets M1 , M2 , ..., Mn . Find the minimum value of n.
4 Let a triangle ABC inscribed in circle c(O, R) and D an arbitrary point on BC(different from
the midpoint).The circumscribed circle of BOD,which is (c1 ), meets c(O, R) at K and AB
at Z.The circumscribed circle of COD (c2 ),meets c(O, R) at M and AC at E.Finally, the
circumscribed circle of AEZ (c3 ),meets c(O, R) at N .Prove that 4ABC = 4KM N .

This file was downloaded from the AoPS Math Olympiad Resources Page
http://www.artofproblemsolving.com/

35

Page 1

Hong kong
National Olympiad
2013

1 Let a, b, c be positive real numbers such that ab + bc + ca = 1. Prove that


s
s
s

1
3
3
3
4
4
4
+ 6 3b +
+ 6 3c +
+ 6 3a
a
b
c
abc
When does inequality hold?
2 For any positive integer a, define M (a) to be the number of positive integers b for which
a + b divides ab. Find all integer(s) a with 1 a 2013 such that M (a) attains the largest
possible value in the range of a.
3 Let ABC be a triangle with CA > BC > AB. Let O and H be the circumcentre and
orthocentre of triangle ABC respectively. Denote by D and E the midpoints of the arcs AB
and AC of the circumcircle of triangle ABC not containing the opposite vertices. Let D0 be
the reflection of D about AB and E 0 the reflection of E about AC. Prove that O, H, D0 , E 0
are concylic if and only if A, D0 , E 0 are collinear.
4 In a chess tournament there are n > 2 players. Every two players play against each other
exactly once. It is known that exactly n games end as a tie. For any set S of players, including
A and B, we say that A admires B in that set if i) A does not beat B; or ii) there exists a
sequence of players C1 , C2 , . . . , Ck in S, such that A does not beat C1 , Ck does not beat B,
and Ci does not beat Ci+1 for 1 i k 1. A set of four players is said to be harmonic
if each of the four players admires everyone else in the set. Find, in terms of n, the largest
possible number of harmonic sets.

This file was downloaded from the AoPS Math Olympiad Resources Page
http://www.artofproblemsolving.com/

36

Page 1

Hungary
Krschk Math Competition
2013

1 Let a, b be positive real numbers satisfying 2ab = a b. Denote for any positive integer k xk
and yk to be the closest integer to ak and bk, respectively (if there are two closest integers,
choose the larger one). Prove that any positive integer n appears in the sequence (xk )k1 if
and only if it appears at least three times in the sequence (yk )k1 .
2 Consider the closed polygonal discs P1 , P2 , P3 with the property that for any three points
A P1 , B P2 , C P3 , we have [4ABC] 1. (Here [X] denotes the area of polygon X.)
(a) Prove that min{[P1 ], [P2 ], [P3 ]} < 4. (b) Give an example of polygons P1 , P2 , P3 with the
above property such that [P1 ] > 4 and [P2 ] > 4.
3 Is it true that for integer n 2, and given any non-negative reals `ij , 1 i < j n, we can
find P
a sequenceP
0 a1 , a2 , . . . , an such that for all 1 i < j n to have |ai aj | `ij , yet
still ni=1 ai 1i<jn `ij ?

This file was downloaded from the AoPS Math Olympiad Resources Page
http://www.artofproblemsolving.com/

37

Page 1

India
International Mathematical Olympiad Training Camp
2013

Practice Test

Day 1 - 05 May 2013


1 For a prime p, a natural number n and an integer a, we let Sn (a, p) denote the exponent of
n
p in the prime factorisation of ap 1. For example, S1 (4, 3) = 2 and S2 (6, 2) = 0. Find all
pairs (n, p) such that Sn (2013, p) = 100.
2 Let ABCD by a cyclic quadrilateral with circumcenter O. Let P be the point of intersection
of the diagonals AC and BD, and K, L, M, N the circumcenters of triangles AOP, BOP ,
COP, DOP , respectively. Prove that KL = M N .
3 We define an operation on the set {0, 1} by
0 0 = 0,0 1 = 1,1 0 = 1,1 1 = 0.
For two natural numbers a and b, which are written in base 2 as a = (a1 a2 . . . ak )2 and
b = (b1 b2 . . . bk )2 (possibly with leading 0s), we define a b = c where c written in base 2 is
(c1 c2 . . . ck )2 with ci = ai bi , for 1 i k. For example, we have 7 3 = 4 since 7 = (111)2
and 3 = (011)2 .
For a natural number n, let f (n) = n [n/2], where [x] denotes the largest integer less than
or equal to x. Prove that f is a bijection on the set of natural numbers.

Day 2 - 10 May 2013


1 Let a, b, c be positive real numbers such that a + b + c = 1. If n is a positive integer then
prove that
(3a)n
(3b)n
(3c)n
27
+
+

.
(b + 1)(c + 1) (c + 1)(a + 1) (a + 1)(b + 1)
16
2 In a triangle ABC with B = 90 , D is a point on the segment BC such that the inradii of
\ = then prove that tan2 (/2) = tan(C/2).
triangles ABD and ADC are equal. If ADB
3 A marker is placed at the origin of an integer lattice. Calvin and Hobbes play the following
game. Calvin starts the game and each of them takes turns alternatively. At each turn, one
can choose two (not necessarily distinct) integers a, b, neither of which was chosen earlier by
any player and move the marker by a units in the horizontal direction and b units in the

This file was downloaded from the AoPS Math Olympiad Resources Page
http://www.artofproblemsolving.com/

38

Page 1

India
International Mathematical Olympiad Training Camp
2013

vertical direction. Hobbes wins if the marker is back at the origin any time after the first
turn. Prove or disprove that Calvin can prevent Hobbes from winning.
Note: A move in the horizontal direction by a positive quantity will be towards the right, and
by a negative quantity will be towards the left (and similar directions in the vertical case as
well).

39 This file was downloaded from the AoPS Math Olympiad Resources Page
http://www.artofproblemsolving.com/

Page 2

India
International Mathematical Olympiad Training Camp
2013

Team Selection Test

Day 1 - 15 May 2013


1 Let n 2 be an integer. There are n beads numbered 1, 2, . . . , n. Two necklaces made out
of some of these beads are considered the same if we can get one by rotating the other (with
no flipping allowed). For example, with n 5, the necklace with four beads 1, 5, 3, 2 in the
clockwise order is same as the one with 5, 3, 2, 1 in the clockwise order, but is different from
the one with 1, 2, 3, 5 in the clockwise order.
We denote by D0 (n) (respectively D1 (n)) the number of ways in which we can use all the
beads to make an even number (resp. an odd number) of necklaces each of length at least 3.
Prove that n 1 divides D1 (n) D0 (n).
b > 90 , let O and H denote its circumcenter and orthocenter,
2 In a triangle ABC, with A
respectively. Let K be the reflection of H with respect to A. Prove that K, O and C are
bB
b = 90 .
collinear if and only if A
3 For a positive integer n, a cubic polynomial p(x) is said to be n-good if there exist n distinct
integers a1 , a2 , . . . , an such that all the roots of the polynomial p(x) + ai = 0 are integers for
1 i n. Given a positive integer n prove that there exists an n-good cubic polynomial.

Day 2 - 16 May 2013


1 Find all functions f from the set of real numbers to itself satisfying
f (x(1 + y)) = f (x)(1 + f (y))
for all real numbers x, y.
2 An integer a is called friendly if the equation (m2 + n)(n2 + m) = a(m n)3 has a solution
over the positive integers. a) Prove that there are at least 500 friendly integers in the set
{1, 2, . . . , 2012}. b) Decide whether a = 2 is friendly.
3 Players A and B play a game with N 2012 coins and 2012 boxes arranged around a circle.
Initially A distributes the coins among the boxes so that there is at least 1 coin in each box.
Then the two of them make moves in the order B, A, B, A, . . . by the following rules: (a) On
every move of his B passes 1 coin from every box to an adjacent box. (b) On every move of

40 This file was downloaded from the AoPS Math Olympiad Resources Page
http://www.artofproblemsolving.com/

Page 3

India
International Mathematical Olympiad Training Camp
2013

hers A chooses several coins that were not involved in Bs previous move and are in different
boxes. She passes every coin to and adjacent box. Player As goal is to ensure at least 1 coin
in each box after every move of hers, regardless of how B plays and how many moves are
made. Find the least N that enables her to succeed.

Day 3 - 22 May 2013


1 For a positive integer n, a sum-friendly odd partition of n is a sequence (a1 , a2 , . . . , ak ) of
odd positive integers with a1 a2 ak and a1 + a2 + + ak = n such that for all
positive integers m n, m can be uniquely written as a subsum m = ai1 + ai2 + + air .
(Two subsums ai1 + ai2 + + air and aj1 + aj2 + + ajs with i1 < i2 < < ir and
j1 < j2 < < js are considered the same if r = s and ail = ajl for 1 l r.) For example,
(1, 1, 3, 3) is a sum-friendly odd partition of 8. Find the number of sum-friendly odd partitions
of 9999.
2 In a triangle ABC, let I denote its incenter. Points D, E, F are chosen on the segments
BC, CA, AB, respectively, such that BD + BF = AC and CD + CE = AB. The circumcircles of triangles AEF, BF D, CDE intersect lines AI, BI, CI, respectively, at points K, L, M
(different from A, B, C), respectively. Prove that K, L, M, I are concyclic.
3 Let h 3 be an integer and X the set of all positive integers that are greater than or equal
to 2h. Let S be a nonempty subset of X such that the following two conditions hold:
if a + b S with a h, b h, then ab S; [/*:m] if ab S with a h, b h, then
a + b S.[/*:m] Prove that S = X.

Day 4 - 23 May 2013


1 A positive integer a is called a double number if it has an even number of digits (in base 10)
and its base 10 representation has the form a = a1 a2 ak a1 a2 ak with 0 ai 9 for
1 i k, and a1 6= 0. For example, 283283 is a double number. Determine whether or not
there are infinitely many double numbers a such that a + 1 is a square and a + 1 is not a
power of 10.
2 Let n 2 be an integer and f1 (x), f2 (x), . . . , fn (x) a sequence of polynomials with integer
coefficients. One is allowed to make moves M1 , M2 , . . . as follows: in the k-th move Mk one
chooses an element f (x) of the sequence with degree of f at least 2 and replaces it with

41 This file was downloaded from the AoPS Math Olympiad Resources Page
http://www.artofproblemsolving.com/

Page 4

India
International Mathematical Olympiad Training Camp
2013

(f (x) f (k))/(x k). The process stops when all the elements of the sequence are of degree
1. If f1 (x) = f2 (x) = = fn (x) = xn + 1, determine whether or not it is possible to make
appropriate moves such that the process stops with a sequence of n identical polynomials of
degree 1.
3 In a triangle ABC, with AB 6= BC, E is a point on the line AC such that BE is perpendicular
to AC. A circle passing through A and touching the line BE at a point P 6= B intersects the
line AB for the second time at X. Let Q be a point on the line P B different from P such
that BQ = BP . Let Y be the point of intersection of the lines CP and AQ. Prove that the
points C, X, Y, A are concyclic if and only if CX is perpendicular to AB.

42 This file was downloaded from the AoPS Math Olympiad Resources Page
http://www.artofproblemsolving.com/

Page 5

India
ISI Entrance Examination
2013

1 Let a, b, c be real number greater than 1. Let


S = loga bc + logb ca + logc ab
Find the minimum possible value of S.
2 For x 0, define
f (x) =

1
x + 2 cos x

Find the set {y R : y = f (x), x 0}


3 Let f : R R satisfy

|f (x + y) f (x y) y| y 2

For all (x, y) R2 . Show that f (x) =

x
2

+ c where c is a constant.

4 In a badminton tournament, each of n players play all the other n 1 players. Each game
results in either a win, or a loss. The players then write down the names of those whom
they defeated, and also of those who they defeated. For example, if A beats B and B beats
C, then A writes the names of both B and C. Show that there will be one person, who has
written down the names of all the other n 1 players. [hide=Clarification] Consider a game
between A, B, C, D, E, F, G where A defeats B and C and B defeats E, F , C defeats E. Then
As list will have (B, C, E, F ), and will not include G.
5 Let AD be a diameter of a circle of radius r, and let B, C be points on the circle such that
AB = BC = 2r and A 6= C. Find the ratio CD
r .
6 Let p(x) and q(x) be two polynomials, both of which have their sum of coefficients equal to
s. Let p, q satisfy p(x)3 q(x)3 = p(x3 ) q(x3 ). Show that (i) There exists an integer a 1
and a polynomial r(x) with r(1) 6= 0 such that
p(x) q(x) = (x 1)a r(x).
(ii) Show that s2 = 3a1 , where a is described as above.
7 Find all natural numbers N for which N (N 101) is a perfect square.
8 Let ABCD be a square such that AB lies along the line y = x + 8, and C and D lie on the
parabola y = x2 . Find all possible values of sidelength of the square.

This file was downloaded from the AoPS Math Olympiad Resources Page
http://www.artofproblemsolving.com/

43

Page 1

India
National Olympiad
2013

1 Let 1 and 2 be two circles touching each other externally at R. Let O1 and O2 be the
centres of 1 and 2 , respectively. Let `1 be a line which is tangent to 2 at P and passing
through O1 , and let `2 be the line tangent to 1 at Q and passing through O2 . Let K = `1 `2 .
If KP = KQ then prove that the triangle P QR is equilateral.
2 Find all m, n N and primes p 5 satisfying
m(4m2 + m + 12) = 3(pn 1).
3 Let a, b, c, d N such that a b c d. Show that the equation x4 ax3 bx2 cx d = 0
has no integer solution.
4 Let N be an integer greater than 1 and let Tn be the number of non empty subsets S of
{1, 2, ....., n} with the property that the average of the elements of S is an integer.Prove that
Tn n is always even.
5 In an acute triangle ABC, let O, G, H be its circumcentre, centroid and orthocenter. Let
D BC, E CA and OD BC, HE CA. Let F be the midpoint of AB. If the triangles
ODC, HEA, GF B have the same area, find all the possible values of C.
6 Let a, b, c, x, y, z be six positive real numbers satisfying x + y + z = a + b + c and xyz = abc.
Further, suppose that a x < y < z c and a < b < c. Prove that a = x, b = y and c = z.

This file was downloaded from the AoPS Math Olympiad Resources Page
http://www.artofproblemsolving.com/

44

Page 1

India
Regional Mathematical Olympiad
2013

Mumbai Region

1 Let ABC be an isosceles triangle with AB = AC and let denote its circumcircle. A point
D is on arc AB of not containing C. A point E is on arc AC of not containing B. If
AD = CE prove that BE is parallel to AD.
2 Find all triples (p, q, r) of primes such that pq = r + 1 and 2(p2 + q 2 ) = r2 + 1.
3 A finite non-empty set of integers is called 3-good if the sum of its elements is divisible by 3.
Find the number of 3-good subsets of {0, 1, 2, . . . , 9}.
4 In a triangle ABC, points D and E are on segments BC and AC such that BD = 3DC and
AE = 4EC. Point P is on line ED such that D is the midpoint of segment EP . Lines AP
and BC intersect at point S. Find the ratio BS/SD.
5 Let a1 , b1 , c1 be natural numbers. We define
a2 = gcd(b1 , c1 ),

b2 = gcd(c1 , a1 ),

c2 = gcd(a1 , b1 ),

a3 = lcm(b2 , c2 ),

b3 = lcm(c2 , a2 ),

c3 = lcm(a2 , b2 ).

and
Show that gcd(b3 , c3 ) = a2 .
6 Let P (x) = x3 + ax2 + b and Q(x) = x3 + bx + a, where a and b are nonzero real numbers.
Suppose that the roots of the equation P (x) = 0 are the reciprocals of the roots of the
equation Q(x) = 0. Prove that a and b are integers. Find the greatest common divisor of
P (2013! + 1) and Q(2013! + 1).

This file was downloaded from the AoPS Math Olympiad Resources Page
http://www.artofproblemsolving.com/

45

Page 1

India
Regional Mathematical Olympiad
2013

Region 1

1 Let ABC be an acute-angled triangle. The circle with BC as diameter intersects AB and
AC again at P and Q, respectively. Determine BAC given that the orthocenter of triangle
AP Q lies on .
2 Let f (x) = x3 + ax2 + bx + c and g(x) = x3 + bx2 + cx + a, where a, b, c are integers with
c 6= 0. Suppose that the following conditions hold:
f (1) = 0,[/*:m] the roots of g(x) = 0 are the squares of the roots of f (x) = 0.[/*:m]
Find the value of a2013 + b2013 + c2013 .
3 Find all primes p and q such that p divides q 2 4 and q divides p2 1.
4 Find the number of 10-tuples (a1 , a2 , . . . , a9 , a10 ) of integers such that |a1 | 1 and
a21 + a22 + a23 + + a210 a1 a2 a2 a3 a3 a4 a9 a10 a10 a1 = 2.
5 Let ABC be a triangle with A = 90 and
AB = AC. Let D and E be points on the segment
BC such that BD : DE : EC = 1 : 2 : 3. Prove that DAE = 45
6 Suppose that m and n are integers, such that both the quadratic equations x2 + mx n = 0
and x2 mx + n = 0 have integer roots. Prove that n is divisible by 6.

46 This file was downloaded from the AoPS Math Olympiad Resources Page
http://www.artofproblemsolving.com/

Page 2

India
Regional Mathematical Olympiad
2013

Region 2

1 Prove that there do not exist natural numbers x and y with x > 1 such that ,
x7 1
= y5 + 1
x1
2 In a triangle ABC, AD is the altitude from A, and H is the orthocentre. Let K be the centre
of the circle passing through D and tangent to BH at H. Prove that the line DK bisects
AC.
3 Consider the expression
20132 + 20142 + 20152 + + n2
Prove that there exists a natural number n > 2013 for which one can change a suitable
number of plus signs to minus signs in the above expression to make the resulting expression
equal 9999
4 Let ABC be a triangle with A = 90 and
AB = AC. Let D and E be points on the segment
BC such that BD : DE : EC = 1 : 2 : 3. Prove that DAE = 45
5 Let n 3 be a natural number and let P be a polygon with n sides. Let a1 , a2 , , an be
the lengths of sides of P and let p be its perimeter. Prove that
a1
a2
an
+
+ +
<2
p a1 p a2
p an
6 For a natural number n, let T (n) denote the number of ways we can place n objects of weights
1, 2, , n on a balance such that the sum of the weights in each pan is the same. Prove that
T (100) > T (99).

47 This file was downloaded from the AoPS Math Olympiad Resources Page
http://www.artofproblemsolving.com/

Page 3

India
Regional Mathematical Olympiad
2013

Region 3

1 Find the number of eight-digit numbers the sum of whose digits is 4


2 Find all 4-tuples (a, b, c, d) of natural numbers with a b c and a! + b! + c! = 3d
3 In an acute-angled triangle ABC with AB < AC, the circle touches AB at B and passes
through C intersecting AC again at D. Prove that the orthocentre of triangle ABD lies on
if and only if it lies on the perpendicular bisector of BC.
4 A polynomial is called Fermat polynomial if it can be written as the sum of squares of two
polynomials with integer coefficients. Suppose that f (x) is a Fermat polynomial such that
f (0) = 1000. Prove that f (x) + 2x is not a fermat polynomial
5 Let ABC be a triangle which it not right-angled. De
fine a sequence of triangles Ai Bi Ci , with i 0, as follows: A0 B0 C0 is the triangle ABC and,
for i 0, Ai+1 , Bi+1 , Ci+1 are the reflections of the orthocentre of triangle Ai Bi Ci in the sides
Bi Ci ,Ci Ai ,Ai Bi , respectively. Assume that Am = An for some distinct natural numbers
m, n. Prove that A = 60 .
6 Let n 4 be a natural number. Let A1 A2 An be a regular polygon and X = {1, 2, 3...., n}.
A subset {i1 , i2 , , ik } of X, with k 3 and i1 < i2 < < ik , is called a good subset if
the angles of the polygon Ai1 Ai2 Aik , when arranged in the increasing order, are in an
arithmetic progression. If n is a prime, show that a proper good subset of X contains exactly
four elements.

48 This file was downloaded from the AoPS Math Olympiad Resources Page
http://www.artofproblemsolving.com/

Page 4

India
Regional Mathematical Olympiad
2013

Region 4

1 Let be a circle with centre O. Let be another circle passing through O and intersecting
at points A and B. A diameter CD of intersects at a point P different from O. Prove
that AP C = BP D
2 Determine the smallest prime that does not divide any
five-digit number whose digits are in a strictly increasing order.
3 Given real numbers a, b, c, d, e > 1. Prove that
b2
c2
d2
e2
a2
+
+
+
+
20
c1 d1 e1 a1 b1
4 Let x be a non-zero real numbers such that x4 +
Prove that x + x1 is a rational number.

1
x4

and x5 +

1
x5

are both rational numbers.

5 In a triangle ABC, let H denote its orthocentre. Let P be the reflection of A with respect to
BC. The circumcircle of triangle ABP intersects the line BH again at Q, and the circumcircle
of triangle ACP intersects the line CH again at R. Prove that H is the incentre of triangle
P QR.
6 Suppose that the vertices of a regular polygon of 20 sides are coloured with three colours red, blue and green - such that there are exactly three red vertices. Prove that there are three
vertices A, B, C of the polygon having the same colour such that triangle ABC is isosceles.

49 This file was downloaded from the AoPS Math Olympiad Resources Page
http://www.artofproblemsolving.com/

Page 5

Indonesia
National Science Olympiad
2013

Day 1
1 In a 4 6 grid, all edges and diagonals are drawn (see attachment). Determine the number of
parallelograms in the grid that uses only the line segments drawn and none of its four angles
are right.
2 Let ABC be an acute triangle and be its circumcircle. The bisector of BAC intersects
at [another point] M . Let P be a point on AM and inside 4ABC. Lines passing P that are
parallel to AB and AC intersects BC on E, F respectively. Lines M E, M F intersects at
points K, L respectively. Prove that AM, BL, CK are concurrent.
M
3 Determine all positive real M such that for any positive reals a, b, c, at least one of a + , b +
ab
M
M
,c +
is greater than or equal to 1 + M .
bc
ca
4 Suppose p > 3 is a prime number and
S=

ijk

2i<j<kp1

Prove that S + 1 is divisible by p.

This file was downloaded from the AoPS Math Olympiad Resources Page
http://www.artofproblemsolving.com/

50

Page 1

Indonesia
National Science Olympiad
2013

Day 2
5 Let P be a quadratic (polynomial of degree two) with a positive leading coefficient and
negative discriminant. Prove that there exists three quadratics P1 , P2 , P3 such that: - P (x) =
P1 (x) + P2 (x) + P3 (x) - P1 , P2 , P3 have positive leading coefficients and zero discriminants
(and hence each has a double root) - The roots of P1 , P2 , P3 are different
6 A positive integer n is called strong if there exists a positive integer x such that xnx + 1 is
divisible by 2n .
a. Prove that 2013 is strong. b. If m is strong, determine the smallest y (in terms of m) such
that y my + 1 is divisible by 2m .
7 Let ABCD be a parallelogram. Construct squares ABC1 D1 , BCD2 A2 , CDA3 B3 , DAB4 C4
on the outer side of the parallelogram. Construct a square having B4 D1 as one of its sides and
it is on the outer side of AB4 D1 and call its center OA . Similarly do it for C1 A2 , D2 B3 , A3 C4
to obtain OB , OC , OD . Prove that AOA = BOB = COC = DOD .
8 Let A be a set of positive integers. A is called balanced if [and only if] the number of
3-element subsets of A whose elements add up to a multiple of 3 is equal to the number of
3-element subsets of A whose elements add up to not a multiple of 3.
a. Find a 9-element balanced set. b. Prove that no set of 2013 elements can be balanced.

51 This file was downloaded from the AoPS Math Olympiad Resources Page
http://www.artofproblemsolving.com/

Page 2

APMO 2013

1 Let ABC be an acute triangle with altitudes AD, BE, and CF , and let O be the center of
its circumcircle. Show that the segments OA, OF , OB, OD, OC, OE dissect the triangle
ABC into three pairs of triangles that have equal areas.
n2 + 1
2 Determine all positive integers n for which 2
is an integer. Here [r] denotes the
[ n] + 2
greatest integer less than or equal to r.
3 For 2k real numbers a1 , a2 , ..., ak , b1 , b2 , ..., bk define a sequence of numbers Xn by
Xn =

k
X
[ai n + bi ]

(n = 1, 2, ...).

i=1

If the sequence XN forms an arithmetic progression, show that


Here [r] denotes the greatest integer less than or equal to r.

Pk

i=1 ai

must be an integer.

4 Let a and b be positive integers, and let A and B be finite sets of integers satisfying (i) A and
B are disjoint; (ii) if an integer i belongs to either to A or to B, then either i + a belongs to
A or i b belongs to B. Prove that a |A| = b |B|. (Here |X| denotes the number of elements
in the set X.)
5 Let ABCD be a quadrilateral inscribed in a circle , and let P be a point on the extension
of AC such that P B and P D are tangent to . The tangent at C intersects P D at Q and
the line AD at R. Let E be the second point of intersection between AQ and . Prove that
B, E, R are collinear.

This file was downloaded from the AoPS Math Olympiad Resources Page
http://www.artofproblemsolving.com/

52

Page 1

Balkan MO 2013

1 In a triangle ABC, the excircle a opposite A touches AB at P and AC at Q, while the


excircle b opposite B touches BA at M and BC at N . Let K be the projection of C onto
M N and let L be the projection of C onto P Q. Show that the quadrilateral M KLP is cyclic.
(Bulgaria)
2 Determine all positive integers x, y and z such that x5 + 4y = 2013z .
(Serbia)
3 Let S be the set of positive real numbers. Find all functions f : S 3 S such that, for all
positive real numbers x, y, z and k, the following three conditions are satisfied:
(a) xf (x, y, z) = zf (z, y, x),
(b) f (x, ky, k 2 z) = kf (x, y, z),
(c) f (1, k, k + 1) = k + 1.
(United Kingdom)
4 In a mathematical competition, some competitors are friends; friendship is mutual, that is,
when A is a friend of B, then B is also a friend of A. We say that n 3 different competitors
A1 , A2 , . . . , An form a weakly-friendly cycle if Ai is not a friend of Ai+1 for 1 i n (where
An+1 = A1 ), and there are no other pairs of non-friends among the components of the cycle.
The following property is satisfied:
for every competitor C and every weakly-friendly cycle S of competitors not including C,
the set of competitors D in S which are not friends of C has at most one element
Prove that all competitors of this mathematical competition can be arranged into three rooms,
such that every two competitors in the same room are friends.
(Serbia)

This file was downloaded from the AoPS Math Olympiad Resources Page
http://www.artofproblemsolving.com/

53

Page 1

Baltic Way 2013

1 Let n be a positive integer. Assume that n numbers are to be chosen from the table
n 1
2n 1
..
..
.
.
(n 1)n (n 1)n + 1 n2 1
0
n
..
.

1
n+1
..
.

with no two of them from the same row or the same column. Find the maximal value of the
product of these n numbers.
2 Let k and n be positive integers and let x1 , x2 , , xk , y1 , y2 , , yn be distinct integers. A
polynomial P with integer coefficients satisfies
P (x1 ) = P (x2 ) = = P (xk ) = 54
P (y1 ) = P (y2 ) = = P (yn ) = 2013.
Determine the maximal value of kn.
3 Let R denote the set of real numbers. Find all functions f : R R such that
f (xf (y) + y) + f (f (x)) = f (yf (x) y) + y
for all x, y R
4 Prove that the following inequality holds for all positive real numbers x, y, z:
x3
y3
z3
x+y+z
+
+

.
y 2 + z 2 z 2 + x2 x2 + y 2
2
5 Numbers 0 and 2013 are written at two opposite vertices of a cube. Some real numbers are
to be written at the remaining 6 vertices of the cube. On each edge of the cube the difference
between the numbers at its endpoints is written. When is the sum of squares of the numbers
written on the edges minimal?
6 Santa Claus has at least n gifts for n children. For i {1, 2, ..., n}, the i-th child considers
xi > 0 of these items to be desirable. Assume that
1
1
+ +
1.
x1
xn
Prove that Santa Claus can give each child a gift that this child likes.

This file was downloaded from the AoPS Math Olympiad Resources Page
http://www.artofproblemsolving.com/

54

Page 1

Baltic Way 2013

7 A positive integer is written on a blackboard. Players A and B play the following game: in
each move one has to choose a proper divisor m of the number n written on the blackboard
(1 < m < n) and replaces n with n m. Player A makes the first move, then players move
alternately. The player who cant make a move loses the game. For which starting numbers
is there a winning strategy for player B?
8 There are n rooms in a sauna, each has unlimited capacity. No room may be attended by a
female and a male simultaneously. Moreover, males want to share a room only with males
that they dont know and females want to share a room only with females that they know.
Find the biggest number k such that any k couples can visit the sauna at the same time,
given that two males know each other if and only if their wives know each other.
9 In a country there are 2014 airports, no three of them lying on a line. Two airports are
connected by a direct flight if and only if the line passing through them divides the country
in two parts, each with 1006 airports in it. Show that there are no two airports such that one
can travel from the first to the second, visiting each of the 2014 airports exactly once.
11 In an acute triangle ABC with AC > AB, let D be the projection of A on BC, and let E
and F be the projections of D on AB and AC, respectively. Let G be the intersection point
of the lines AD and EF . Let H be the second intersection point of the line AD and the
circumcircle of triangle ABC. Prove that
AG AH = AD2
12 A trapezoid ABCD with bases AB and CD is such that the circumcircle of the triangle BCD
intersects the line AD in a point E, distinct from A and D. Prove that the circumcircle oF
the triangle ABE is tangent to the line BC.
13 All faces of a tetrahedron are right-angled triangles. It is known that three of its edges have
the same length s. Find the volume of the tetrahedron.
14 Circles and
of the same radius intersect in two points, one of which is P . Denote by A and B, respectively,
the points diametrically opposite to P on each of and
. A third circle of the same radius passes through P and intersects and
in the points X and Y , respectively. Show that the line XY is parallel to the line AB.
15 Four circles in a plane have a common center. Their radii form a strictly increasing arithmetic
progression. Prove that there is no square with each vertex lying on a different circle.

55

This file was downloaded from the AoPS Math Olympiad Resources Page
http://www.artofproblemsolving.com/

Page 2

Baltic Way 2013

16 We call a positive integer n delightful if there exists an integer k, 1 < k < n, such that
1 + 2 + + (k 1) = (k + 1) + (k + 2) + + n
Does there exist a delightful number N satisfying the inequalities
20132013 <

N
< 20132013 + 4?
20132013

17 Let c and n > c be positive integers. Marys teacher writes n positive integers on a blackboard.
Is it true that for all n and c Mary can always label the numbers written by the teacher by
a1 , . . . , an in such an order that the cyclic product (a1 a2 ) (a2 a3 ) (an1 an ) (an a1 )
would be congruent to either 0 or c modulo n?
18 Find all pairs (x, y) of integers such that y 3 1 = x4 + x2 .
19 Let a0 be a positive integer and an = 5an1 + 4 for all n 1. Can a0 be chosen so that a54
is a multiple of 2013?
20 Find all polynomials f with non-negative integer coefficients such that for all primes p and
positive integers n there exist a prime q and a positive integer m such that f (pn ) = q m .

56

This file was downloaded from the AoPS Math Olympiad Resources Page
http://www.artofproblemsolving.com/

Page 3

Benelux 2013
Dordrecht

1 Let n 3 be an integer. A frog is to jump along the real axis, starting at the point 0 and
making n jumps: one of length 1, one of length 2, . . . , one of length n. It may perform these
n jumps in any order. If at some point the frog is sitting on a number a 0, its next jump
must be to the right (towards the positive numbers). If at some point the frog is sitting on a
number a > 0, its next jump must be to the left (towards the negative numbers). Find the
largest positive integer k for which the frog can perform its jumps in such an order that it
never lands on any of the numbers 1, 2, . . . , k.
2 Find all functions f : R R such that
f (x + y) + y f (f (f (x)))
holds for all x, y R.
3 Let 4ABC be a triangle with circumcircle , and let I be the center of the incircle of 4ABC.
The lines AI, BI and CI intersect in D 6= A, E 6= B and F 6= C. The tangent lines to
in F , D and E intersect the lines AI, BI and CI in R, S and T , respectively. Prove that
|AR| |BS| |CT | = |ID| |IE| |IF |.
4 a) Find all positive integers g with the following property: for each odd prime number p there
exists a positive integer n such that p divides the two integers
gn n

and

g n+1 (n + 1).

b) Find all positive integers g with the following property: for each odd prime number p there
exists a positive integer n such that p divides the two integers
g n n2

and g n+1 (n + 1)2 .

This file was downloaded from the AoPS Math Olympiad Resources Page
http://www.artofproblemsolving.com/

57

Page 1

CentroAmerican 2013

Day 1
1 Juan writes the list of pairs (n, 3n ), with n = 1, 2, 3, ... on a chalkboard. As he writes the list,
he underlines the pairs (n, 3n ) when n and 3n have the same units digit. What is the 2013th
underlined pair?
2 Around a round table the people P1 , P2 , ..., P2013 are seated in a clockwise order. Each person
starts with a certain amount of coins (possibly none); there are a total of 10000 coins. Starting
with P1 and proceeding in clockwise order, each person does the following on their turn:
If they have an even number of coins, they give all of their coins to their neighbor to the
left. [/*:m] If they have an odd number of coins, they give their neighbor to the left an odd
number of coins (at least 1 and at most all of their coins) and keep the rest.[/*:m]
Prove that, repeating this procedure, there will necessarily be a point where one person has
all of the coins.
3 Let ABCD be a convex quadrilateral and let M be the midpoint of side AB. The circle
passing through D and tangent to AB at A intersects the segment DM at E. The circle
passing through C and tangent to AB at B intersects the segment CM at F . Suppose that
the lines AF and BE intersect at a point which belongs to the perpendicular bisector of side
AB. Prove that A, E, and C are collinear if and only if B, F , and D are collinear.

This file was downloaded from the AoPS Math Olympiad Resources Page
http://www.artofproblemsolving.com/

58

Page 1

CentroAmerican 2013

Day 2
1 Ana and Beatriz take turns in a game that starts with a square of side 1 drawn on an infinite
grid. Each turn consists of drawing a square that does not overlap with the rectangle already
drawn, in such a way that one of its sides is a (complete) side of the figure already drawn. A
player wins if she completes a rectangle whose area is a multiple of 5. If Ana goes first, does
either player have a winning strategy?
2 Let ABC be an acute triangle and let be its circumcircle. The bisector of A intersects
BC at D, at K (different from A), and the
line through B tangent to at X. Show that
AD
K is the midpoint of AX if and only if DC
= 2.
3 Determine all pairs of non-constant polynomials p(x) and q(x), each with leading coefficient
1, degree n, and n roots which are non-negative integers, that satisfy p(x) q(x) = 1.

59

This file was downloaded from the AoPS Math Olympiad Resources Page
http://www.artofproblemsolving.com/

Page 2

Cono Sur Olympiad 2013

Day 1
1 Four distinct points are marked in a line. For each point, the sum of the distances from said
point to the other three is calculated; getting in total 4 numbers.
Decide whether these 4 numbers can be, in some order: a) 29, 29, 35, 37 b) 28, 29, 35, 37 c)
28, 34, 34, 37
2 In a triangle ABC, let M be the midpoint of BC and I the incenter of ABC. If IM = IA,
find the least possible measure of AIM .
3 Nocycleland is a country with 500 cities and 2013 two-way roads, each one of them connecting
two cities. A city A neighbors B if there is one road that connects them, and a city A quasineighbors B if there is a city C such that A neighbors C and C neighbors B. It is known that
in Nocycleland, there are no pair of cities connected directly with more than one road, and
there are no four cities A, B, C and D such that A neighbors B, B neighbors C, C neighbors
D, and D neighbors A. Show that there is at least one city that quasi-neighbors at least 57
other cities.

This file was downloaded from the AoPS Math Olympiad Resources Page
http://www.artofproblemsolving.com/

60

Page 1

Cono Sur Olympiad 2013

Day 2
4 Let M be the set of all integers from 1 to 2013. Each subset of M is given one of k available
colors, with the only condition that if the union of two different subsets A and B is M , then
A and B are given different colors. What is the least possible value of k?
5 Let d(k) be the number of positive divisors of integer k. A number n is called balanced if
d(n 1) d(n) d(n + 1) or d(n 1) d(n) d(n + 1). Show that there are infinitely
many balanced numbers.
6 Let ABCD be a convex quadrilateral. Let n 2 be a whole number. Prove that there are n
triangles with the same area that satisfy all of the following properties:
a) Their interiors are disjoint, that is, the triangles do not overlap. b) Each triangle lies either
4n
in ABCD or inside of it. c) The sum of the areas of all of these triangles is at least 4n+1
the
area of ABCD.

61

This file was downloaded from the AoPS Math Olympiad Resources Page
http://www.artofproblemsolving.com/

Page 2

EGMO 2013

Day 1 - 10 April 2013


1 The side BC of the triangle ABC is extended beyond C to D so that CD = BC. The side
CA is extended beyond A to E so that AE = 2CA. Prove that, if AD = BE, then the
triangle ABC is right-angled.
2 Determine all integers m for which the m m square can be dissected into five rectangles,
the side lengths of which are the integers 1, 2, 3, . . . , 10 in some order.
3 Let n be a positive integer.
(a) Prove that there exists a set S of 6n pairwise different positive integers, such that the
least common multiple of any two elements of S is no larger than 32n2 .
(b) Prove that every set T of 6n pairwise different positive integers contains two elements the
least common multiple of which is larger than 9n2 .

This file was downloaded from the AoPS Math Olympiad Resources Page
http://www.artofproblemsolving.com/

62

Page 1

EGMO 2013

Day 2 - 11 April 2013


4 Find all positive integers a and b for which there are three consecutive integers at which the
polynomial
n5 + a
P (n) =
b
takes integer values.
5 Let be the circumcircle of the triangle ABC. The circle is tangent to the sides AC
and BC, and it is internally tangent to the circle at the point P . A line parallel to AB
intersecting the interior of triangle ABC is tangent to at Q.
Prove that ACP = QCB.
6 Snow White and the Seven Dwarves are living in their house in the forest. On each of 16
consecutive days, some of the dwarves worked in the diamond mine while the remaining
dwarves collected berries in the forest. No dwarf performed both types of work on the same
day. On any two different (not necessarily consecutive) days, at least three dwarves each
performed both types of work. Further, on the first day, all seven dwarves worked in the
diamond mine.
Prove that, on one of these 16 days, all seven dwarves were collecting berries.

63

This file was downloaded from the AoPS Math Olympiad Resources Page
http://www.artofproblemsolving.com/

Page 2

European Mathematical Cup 2013

Junior

1 For m N define m? be the product of first m primes. Determine if there exists positive
integers m, n with the following property :
m? = n(n + 1)(n + 2)(n + 3)
Proposed by Matko Ljulj
2 Let P be a point inside a triangle ABC. A line through P parallel to AB meets BC and
CA at points L and F , respectively. A line through P parallel to BC meets CA and BA
at points M and D respectively, and a line through P parallel to CA meets AB and BC at
points N and E respectively. Prove

Proposed by Steve Dinh


3 We are given a combination lock consisting of 6 rotating discs. Each disc consists of digits
0, 1, 2, . . . , 9 in that order (after digit 9 comes 0). Lock is opened by exactly one combination.
A move consists of turning one of the discs one digit in any direction and the lock opens instantly
if the current combination is correct. Discs are initially put in the position 000000, and we know
that this combination is not correct.
a) What is the least number of moves necessary to ensure that we have found the correct combination? b) What is the least number of moves necessary to ensure that we have found the
correct combination, if we know that none of the combinations 000000, 111111, 222222, . . . , 999999
is correct?
Proposed by Ognjen Stipeti and Grgur Valenti
4 Let a, b, c be positive reals satisfying :
a
b
c
ab
bc
ca
+
+

+
+
1+b+c 1+c+a 1+a+b
1+a+b 1+b+c 1+c+a
Then prove that :

a2 + b2 + c2
+ a + b + c + 2 2( ab + bc + ca)
ab + bc + ca
Proposed by Dimitar Trenevski

This file was downloaded from the AoPS Math Olympiad Resources Page
http://www.artofproblemsolving.com/

64

Page 1

European Mathematical Cup 2013

Senior

1 In each field of a table there is a real number. We call such n n table silly if each
entry equals the product of all the numbers in the neighbouring fields.
a) Find all 2 2 silly tables. b) Find all 3 3 silly tables.
2 Palindrome is a sequence of digits which doesnt change if we reverse the order of its
digits. Prove that a sequence (xn )
n=0 defined as
xn = 2013 + 317n
contains infinitely many numbers with their decimal expansions being palindromes.
3 We call a sequence of n digits one or zero a code. Subsequence of a code is a palindrome
if it is the same after we reverse the order of its digits. A palindrome is called nice if its
digits occur consecutively in the code. (Code (1101) contains 10 palindromes, of which
6 are nice.)
a) What is the least number of palindromes in a code?
b) What is the least number of nice palindromes in a code?
4 Given a triangle ABC let D, E, F be orthogonal projections from A, B, C to the
opposite sides respectively. Let X, Y , Z denote midpoints of AD, BE, CF respectively.
Prove that perpendiculars from D to Y Z, from E to XZ and from F to XY are
concurrent.

65

This file was downloaded from the AoPS Math Olympiad Resources Page
http://www.artofproblemsolving.com/

Page 2

IberoAmerican 2013

Day 1
1 A set S of positive integers is said to be channeler if for any three distinct numbers a, b, c S,
we have a | bc, b | ca, c | ab.
a) Prove that for any finite set of positive integers {c1 , c2 , . . . , cn } there exist infinitely many
positive integers k, such that the set {kc1 , kc2 , . . . , kcn } is a channeler set.
b) Prove that for any integer n 3 there is a channeler set who has exactly n elements, and
such that no integer greater than 1 divides all of its elements.
2 Let X and Y be the diameters extremes of a circunference and N be the midpoint of one
of the arcs XY of . Let A and B be two points on the segment XY . The lines N A and
N B cuts again in C and D, respectively. The tangents to at C and at D meets in P .
Let M the the intersection point between XY and N P . Prove that M is the midpoint of the
segment AB.
3 Let A = {1, ..., n} with n>5. Prove that one can find B a finite set of positive integers such
that A is a subset of B and
X
Y
x2 =
x
xB

xB

This file was downloaded from the AoPS Math Olympiad Resources Page
http://www.artofproblemsolving.com/

66

Page 1

IberoAmerican 2013

Day 2
4 Let be a circunference and O its center. AE is a diameter of and B the midpoint of one
of the arcs AE of . The point D 6= E in on the segment OE. The point C is such that the
quadrilateral ABCD is a parallelogram, with AB parallel to CD and BC parallel to AD.
The lines EB and CD meets at point F . The line OF cuts the minor arc EB of at I.
Prove that the line EI is the angle bissector of BEC.
5 Let A and B be two sets such that A B is the set of the positive integers, and A B is the
empty set. It is known that if two positive integers have a prime larger than 2013 as their
difference, then one of them is in A and the other is in B. Find all the possibilities for the
sets A and B.
6 A beautiful configuration of points is a set of n colored points, such that if a triangle with
vertices in the set has an angle of at least 120 degrees, then exactly 2 of its vertices are colored
with the same color. Determine the maximum possible value of n.

67

This file was downloaded from the AoPS Math Olympiad Resources Page
http://www.artofproblemsolving.com/

Page 2

International Zhautykov Olympiad 2013

Day 1 - 15 January 2013


1 Given a trapezoid ABCD (AD k BC) with ABC > 90 . Point M is chosen on the lateral
side AB. Let O1 and O2 be the circumcenters of the triangles M AD and M BC, respectively.
The circumcircles of the triangles M O1 D and M O2 C meet again at the point N . Prove that
the line O1 O2 passes through the point N .
2 Find all odd positive integers n > 1 such that there is a permutation a1 , a2 , a3 , . . . , an of the
numbers 1, 2, 3, . . . , n where n divides one of the numbers a2k ak+1 1 and a2k ak+1 + 1 for
each k, 1 k n (we assume an+1 = a1 ).
3 Let a, b, c, and d be positive real numbers such that abcd = 1. Prove that
(a 1)(c + 1) (b 1)(d + 1) (c 1)(a + 1) (d 1)(b + 1)
+
+
+
0.
1 + bc + c
1 + cd + d
1 + da + a
1 + ab + b
Proposed by Orif Ibrogimov, Uzbekistan.

This file was downloaded from the AoPS Math Olympiad Resources Page
http://www.artofproblemsolving.com/

68

Page 1

International Zhautykov Olympiad 2013

Day 2 - 16 January 2013


1 A quadratic trinomial p(x) with real coefficients is given. Prove that there is a positive integer
n such that the equation p(x) = n1 has no rational roots.
2 Given convex hexagon ABCDEF with AB k DE, BC k EF , and CD k F A . The distance
between the lines AB and DE is equal to the distance between the lines BC and EF and to
the distance between the lines CD and F A. Prove that the sum AD + BE + CF does not
exceed the perimeter of hexagon ABCDEF .
3 A 10 10 table consists of 100 unit cells. A block is a 2 2 square consisting of 4 unit cells of
the table. A set C of n blocks covers the table (i.e. each cell of the table is covered by some
block of C ) but no n 1 blocks of C cover the table. Find the largest possible value of n.

69

This file was downloaded from the AoPS Math Olympiad Resources Page
http://www.artofproblemsolving.com/

Page 2

Junior Balkan MO 2013

1 Find all ordered pairs (a, b) of positive integers for which the numbers
are both positive integers.

a3 b 1
b3 a + 1
and
a+1
b1

2 Let ABC be an acute-angled triangle with AB < AC and let O be the centre of its circumcircle
. Let D be a point on the line segment BC such that BAD = CAO. Let E be the
second point of intersection of and the line AD. If M , N and P are the midpoints of the
line segments BE, OD and AC, respectively, show that the points M , N and P are collinear.
3 Show that

a + 2b +

2
a+1


b + 2a +

2
b+1


16

for all positive real numbers a and b such that ab 1.


4 Let n be a positive integer. Two players, Alice and Bob, are playing the following game: Alice chooses n real numbers; not necessarily distinct. - Alice writes all pairwise sums on a
sheet of paper and gives it to Bob. (There are n(n1)
such sums; not necessarily distinct.) 2
Bob wins if he finds correctly the initial n numbers chosen by Alice with only one guess. Can
Bob be sure to win for the following cases?
a. n = 5 b. n = 6 c. n = 8
Justify your answer(s).
[For example, when n = 4, Alice may choose the numbers 1, 5, 7, 9, which have the same
pairwise sums as the numbers 2, 4, 6, 10, and hence Bob cannot be sure to win.]

This file was downloaded from the AoPS Math Olympiad Resources Page
http://www.artofproblemsolving.com/

70

Page 1

Lusophon Mathematical Olympiad 2013

1 If Xiluva puts two oranges in each basket, four oranges are in excess. If she puts five oranges
in each basket, one basket is in excess. How many oranges and baskets has Xiluva?
2 Let ABC be an acute triangle. The circumference with diameter AB intersects sides AC and
BC at E and F respectively. The tangent lines to the circumference at the points E and F
meet at P . Show that P belongs to the altitude from C of triangle ABC.
3 An event occurs many years ago. It occurs periodically in x consecutive years, then there is a
break of y consecutive years. We know that the event occured in 1964, 1986, 1996, 2008 and
it didnt occur in 1976, 1993, 2006, 2013. What is the first year in that the event will occur
again?
4 Find all the pairs (x, y) of positive integers that satisfy the equation x2 xy + 2x 3y = 2013.
5 Find all the numbers of 5 non-zero digits such that deleting consecutively the digit of the left,
in each step, we obtain a divisor of the previous number.
6 Consider a triangle ABC. Let S be a circumference in the interior of the triangle that is
tangent to the sides BC, CA, AB at the points D, E, F respectively. In the exterior of the
triangle we draw three circumferences SA , SB , SC . The circumference SA is tangent to BC
at L and to the prolongation of the lines AB, AC at the points M , N respectively. The
circumference SB is tangent to AC at E and to the prolongation of the line BC at P . The
circumference SC is tangent to AB at F and to the prolongation of the line BC at Q. Show
that the lines EP , F Q and AL meet at a point of the circumference S.

This file was downloaded from the AoPS Math Olympiad Resources Page
http://www.artofproblemsolving.com/

71

Page 1

Middle European Mathematical Olympiad 2013

Individual Competition

1 Let a, b, c be positive real numbers such that


a+b+c=

1
1
1
+ 2 + 2.
2
a
b
c

Prove that
2(a + b + c)

p
p
p
3
3
3
7a2 b + 1 + 7b2 c + 1 + 7c2 a + 1.

Find all triples (a, b, c) for which equality holds.


2 Let n be a positive integer. On a board consisting of 4n 4n squares, exactly 4n tokens are
placed so that each row and each column contains one token. In a step, a token is moved
horizontally of vertically to a neighbouring square. Several tokens may occupy the same
square at the same time. The tokens are to be moved to occupy all the squares of one of the
two diagonals. Determine the smallest number k(n) such that for any initial situation, we
can do it in at most k(n) steps.
3 Let ABC be an isosceles triangle with AC = BC. Let N be a point inside the triangle such
that 2AN B = 180 + ACB. Let D be the intersection of the line BN and the line parallel
to AN that passes through C. Let P be the intersection of the angle bisectors of the angles
CAN and ABN . Show that the lines DP and AN are perpendicular.
4 Let a and b be positive integers. Prove that there exist positive integers x and y such that


x+y
= ax + by.
2

This file was downloaded from the AoPS Math Olympiad Resources Page
http://www.artofproblemsolving.com/

72

Page 1

Middle European Mathematical Olympiad 2013

Team Competition

1 Find all functions f : R R such that


f (xf (x) + 2y) = f (x2 ) + f (y) + x + y 1
holds for all x, y R.
2 Let x, y, z, w be nonzero real numbers such that x + y 6= 0, z + w 6= 0, and xy + zw 0.
Prove that




x+y
y
z + w 1 1  x z 1
w 1
+
+
+
+
+
z+w
x+y
2
z
x
w
y
3 There are n 2 houses on the northern side of a street. Going from the west to the east, the
houses are numbered from 1 to n. The number of each house is shown on a plate. One day
the inhabitants of the street make fun of the postman by shuffling their number plates in the
following way: for each pair of neighbouring houses, the currnet number plates are swapped
exactly once during the day. How many different sequences of number plates are possible at
the end of the day?
4 Consider finitely many points in the plane with no three points on a line. All these points
can be coloured red or green such that any triangle with vertices of the same colour contains
at least one point of the other colour in its interior. What is the maximal possible number of
points with this property?
5 Let ABC be and acute triangle. Construct a triangle P QR such that AB = 2P Q, BC = 2QR,
CA = 2RP , and the lines P Q, QR, and RP pass through the points A, B, and C, respectively.
(All six points A, B, C, P, Q, and R are distinct.)
6 Let K be a point inside an acute triangle ABC, such that BC is a common tangent of the
circumcircles of AKB and AKC. Let D be the intersection of the lines CK and AB, and
let E be the intersection of the lines BK and AC . Let F be the intersection of the line BC
and the perpendicular bisector of the segment DE. The circumcircle of ABC and the circle
k with centre F and radius F D intersect at points P and Q. Prove that the segment P Q is
a diameter of k.
7 The numbers from 1 to 20132 are written row by row into a table consisting of 2013 2013
cells. Afterwards, all columns and all rows containing at least one of the perfect squares
1, 4, 9, , 20132 are simultaneously deleted. How many cells remain?

73

This file was downloaded from the AoPS Math Olympiad Resources Page
http://www.artofproblemsolving.com/

Page 2

Middle European Mathematical Olympiad 2013

8 The expression
     
is written on the blackboard. Tow players, A and B, play a game, taking turns. Player A
takes the first turn. In each turn, the player on turn replaces a symbol  by a positive integer.
After all the symbols  are replace, player A replaces each of the signs by either + or -,
independently of each other. Player A wins if the value of the expression on the blackboard
is not divisible by any of the numbers 11, 12, , 18. Otherwise, player B wins. Determine
which player has a winning strategy.

74

This file was downloaded from the AoPS Math Olympiad Resources Page
http://www.artofproblemsolving.com/

Page 3

Pan African 2013

Day 1 - 28 June 2013


1 A positive integer n is such that n(n + 2013) is a perfect square. a) Show that n cannot be
prime. b) Find a value of n such that n(n + 2013) is a perfect square.
2 Find all functions f : R R such that f (x)f (y) + f (x + y) = xy for all real numbers x and
y.
3 Let ABCDEF be a convex hexagon with A = D and B = E . Let K and L be the
midpoints of the sides AB and DE respectively. Prove that the sum of the areas of triangles
F AK, KCB and CF L is equal to half of the area of the hexagon if and only if
BC
EF
=
.
CD
FA

This file was downloaded from the AoPS Math Olympiad Resources Page
http://www.artofproblemsolving.com/

75

Page 1

Pan African 2013

Day 2
1 Let ABCD be a convex quadrilateral with AB parallel to CD. Let P and Q be the midpoints
of AC and BD, respectively. Prove that if ABP = CBD, then BCQ = ACD.
2 The cells of an n n board with n 5 are coloured black or white so that no three adjacent
squares in a row, column or diagonal are the same colour. Show that for any 3 3 square
within the board, two of its corner squares are coloured black and two are coloured white.
3 Let x, y, and z be real numbers such that x < y < z < 6. Solve the system of inequalities:

1
1

+
2
yx zy
1

+2x
6z

76

This file was downloaded from the AoPS Math Olympiad Resources Page
http://www.artofproblemsolving.com/

Page 2

Rioplatense Mathematical Olympiad, Level 3 2013

Day 1
1 Let a, b, c, d be real positive numbers such that a2 + b2 + c2 + d2 = 1. Prove that (1 a)(1
b)(1 c)(1 d) abcd.
2 Let ABCD be a square, and let E and F be points in AB and BC respectively such that
BE = BF . In the triangle EBC, let N be the foot of the altitude relative to EC. Let G
be the intersection between AD and the extension of the previously mentioned altitude. F G
and EC intersect at point P , and the lines N F and DC intersect at point T . Prove that the
line DP is perpendicular to the line BT .
3 A division of a group of people into various groups is called k-regular if the number of groups
is less or equal to k and two people that know each other are in different groups. Let A, B,
and C groups of people such that there are is no person in A and no person in B that know
each other. Suppose that the group A C has an a-regular division and the group B C has
a b-regular division. For each a and b, determine the least possible value of k for which it is
guaranteed that the group A B C has a k-regular division.

This file was downloaded from the AoPS Math Olympiad Resources Page
http://www.artofproblemsolving.com/

77

Page 1

Rioplatense Mathematical Olympiad, Level 3 2013

Day 2
4 Two players A and B play alternatively in a convex polygon with n 5 sides. In each
turn, the corresponding player has to draw a diagonal that does not cut inside the polygon
previously drawn diagonals. A player loses if after his turn, one quadrilateral is formed such
that its two diagonals are not drawn. A starts the game. For each positive integer n, find a
winning strategy for one of the players.
5 Find all positive integers n for which there exist two distinct numbers of n digits, a1 a2 . . . an
and b1 b2 . . . bn , such that the number of 2n digits a1 a2 . . . an b1 b2 . . . bn is divisible by b1 b2 . . . bn a1 a2 . . . an .
6 Let ABC be an acute scalene triangle, H its orthocenter and G its geocenter. The circumference with diameter AH cuts the circumcircle of BHC in A0 (A0 6= H). Points B 0 and C 0
are defined similarly. Show that the points A0 , B 0 , C 0 , and G lie in one circumference.

78

This file was downloaded from the AoPS Math Olympiad Resources Page
http://www.artofproblemsolving.com/

Page 2

Romanian Masters In Mathematics 2013

Day 1 - 01 March 2013


1 For a positive integer a, define a sequence of integers x1 , x2 , . . . by letting x1 = a and xn+1 =
2xn + 1 for n 1. Let yn = 2xn 1. Determine the largest possible k such that, for some
positive integer a, the numbers y1 , . . . , yk are all prime.
2 Does there exist a pair (g, h) of functions g, h : R R such that the only function f : R R
satisfying f (g(x)) = g(f (x)) and f (h(x)) = h(f (x)) for all x R is identity function f (x)
x?
3 Let ABCD be a quadrilateral inscribed in a circle . The lines AB and CD meet at P ,
the lines AD and BC meet at Q, and the diagonals AC and BD meet at R. Let M be the
midpoint of the segment P Q, and let K be the common point of the segment M R and the
circle . Prove that the circumcircle of the triangle KP Q and are tangent to one another.

This file was downloaded from the AoPS Math Olympiad Resources Page
http://www.artofproblemsolving.com/

79

Page 1

Romanian Masters In Mathematics 2013

Day 2 - 02 March 2013


1 Suppose two convex quadrangles in the plane P and P 0 , share a point O such that, for every
line l trough O, the segment along which l and P meet is longer then the segment along which
l and P 0 meet. Is it possible that the ratio of the area of P 0 to the area of P be greater then
1.9?
2 Given a positive integer k 2, set a1 = 1 and, for every integer n 2, let an be the smallest
solution of equation
n1
X r x 
k
x=1+
ai
i=1

that exceeds an1 . Prove that all primes are among the terms of the sequence a1 , a2 , . . .
3 A token is placed at each vertex of a regular 2n-gon. A move consists in choosing an edge of
the 2n-gon and swapping the two tokens placed at the endpoints of that edge. After a
finite number of moves have been performed, it turns out that every two tokens have been
swapped exactly once. Prove that some edge has never been chosen.

80

This file was downloaded from the AoPS Math Olympiad Resources Page
http://www.artofproblemsolving.com/

Page 2

Tuymaada Olympiad 2013

Juniors

Day 1
1 100 heaps of stones lie on a table. Two players make moves in turn. At each move, a player
can remove any non-zero number of stones from the table, so that at least one heap is left
untouched. The player that cannot move loses. Determine, for each initial position, which of
the players, the first or the second, has a winning strategy.
K. Kokhas
EDIT. It is indeed confirmed by the sender that empty heaps are still heaps, so the third
post contains the right guess of an interpretation.
2 ABCDEF is a convex hexagon, such that in it AC k DF , BD k AE and CE k BF . Prove
that
AB 2 + CD2 + EF 2 = BC 2 + DE 2 + AF 2 .
N. Sedrakyan
3 For every positive real numbers a and b prove the inequality
r

1 a2 + b2 2 2
ab
+
.
3
2
31 1
+
a b
A. Khabrov
4 The vertices of a connected graph cannot be coloured with less than n + 1 colours (so that
n(n 1)
adjacent vertices have different colours). Prove that
edges can be removed from the
2
graph so that it remains connected.
V. Dolnikov
EDIT. It is confirmed by the official solution that the graph is tacitly assumed to be finite.

Day 2

This file was downloaded from the AoPS Math Olympiad Resources Page
http://www.artofproblemsolving.com/

81

Page 1

Tuymaada Olympiad 2013

5 Each face of a 7 7 7 cube is divided into unit squares. What is the maximum number of
squares that can be chosen so that no two chosen squares have a common point?
A. Chukhnov
6 Quadratic trinomials with positive leading coefficients are arranged in the squares of a 6 6
table. Their 108 coefficients are all integers from 60 to 47 (each number is used once). Prove
that at least in one column the sum of all trinomials has a real root.
K. Kokhas F. Petrov
7 Solve the equation p2 pq q 3 = 1 in prime numbers.
A. Golovanov
8 The point A1 on the perimeter of a convex quadrilateral ABCD is such that the line AA1
divides the quadrilateral into two parts of equal area. The points B1 , C1 , D1 are defined
similarly. Prove that the area of the quadrilateral A1 B1 C1 D1 is greater than a quarter of the
area of ABCD.
L. Emelyanov

82

This file was downloaded from the AoPS Math Olympiad Resources Page
http://www.artofproblemsolving.com/

Page 2

Tuymaada Olympiad 2013

Seniors
Day 1
1 100 heaps of stones lie on a table. Two players make moves in turn. At each move, a player
can remove any non-zero number of stones from the table, so that at least one heap is left
untouched. The player that cannot move loses. Determine, for each initial position, which of
the players, the first or the second, has a winning strategy.
K. Kokhas
EDIT. It is indeed confirmed by the sender that empty heaps are still heaps, so the third
post contains the right guess of an interpretation.
2 Points X and Y inside the rhombus ABCD are such that Y is inside the convex quadrilateral
BXDC and 2XBY = 2XDY = ABC. Prove that the lines AX and CY are parallel.
S. Berlov
3 The vertices of a connected graph cannot be coloured with less than n + 1 colours (so that
n(n 1)
edges can be removed from the
adjacent vertices have different colours). Prove that
2
graph so that it remains connected.
V. Dolnikov
EDIT. It is confirmed by the official solution that the graph is tacitly assumed to be finite.
4 Prove that if x, y, z are positive real numbers and xyz = 1 then
x3
y3
z3
3
+
+
.
2
2
2
x +y y +z z +x
2
A. Golovanov

Day 2
5 Prove that every polynomial of fourth degree can be represented in the form P (Q(x)) +
R(S(x)), where P, Q, R, S are quadratic trinomials.
A. Golovanov
EDIT. It is confirmed that assuming the coefficients to be real, while solving the problem,
earned a maximum score.

83

This file was downloaded from the AoPS Math Olympiad Resources Page
http://www.artofproblemsolving.com/

Page 3

Tuymaada Olympiad 2013

6 Solve the equation p2 pq q 3 = 1 in prime numbers.


A. Golovanov
7 Points A1 , A2 , A3 , A4 are the vertices of a regular tetrahedron of edge length 1. The points
B1 and B2 lie inside the figure bounded by the plane A1 A2 A3 and the spheres of radius 1 and
centres A1 , A2 , A3 . Prove that B1 B2 < max{B1 A1 , B1 A2 , B1 A3 , B1 A4 }.
A. Kupavsky
8 Cards numbered from 1 to 2n are distributed among k children, 1 k 2n , so that each
child gets at least one card. Prove that the number of ways to do that is divisible by 2k1
but not by 2k .
M. Ivanov

84

This file was downloaded from the AoPS Math Olympiad Resources Page
http://www.artofproblemsolving.com/

Page 4

Iran
National Math Olympiad (3rd Round)
2013

Algebra

1 Let a0 , a1 , . . . , an N. Prove that there exist positive integers b0 , b1 , . . . , bn such that for
0 i n : ai bi 2ai and polynomial
P (x) = b0 + b1 x + + bn xn
is irreducible over Q[x]. (10 points)
2 Real numbers a1 , a2 , . . . , an add up to zero. Find the maximum of a1 x1 + a2 x2 + + an xn in
term of ai s, when xi s vary in real numbers such that (x1 x2 )2 + (x2 x3 )2 + + (xn1
xn )2 1. (15 points)
3 For every positive integer n 2, Prove that there is no ntuple of distinct
Qcomplex numbers
(x1 , x2 , . . . , xn ) such that for each 1 k n following equality holds.
1in (xk xi ) =
i6=k
Q
(x
+
x
)
(20
points)
i
k
1in
i6=k

4 Find all functions f : R R such that f (0) Q and


f (x + f (y)2 ) = f (x + y)2 .
(25 points)
5 Prove that there is no polynomial P C[x] such that set {P (z) | |z| = 1} in complex plane
forms a polygon. In other words, a complex polynomial cant map the unit circle to a polygon.
(30 points)

This file was downloaded from the AoPS Math Olympiad Resources Page
http://www.artofproblemsolving.com/

85

Page 1

Iran
National Math Olympiad (3rd Round)
2013

Combinatorics
1 Assume that the following generating function equation is correct, prove the following state3i

6j+3 ) = 1 Statement: The number of partitions of n to numbers


ment:
i=1 (1+x )j=1 (1x
not of the form 6k+1 or 6k1 is equal to the number of partitions of n in which each summand
appears at least twice. (10 points)
Proposed by Morteza Saghafian
2 How many rooks can be placed in an n n chessboard such that each rook is threatened by
at most 2k rooks? (15 points)
Proposed by Mostafa Einollah zadeh
3 n cars are racing. At first they have a particular order. At each moment a car may overtake
another car. No two overtaking actions occur at the same time, and except moments a car is
passing another, the cars always have an order. A set of overtaking actions is called small
if any car overtakes at most once. A set of overtaking actions is called complete if any car
overtakes exactly once. If F is the set of all possible orders of the cars after a small set of
overtaking actions and G is the set of all possible orders of the cars after a complete set of
overtaking actions, prove that
| F |= 2 | G |
(20 points)
Proposed by Morteza Saghafian
4 We have constructed a rhombus by attaching two equal equilateral triangles. By putting
n 1 points on all 3 sides of each triangle we have divided the sides to n equal segments. By
drawing line segements between correspounding points on each side of the triangles we have
divided the rhombus into 2n2 equal triangles. We write the numbers 1, 2, . . . , 2n2 on these
triangles in a way no number appears twice. On the common segment of each two triangles
we write the positive difference of the numbers written on those triangles. Find the maximum
sum of all numbers written on the segments.
(25 points)
Proposed by Amirali Moinfar
5 Consider a graph with n vertices and 7n
4 edges. (a) Prove that there are two cycles of equal
length. (25 points) (b) Can you give a smaller function than 7n
4 that still fits in part (a)?
Prove your claim. We say function a(n) is smaller than b(n) if there exists an N such that
for each n > N ,a(n) < b(n) (At most 5 points)
Proposed by Afrooz Jabalameli

86 This file was downloaded from the AoPS Math Olympiad Resources Page
http://www.artofproblemsolving.com/

Page 2

Iran
National Math Olympiad (3rd Round)
2013

Final Exam

1 An n-stick is a connected figure consisting of n matches of length 1 which are placed horizontally or vertically and no two touch each other at points other than their ends. Two shapes
that can be transformed into each other by moving, rotating or flipping are considered the
same. An n-mino is a shape which is built by connecting n squares of side length 1 on their
sides such that theres a path on the squares between each two squares of the n-mino. Let
Sn be the number of n-sticks and Mn the number of n-minos, e.g. S3 = 5 And M3 = 2.
(a) Prove that for any natural n, Sn Mn+1 . (b) Prove that for large enough n we have
(2.4)n Sn (16)n . A grid segment is a segment on the plane of length 1 which its both
ends are integer points. A polystick is called wise if using it and its rotations or flips we
can cover all grid segments without overlapping, otherwise its called unwise. (c) Prove that
there are at least 2n6 different unwise n-sticks. (d) Prove that any polystick which is in form
of a path only going up and right is wise. (e) Extra points: Prove that for large enough n we
have 3n Sn 12n
Time allowed for this exam was 2 hours.
2 We define the distance between two circles , 0 by the length of the common external tangent
of the circles and show it by d(, 0 ). If two circles doesnt have a common external tangent
then the distance between them is undefined. A point is also a circle with radius 0 and the
distance between two cirlces can be zero. (a) Centroid. n circles 1 , . . . , n are fixed on the
plane. Prove that there exists a unique circle such that for each circle on the plane the
square of distance between and minus the sum of squares of distances of from each of
the i s 1 i n is constant, in other words:
d(, )2

1 Xn
d(i , )2 = constant
n
i=1

(b) Perpendicular Bisector. Suppose that the circle has the same distance from 1 , 2 .
Consider 3 a circle tangent to both of the common external tangents of 1 , 2 . Prove that
the distance of from centroid of 1 , 2 is not more than the distance of and 3 . (If the
distances are all defined) (c) Circumcentre. Let C be the set of all circles that each of
them has the same distance from fixed circles 1 , 2 , 3 . Prove that there exists a point on
the plane which is the external homothety center of each two elements of C. (d) Regular
Tetrahedron. Does there exist 4 circles on the plane which the distance between each two
of them equals to 1?
Time allowed for this problem was 150 minutes.

87 This file was downloaded from the AoPS Math Olympiad Resources Page
http://www.artofproblemsolving.com/

Page 3

Iran
National Math Olympiad (3rd Round)
2013

3 Real function f generates real function g if there exists a natural k such that f k = g and
we show this by f g. In this question we are trying to find some properties for relation ,
for example its trivial that if f g and g h then f h.(transitivity)
(a) Give an example of two real functions f, g such that f 6= g ,f g and g f . (b) Prove
that for each real function f there exists a finite number of real functions g such that f g
and g f . (c) Does there exist a real function g such that no function generates it, except
for g itself? (d) Does there exist a real function which generates both x3 and x5 ? (e) Prove
that if a function generates two polynomials of degree 1 P, Q then there exists a polynomial
R of degree 1 which generates P and Q.
Time allowed for this problem was 75 minutes.
4 A polygon A that doesnt intersect itself and has perimeter p is called Rotund if for each
two points x, y on the sides of this polygon which their distance on the plane is less than 1
their distance on the polygon is at most p4 . (Distance on the polygon is the length of smaller
path between two points on the polygon) Now we shall prove that we can fit a circle with
radius 41 in any rotund polygon. The mathematicians of two planets earth and Tarator have
two different approaches to prove the statement. In both approaches by inner chord we
mean a segment with both endpoints on the polygon, and diagonal is an inner chord with
vertices of the polygon as the endpoints. Earth approach: Maximal Chord We know
the fact that for every polygon, there exists an inner chord xy with a length of at most 1
such that for any inner chord x0 y 0 with length of at most 1 the distance on the polygon of
x, y is more than the distance on the polygon of x0 , y 0 . This chord is called the maximal
chord. On the rotund polygon A0 theres two different situations for maximal chord: (a)
Prove that if the length of the maximal chord is exactly 1, then a semicircle with diameter
maximal chord fits completely inside A0 , so we can fit a circle with radius 41 in A0 . (b) Prove
that if the length of the maximal chord is less than one we still can fit a circle with radius 14
in A0 . Tarator approach: Triangulation Statement 1: For any polygon that the length
of all sides is less than one and no circle with radius 41 fits completely inside it, there exists a
triangulation of it using diagonals such that no diagonal with length more than 1 appears in
the triangulation. Statement 2: For any polygon that no circle with radius 41 fits completely
inside it, can be divided into traingles that their sides are inner chords with length of at
most 1. The mathematicians of planet Tarator proved that if the second statement is true,
for each rotund polygon there exists a circle with radius 14 that fits completely inside it. (c)
Prove that if the second statement is true, then for each rotund polygon there exists a circle
with radius 41 that fits completely inside it. They found out that if the first statement is true
then the second statement is also true, so they put a bounty of a doogh on proving the first
statement. A young earth mathematician named J.N., found a counterexample for statement
1, thus recieving the bounty. (d) Find a 1392-gon that is counterexample for statement 1.
But the Tarators are not disappointed and they are still trying to prove the second statement.
(e) (Extra points) Prove or disprove the second statement.

88 This file was downloaded from the AoPS Math Olympiad Resources Page
http://www.artofproblemsolving.com/

Page 4

Iran
National Math Olympiad (3rd Round)
2013

Time allowed for this problem was 150 minutes.


5 A subsum of n real numbers a1 , . . . , an is a sum of elements of a subset of the set {a1 , . . . , an }.
In other words a subsum is 1 a1 + + n an in which for each 1 i n ,i is either 0 or
1. Years ago, there was a valuable list containing n real not necessarily distinct numbers and
their 2n 1 subsums. Some mysterious creatures from planet Tarator has stolen the list,
but we still have the subsums. (a) Prove that we can recover the numbers uniquely if all of
the subsums are positive. (b) Prove that we can recover the numbers uniquely if all of the
subsums are non-zero. (c) Prove that theres an example of the subsums for n = 1392 such
that we can not recover the numbers uniquely.
Note: If a subsum is sum of element of two different subsets, it appears twice. Time allowed
for this question was 75 minutes.
6 Planet Tarator is a planet in the Yoghurty way galaxy. This planet has a shape of convex
1392-hedron. On earth we dont have any other information about sides of planet tarator.
We have discovered that each side of the planet is a country, and has its own currency. Each
two neighbour countries have their own constant exchange rate, regardless of other exchange
rates. Anybody who travels on land and crosses the border must change all his money to the
currency of the destination country, and theres no other way to change the money. Incredibly,
a persons money may change after crossing some borders and getting back to the point he
started, but its guaranteed that crossing a border and then coming back doesnt change the
money. On a research project a group of tourists were chosen and given same amount of
money to travel around the Tarator planet and come back to the point they started. They
always travel on land and their path is a nonplanar polygon which doesnt intersect itself.
What is the maximum number of tourists that may have a pairwise different final amount of
money?
Note 1: Tourists spend no money during travel! Note 2: The only constant of the problem
is 1392, the number of the sides. The exchange rates and the way the sides are arranged are
unknown. Answer must be a constant number, regardless of the variables. Note 3: The
maximum must be among all possible polyhedras.
Time allowed for this problem was 90 minutes.
7 An equation P (x) = Q(y) is called Interesting if P and Q are polynomials with degree at
least one and integer coefficients and the equations has an infinite number of answers in N.
An interesting equation P (x) = Q(y) yields in interesting equation F (x) = G(y) if there
exists polynomial R(x) Q[x] such that F (x) R(P (x)) and G(x) R(Q(x)). (a) Suppose
that S is an infinite subset of N N.S is an answer of interesting equation P (x) = Q(y) if
each element of S is an answer of this equation. Prove that for each S theres an interesting
equation P0 (x) = Q0 (y) such that if there exists any interesting equation that S is an answer
of it, P0 (x) = Q0 (y) yields in that equation. (b) Define the degree of an interesting equation

89 This file was downloaded from the AoPS Math Olympiad Resources Page
http://www.artofproblemsolving.com/

Page 5

Iran
National Math Olympiad (3rd Round)
2013

P (x) = Q(y) by max{deg(P ), deg(Q)}. An interesting equation is called primary if theres


no other interesting equation with lower degree that yields in it. Prove that if P (x) = Q(y)
is a primary interesting equation and P and Q are monic then (deg(P ), deg(Q)) = 1.
Time allowed for this question was 2 hours.
8 Let A1 A2 A3 A4 A5 be a convex 5-gon in which the coordinates of all of its vertices are rational.
For each 1 i 5 define Bi the intersection of lines Ai+1 Ai+2 and Ai+3 Ai+4 . (Ai = Ai+5 )
Prove that at most 3 lines from the lines Ai Bi (1 i 5) are concurrent.
Time allowed for this problem was 75 minutes.

90 This file was downloaded from the AoPS Math Olympiad Resources Page
http://www.artofproblemsolving.com/

Page 6

Iran
National Math Olympiad (3rd Round)
2013

Geometry

1 Let ABCDE be a pentagon inscribe in a circle (O). Let BE AD = T . Suppose the parallel
line with CD which passes through T which cut AB, CE at X, Y . If be the circumcircle of
triangle AXY then prove that is tangent to (O).
2 Let ABC be a triangle with circumcircle (O). Let M, N be the midpoint of arc AB, AC
which does not contain C, B and let M 0 , N 0 be the point of tangency of incircle of 4ABC
with AB, AC. Suppose that X, Y are foot of perpendicular of A to M M 0 , N N 0 . If I is the
incenter of 4ABC then prove that quadrilateral AXIY is cyclic if and only if b + c = 2a.
3 Suppose line ` and four points A, B, C, D lies on `. Suppose that circles 1 , 2 passes through
A, B and circles 10 , 20 passes through C, D. If 1 10 and 2 20 then prove that lines
O1 O20 , O2 O10 , ` are concurrent where O1 , O2 , O10 , O20 are center of 1 , 2 , 10 , 20 .
4 In a triangle ABC with circumcircle (O) suppose that A-altitude cut (O) at D. Let altitude
of B, C cut AC, AB at E, F . H is orthocenter and T is midpoint of AH. Parallel line with
EF passes through T cut AB, AC at X, Y . Prove that XDF = Y DE.
5 Let ABC be triangle with circumcircle (O). Let AO cut (O) again at A0 . Perpendicular
bisector of OA0 cut BC at PA . PB , PC define similarly. Prove that :
I) Point PA , PB , PC are collinear.
II ) Prove that the distance of O from this line is equal to
circumcircle.

R
2

where R is the radius of the

91 This file was downloaded from the AoPS Math Olympiad Resources Page
http://www.artofproblemsolving.com/

Page 7

Iran
National Math Olympiad (3rd Round)
2013

Number Theory

1 Let p a prime number and d a divisor of p 1. Find the product of elements in Zp with order
d. ( mod p). (10 points)
2 Suppose that a, b are two odd positive integers such that 2ab + 1 | a2 + b2 + 1. Prove that
a = b. (15 points)
3 Let p > 3 a prime number. Prove that there exist x, y Z such that p = 2x2 + 3y 2 if and
only if p 5, 11 ( mod 24) (20 points)
4 Prime p = n2 + 1 is given. Find the sets of solutions to the below equation:
x2 (n2 + 1)y 2 = n2 .
(25 points)
5 p = 3k+ 1 is a prime number. For each m Zp , define function L as follow: L(m) =
P
x(x3 +m)
a) For every m Zp and t Zp prove that L(m) = L(mt3 ). (5 points)
xZp
p
b) Prove that there is a partition of Zp = A B C such that |A| = |B|= |C| = p1
3 and L
xA
a
x B . (7
on each set is constant. Equivalently there are a, b, c for which L(x) = b

c
xC
points)
c) Prove that a + b + c = 3. (4 points)
d) Prove that a2 + b2 + c2 = 6p + 3. (12 points)
e) Let X =
(2 points)

2a+b+3
,Y
3

ba
3 ,

show that X, Y Z and also show that :p = X 2 + XY + Y 2 .

(Zp = Zp \ {0})

92 This file was downloaded from the AoPS Math Olympiad Resources Page
http://www.artofproblemsolving.com/

Page 8

Iran
National Math Olympiad (Second Round)
2013

Day 1
1 Find all pairs (a, b) of positive integers for which gcd(a, b) = 1, and
a = 92 and b = 13, then b/a = 13.92 )

a
b

= b.a. (For example, if

2 Let n be a natural number and suppose that w1 , w2 , . . . , wn are n weights . We call the set
of {w1 , w2 , . . . , wn } to be a Perfect P
Set if we can achieve all of the 1, 2, . . . , W weights with
sums of w1 , w2 , . . . , wn , where W = ni=1 wi . Prove that if we delete the maximum weight of
a Perfect Set, the other weights make again a Perfect Set.
3 Let M be the midpoint of (the smaller) arc BC in circumcircle of triangle ABC. Suppose that
the altitude drawn from A intersects the circle at N . Draw two lines through circumcenter
O of ABC paralell to M B and M C, which intersect AB and AC at K and L, respectively.
Prove that N K = N L.

This file was downloaded from the AoPS Math Olympiad Resources Page
http://www.artofproblemsolving.com/

93

Page 1

Iran
National Math Olympiad (Second Round)
2013

Day 2
1 Let P be a point out of circle C. Let P A and P B be the tangents to the circle drawn from
C. Choose a point K on AB . Suppose that the circumcircle of triangle P BK intersects C
again at T . Let P 0 be the reflection of P with respect to A. Prove that
P BT = P 0 KA
2 Suppose a m n table. We write an integer in each cell of the table. In each move, we chose
a column, a row, or a diagonal (diagonal is the set of cells which the difference between their
row number and their column number is constant) and add either +1 or 1 to all of its cells.
Prove that if for all arbitrary 3 3 table we can change all numbers to zero, then we can
change all numbers of m n table to zero.
(Hint: First of all think about it how we can change number of 3 3 table to zero.)
3 Let {an }
n=1 be a sequence of positive integers for which

 

2an
2an+1
an+2 =
+
.
an+1
an
Prove that there exists a positive integer m such that am = 4 and am+1 {3, 4}.
Note. [x] is the greatest integer not exceeding x.

94 This file was downloaded from the AoPS Math Olympiad Resources Page
http://www.artofproblemsolving.com/

Page 2

Iran
Team Selection Test
2013

TST 1

Day 1
1 In acute-angled triangle ABC, let H be the foot of perpendicular from A to BC and also
suppose that J and I are excenters oposite to the side AH in triangles ABH and ACH. If
P is the point that incircle touches BC, prove that I, J, P, H are concyclic.
2 Find the maximum number of subsets from {1, ..., n} such that for any two of them like A, B
if A B then |B A| 3. (Here |X| is the number of elements of the set X.)
3 For nonnegative integers m and n, define the sequence a(m, n) of real numbers as follows.
Set a(0, 0) = 2 and for every natural number n, set a(0, n) = 1 and a(n, 0) = 2. Then for
m, n 1, define
a(m, n) = a(m 1, n) + a(m, n 1).
P
Prove that for every natural number k, all the roots of the polynomial Pk (x) = ki=0 a(i, 2k +
1 2i)xi are real.

Day 2
4 m and n are two nonnegative integers. In the Philosophers Chess, The chessboard is an
infinite grid of identical regular hexagons and a new piece named the Donkey moves on it as
follows:
Starting from one of the hexagons, the Donkey moves m cells in one of the 6 directions, then
it turns 60 degrees clockwise and after that moves n cells in this new direction until it reaches
its final cell.
At most how many cells are in the Philosophers chessboard such that one cannot go from
anyone of them to the other with a finite number of movements of the Donkey?
Proposed by Shayan Dashmiz
5 Do there exist natural numbers a, b and c such that a2 +b2 +c2 is divisible by 2013(ab+bc+ca)?
Proposed by Mahan Malihi

This file was downloaded from the AoPS Math Olympiad Resources Page
http://www.artofproblemsolving.com/

95

Page 1

Iran
Team Selection Test
2013

6 Points A, B, C and D lie on line l in this order. Two circular arcs C1 and C2 , which both
lie on one side of line l, pass through points A and B and two circular arcs C3 and C4 pass
through points C and D such that C1 is tangent to C3 and C2 is tangent to C4 . Prove that
the common external tangent of C2 and C3 and the common external tangent of C1 and C4
meet each other on line l.
Proposed by Ali Khezeli

96 This file was downloaded from the AoPS Math Olympiad Resources Page
http://www.artofproblemsolving.com/

Page 2

Iran
Team Selection Test
2013

TST 2

Day 1
7 Nonnegative real numbers p1 , . . . , pn and q1 , . . . , qn are such that p1 + + pn = q1 + + qn
Among all the matrices with nonnegative entries having pi as sum of the i-th rows entries
and qj as sum of the j-th columns entries, find the maximum sum of the entries on the main
diagonal.
8 Find all Arithmetic progressions a1 , a2 , ... of natural numbers for which there exists natural
number N > 1 such that for every k N:
a1 a2 ...ak | aN +1 aN +2 ...aN +k
9 find all functions f, g : R+ R+ such that f is increasing and also:
f (f (x) + 2g(x) + 3f (y)) = g(x) + 2f (x) + 3g(y)
g(f (x) + y + g(y)) = 2x g(x) + f (y) + y

Day 2
10 On each edge of a graph is written a real number,such that for every even tour of this
graph,sum the edges with signs alternatively positive and negative is zero.prove that one can
assign to each of the vertices of the graph a real number such that sum of the numbers on
two adjacent vertices is the number on the edge between them.(tour is a closed path from the
edges of the graph that may have repeated edges or vertices)
11 Let a, b, c be sides of a triangle such that a b c. prove that:
q
q
p

a(a + b ab) + b(a + c ac) + c(b + c bc) a + b + c


12 Let ABCD be a cyclic quadrilateral that inscribed in the circle .Let I1 , I2 and r1 , r2 be
incenters and radii of incircles of triangles ACD and ABC,respectively.assume that r1 = r2 .
let 0 be a circle that touches AB, AD and touches at T . tangents from A, T to meet at
the point K.prove that I1 , I2 , K lie on a line.

97 This file was downloaded from the AoPS Math Olympiad Resources Page
http://www.artofproblemsolving.com/

Page 3

Iran
Team Selection Test
2013

TST 3
Day 1
13 P is an arbitrary point inside acute triangle ABC. Let A1 , B1 , C1 be the reflections of point
P with respect to sides BC, CA, AB. Prove that the centroid of triangle A1 B1 C1 lies inside
triangle ABC.
14 we are given n rectangles in the plane. Prove that between 4n right angles formed by these

rectangles there are at least [4 n] distinct right angles.


15 a) Does there exist a sequence a1 < a2 < . . . of positive integers, such that there is a positive
integer N that m > N , am has exactly d(m) 1 divisors among ai s?
b) Does there exist a sequence a1 < a2 < . . . of positive integers, such that there is a positive
integer N that m > N , am has exactly d(m) + 1 divisors among ai s?

Day 2
16 The function f : Z Z has the property that for all integers m and n
f (m) + f (n) + f (f (m2 + n2 )) = 1.
We know that integers a and b exist such that f (a) f (b) = 3. Prove that integers c and d
can be found such that f (c) f (d) = 1.
Proposed by Amirhossein Gorzi
17 In triangle ABC, AD and AH are the angle bisector and the altitude of vertex A, respectively.
The perpendicular bisector of AD, intersects the semicircles with diameters AB and AC which
are drawn outside triangle ABC in X and Y , respectively. Prove that the quadrilateral
XY DH is concyclic.
Proposed by Mahan Malihi
18 A special kind of parallelogram tile is made up by attaching the legs of two right isosceles
triangles of side length 1. We want to put a number of these tiles on the floor of an n n
room such that the distance from each vertex of each tile to the sides of the room is an integer
and also no two tiles overlap. Prove that at least an area n of the room will not be covered
by the tiles.
Proposed by Ali Khezeli

98 This file was downloaded from the AoPS Math Olympiad Resources Page
http://www.artofproblemsolving.com/

Page 4

Italy
ITAMO
2013

1 A model car is tested on some closed circuit 600 meters long, consisting of flat stretches,
uphill and downhill. All uphill and downhill have the same slope. The test highlights the
following facts:
(a) The velocity of the car depends only on the fact that the car is driving along a stretch
of uphill, plane or downhill; calling these three velocities vs , vp and vd respectively, we have
vs < vp < vd ;
(b) vs , vp and vd , expressed in meter per second, are integers.
(c) Whatever may be the structure of the circuit, the time taken to complete the circuit is
always 50 seconds.
Find all possible values of vs , vp and vd .
2 In triangle ABC, suppose we have a > b, where a = BC and b = AC. Let M be the midpoint
of AB, and , are inscircles of the triangles ACM and BCM respectively. Let then A0 and
B 0 be the points of tangency of and on CM . Prove that A0 B 0 = ab
2 .
3 Each integer is colored with one of two colors, red or blue. It is known that, for every finite set
A of consecutive integers, the absolute value of the difference between the number of red and
blue integers in the set A is at most 1000. Prove that there exists a set of 2000 consecutive
integers in which there are exactly 1000 red numbers and 1000 numbers blue.
4 5654b is a power of a prime number. Find b if b > 6.
5 ABC is an isosceles triangle with AB = AC and the angle in A is less than 60 . Let D be
a point on AC such that DBC = BAC. E is the intersection between the perpendicular
bisector of BD and the line parallel to BC passing through A. F is a point on the line AC
such that F A = 2AC (A is between F and C). Show that EB and AC are parallel and that
the perpendicular from F to AB, the perpendicular from E to AC and BD are concurrent.
6 Two magicians are performing the following game. Initially the first magician encloses the
second magician in a cabin where he can neither see nor hear anything. To start the game,
the first magician invites Daniel, from the audience, to put on each square of a chessboard
n n, at his (Daniels) discretion, a token black or white. Then the first magician asks
Daniel to show him a square C of his own choice. At this point, the first magician chooses a
square D (not necessarily different from C) and replaces the token that is on D with other
color token (white with black or black with white). Then he opens the cabin in which the
second magician was held. Looking at the chessboard, the second magician guesses what is
the square C. For what value of n, the two magicians have a strategy such that the second
magician makes a successful guess.

This file was downloaded from the AoPS Math Olympiad Resources Page
http://www.artofproblemsolving.com/

99

Page 1

Kazakhstan
National Olympiad
2013

Grade 9

Day 1
1 On the board written numbers from 1 to 25 . Bob can pick any three of them say a, b, c and
replace by a3 + b3 + c3 . Prove that last number on the board can not be 20133 .
2 Prove that for all natural n there exists a, b, c such that n = gcd(a, b)(c2 ab) + gcd(b, c)(a2
bc) + gcd(c, a)(b2 ca).

Day 2
1 Given triangle ABC with incenter I. Let P,Q be point on circumcircle such that AP I =
CP I and BQI = CQI.Prove that BP, AQ and OI are concurrent.
2 a)Does there exist for any rational number ab some rational numbers x1 , x2 , ....xn such that
x1 x2 .... xn = 1 and x1 + x2 + .... + xn = ab a)Does there exist for any rational number ab
some rational numbers x1 , x2 , ....xn such that x1 x2 .... xn = ab and x1 + x2 + .... + xn = 1

This file was downloaded from the AoPS Math Olympiad Resources Page
http://www.artofproblemsolving.com/

100

Page 1

Kazakhstan
National Olympiad
2013

Grade 10

Day 1
1 On the board written numbers from 1 to 25 . Bob can pick any three of them say a, b, c and
replace by a3 + b3 + c3 . Prove that last number on the board can not be 20133 .
2 Let for natural numbers a, b, c and any natural n we have that (abc)n divides ((an 1)(bn
1)(cn 1) + 1)3 . Prove that then a = b = c.
3 Let ABCD be cyclic quadrilateral. Let AC and BD intersect at R, and let AB and CD
AM
CN
intersect at K. Let M and N are points on AB and CD such that M
B = N D . Let P and
Q be the intersections of M N with the diagonals of ABCD. Prove that circumcircles of
triangles KM N and P QR are tangent at a fixed point.

Day 2
1 Find maximum value of |a2 bc + 1| + |b2 ac + 1| + |c2 ba + 1| when a, b, c are reals in
[2; 2].
2 Let in triangle ABC incircle touches sides AB, BC, CA at C1 , A1 , B1 respectively. Let
1
1
BC1 + AC1 .Prove that if X is intersection of incircle and CC1 then 3CX = CC1

2
CA1

3 How many non-intersecting pairs of paths we have from (0,0) to (n,n) so that path can move
two ways:top or right?

101 This file was downloaded from the AoPS Math Olympiad Resources Page
http://www.artofproblemsolving.com/

Page 2

Kazakhstan
National Olympiad
2013

Grade 11

Day 1
1 Find all triples of positive integer (m, n, k) such that k m |mn 1 and k n |nm 1
2 Given triangle ABC with incenter I. Let P,Q be point on circumcircle such that AP I =
CP I and BQI = CQI.Prove that BP, AQ and OI are concurrent.
3 Consider the following sequence : a1 = 1; an =
2 an (n N)

a[ n
]
2
2

a[ n
]
3
3

+ ... +

n
a[ n
]
n .

Prove that a2n <

Day 2
1 Find maximum value of |a2 bc + 1| + |b2 ac + 1| + |c2 ba + 1| when a, b, c are reals in
[2; 2].
2 Let in triangle ABC incircle touches sides AB, BC, CA at C1 , A1 , B1 respectively. Let
1
1
BC1 + AC1 .Prove that if X is intersection of incircle and CC1 then 3CX = CC1

2
CA1

3 How many non-intersecting pairs of paths we have from (0,0) to (n,n) so that path can move
two ways:top or right?

102 This file was downloaded from the AoPS Math Olympiad Resources Page
http://www.artofproblemsolving.com/

Page 3

Korea
Final Round - Korea
2013

1 For a triangle 4ABC(B > C), D is a point on AC satisfying ABD = C. Let I be the
incenter of 4ABC, and circumcircle of 4CDI meets AI at E(6= I). The line passing E and
parallel to AB meets the line BD at P . Let J be the incenter of 4ABD, and A0 be the point
such that AI = IA0 . Let Q be the intersection point of JP and A0 C. Prove that QJ = QA0 .
2 Find all functions f : R R satisfying following conditions. (a) f (x) 0 for all x R. (b)
For a, b, c, d R with ab + bc + cd = 0, equality f (a b) + f (c d) = f (a) + f (b + c) + f (d)
holds.
3 For a positive integer n 2, define set T = {(i, j)|1 i < j n, i|j}. For nonnegative real
numbers x1 , x2 , , xn with x1 + x2 + + xn = 1, find the maximum value of
X
xi xj
(i,j)T

in terms of n.
4 For a triangle ABC, let B1 , C1 be the excenters of B, C. Line B1 C1 meets with the circumcircle of 4ABC at point D(6= A). E is the point which satisfies B1 ECA and C1 EAB.
Let w be the circumcircle of 4ADE. The tangent to the circle w at D meets AE at F .
G, H are the points on AE, w such that DGHAE. The circumcircle of 4HGF meets w at
point I(6= H), and J be the foot of perpendicular from D to AH. Prove that AI passes the
midpoint of DJ.
5 Two coprime positive integers a, b are given. Integer sequence {an }, {bn } satisties

(a + b 2)2n = an + bn 2
Find all prime numbers p such that there exist positive integer n p satisfying p|bn .
6 For any permutation f : {1, 2, , n} {1, 2, , n}, and define
A = {i|i > f (i)}
B = {(i, j)|i < j f (j) < f (i) or f (j) < f (i) < i < j}
C = {(i, j)|i < j f (i) < f (j) or f (i) < f (j) < i < j}
D = {(i, j)|i < j and f (i) > f (j)}
Prove that |A| + 2|B| + |C| = |D|.

This file was downloaded from the AoPS Math Olympiad Resources Page
http://www.artofproblemsolving.com/

103

Page 1

Korea
National Olympiad
2013

Day 1 - 10 November 2013


1 Let P be a point on segment BC. Q, R are points on AC, AB such that P Q k AB and
P R k AC. O, O1 , O2 are the circumcenters of triangle ABC, BP R, P CQ. The circumcircles
of BP R, P CQ meet at point K(6= P ). Prove that OO1 = KO2 .
2 Let a, b, c > 0 such that ab + bc + ca = 3. Prove that
X

(a + b)3

cyc

(2(a + b)(a2 + b2 )) 3

12

3 Prove that there exist monic polynomial f (x) with degree of 6 and having integer coefficients
such that (1) For all integer m, f (m) 6= 0. (2) For all positive odd integer n, there exist
positive integer k such that f (k) is divided by n.
4 {an } is a positive integer sequence such that ai+2 = ai+1 + ai (i 1). For positive integer n,
define {bn } as
4n2
1 X
bn =
ai
a2n+1
i=1

Prove that bn is positive integer, and find the general form of bn .

This file was downloaded from the AoPS Math Olympiad Resources Page
http://www.artofproblemsolving.com/

104

Page 1

Korea
National Olympiad
2013

Day 2 - 10 November 2013


5 Find all functions f : N N satisfying
f (mn) = lcm(m, n) gcd(f (m), f (n))
for all positive integer m, n.
6 Let O be circumcenter of triangle ABC. For a point P on segmet BC, the circle passing
through P, B and tangent to line AB and the circle passing through P, C and tangent to
line AC meet at point Q(6= P ). Let D, E be foot of perpendicular from Q to AB, AC.
(D 6= B, E 6= C) Two lines DE and BC meet at point R. Prove that O, P, Q are collinear if
and only if A, R, Q are collinear.
7 For positive integer k, define integer sequence {bn }, {cn } as follows:
b1 = c1 = 1
b2n = kb2n1 + (k 1)c2n1 , c2n = b2n1 + c2n1
b2n+1 = b2n + (k 1)c2n , c2n+1 = b2n + kc2n
Let ak = b2014 . Find the value of
100
X

(ak

p
1
ak 2 1) 2014

k=1

8 For positive integer a, b, c, d there are a + b + c + d points on plane which none of three are
collinear. Prove there exist two lines l1 , l2 such that (1) l1 , l2 are not parallel. (2) l1 , l2 do
not pass through any of a + b + c + d points. (3) There are a, b, c, d points on each region
separated by two lines l1 , l2 .

105 This file was downloaded from the AoPS Math Olympiad Resources Page
http://www.artofproblemsolving.com/

Page 2

Macedonia
JBMO TST - Macedonia
2013

1 Let x be a real number such that x3 and x2 +x are rational numbers. Prove that x is rational.
2 A triangle ABC is given, and a segment P Q = t on BC such that P is between B and Q
and Q is between P and C. Let P P1 ||AB, P1 is on AC, and P P2 ||AC, P2 is on AB. Points
Q1 and Q2 r defined similar. Prove that the sum of the areas of P QQ1 P1 and P QQ2 P2 does
not depend from the position of P Q on BC.

1
1
1
3 a, b, c > 0 and abc = 1. Prove that 21 ( a + b + c ) + 1+a
+ 1+b
+ 1+c
3. ( The
official problem is with abc = 1 but it can be proved without using it. )
4 A regular hexagon with side length 1 is given. There are m points in its interior such that no
3 are collinear. The hexagon is divided into triangles (triangulated), such that every point
of the m given and every vertex of the hexagon is a vertex of such a triangle. The triangles
dont have
common interior points. Prove that there exists a triangle with area not greater
3 3
than 4(m+2)
.
5 Let p, r be prime numbers, and q natural. Solve the equation (p + q + r)2 = 2p2 + 2q 2 + r2 .

This file was downloaded from the AoPS Math Olympiad Resources Page
http://www.artofproblemsolving.com/

106

Page 1

Macedonia
National Olympiad
2013

1 Let p, q, r be prime numbers. Solve the equation p2q + q 2p = r


2 2n coins are given to a couple of kids. Interchange of the coins occurs when some of the kids
has at least half of all the coins. Then from the coins of one of those kids to the all other kids
are given that much coins as the kid already had. In case when all the coins are at one kid
there is no possibility for interchange. What is the greatest possible number of consecutive
interchanges? (n is natural number)
3 Acute angle triangle is given such that BC is the longest side. Let E and G be the intersection
points from the altitude from A to BC with the circumscribed circle of triangle ABC and
BC respectively. Let the center O of this circle is positioned on the perpendicular line from
A to BE. Let EM be perpendicular to AC and EF be perpendicular to AB. Prove that the
area of F BEG is greater than the area of M F E.
4 Let a, b, c be positive real numbers such that a4 + b4 + c4 = 3. Prove that
9
9
9
+
+
a6 + b6 + c6 + 6
a2 + b4 + c6 a4 + b6 + c2 a6 + b2 + c4
5 An arbitrary triangle ABC is given. There are 2 lines, p and q, that are not parallel to each
other and they are not perpendicular to the sides of the triangle. The perpendicular lines
through points A, B and C to line p we denote with pa , pb , pc and the perpendicular lines to
line q we denote with qa , qb , qc . Let the intersection points of the lines pa , qa , pb , qb , pc and qc
with qb , pb , qc , pc , qa and pa are K, L, P, Q, N and M . Prove that KL, M N and P Q intersect
in one point.

This file was downloaded from the AoPS Math Olympiad Resources Page
http://www.artofproblemsolving.com/

107

Page 1

Mexico
National Olympiad
2013

1 All the prime numbers are written in order, p1 = 2, p2 = 3, p3 = 5, ... Find all pairs of positive
integers a and b with a b 2, such that pa pb divides 2(a b).
2 Let ABCD be a parallelogram with the angle at A obtuse. Let P be a point on segment BD.
The circle with center P passing through A cuts line AD at A and Y and cuts line AB at A
and X. Line AP intersects BC at Q and CD at R. Prove XP Y = XQY + XRY .
3 What is the largest amount of elements that can be taken from the set {1, 2, ..., 2012, 2013},
such that within them there are no distinct three, say a, b,and c, such that a is a divisor or
multiple of b c?
4 A n n n cube is constructed using 1 1 1 cubes, some of them black and others white,
such that in each n 1 1, 1 n 1, and 1 1 n subprism there are exactly two black
cubes, and they are separated by an even number of white cubes (possibly 0). Show it is
possible to replace half of the black cubes with white cubes such that each n 1 1, 1 n 1
and 1 1 n subprism contains exactly one black cube.
5 A pair of integers is special if it is of the form (n, n 1) or (n 1, n) for some positive
integer n. Let n and m be positive integers such that pair (n, m) is not special. Show
(n, m) can be expressed as a sum of two or more different special pairs if and only if n
and m satisfy the inequality n + m (n m)2 . Note: The sum of two pairs is defined as
(a, b) + (c, d) = (a + c, b + d).
6 Let A1 A2 ...A8 be a convex octagon such that all of its sides are equal and its opposite sides
are parallel. For each i = 1, ..., 8, define Bi as the intersection between segments Ai Ai+4 and
Ai1 Ai+1 , where Aj+8 = Aj and Bj+8 = Bj for all j. Show some number i, amongst 1, 2, 3,
and 4 satisfies
Ai Ai+4
3

Bi Bi+4
2

This file was downloaded from the AoPS Math Olympiad Resources Page
http://www.artofproblemsolving.com/

108

Page 1

Moldova
Team Selection Test
2013

Day 1
1 For any positive real numbers x, y, z, prove that

x
y

y
z

z
x

z(x+y)
y(y+z)

x(z+y)
z(x+z)

y(x+z)
x(x+y)

2 We call a triangle 4ABC, Q-angled if tan A, tan B, tan C Q, where A, B, C are


the interior angles of the triangle 4ABC. a) Prove that Q-angled triangles exist; b) Let
triangle 4ABC be Q-angled. Prove that for any non-negative integer n, numbers of the form
En = sinn A sinn B sinn C + cosn A cosn B cosn C are rational.
3 Let ABCD be a cyclic quadrilateral whose diagonals AC and BD meet at E. The extensions
of the sides AD and BC beyond A and B meet at F . Let G be the point such that ECGD
is a parallelogram, and H be the image of E under reflection in AD. Prove that D,H,F ,G
are concyclic.
4 Consider a positive real numberpa and a positive integer m. The sequence (xk )kZ+ is defined
as: x1 = 1, x2 = a, xn+2 = m+1 xm
n+1 xn . a) Prove that the sequence is converging. b) Find
limn xn .

This file was downloaded from the AoPS Math Olympiad Resources Page
http://www.artofproblemsolving.com/

109

Page 1

Moldova
Team Selection Test
2013

Day 2
1 Let m be the number of ordered solutions (a, b, c, d, e) satisfying: 1) a, b, c, d, e Z+ ; 2)
1
1
1
1
1
a + b + c + d + e = 1; Prove that m is odd.
1
2 Let an = 1 + n!( 0!1 + 1!1 + 2!1 + ... + n!
) for any n Z+ . Consider an points in the plane,no 3
of them collinear.The segments between any 2 of them are colored in one of n colors. Prove
that among them there exist 3 points forming a monochromatic triangle.

3 The diagonals of a trapezoid ABCD with AD k BC intersect at point P . Point Q lies


between the parallel lines AD and BC such that the line CD separates points P and Q, and
AQD = CQB. Prove that BQP = DAQ.
4 Prove that for any positive real numbers ai , bi , ci with i = 1, 2, 3, (a31 + b31 + c31 + 1)(a32 + b32 +
c32 + 1)(a33 + b33 + c33 + 1) 34 (a1 + b1 + c1 )(a2 + b2 + c2 )(a3 + b3 + c3 )

110 This file was downloaded from the AoPS Math Olympiad Resources Page
http://www.artofproblemsolving.com/

Page 2

Moldova
Team Selection Test
2013

Day 3
1 Let A = 20132013...2013 be formed by joining 2013, 165 times. Prove that 20132 | A.
(
2x2 + xy = 1
2 Find all pairs of real numbers (x, y) satisfying
3xy
9x2
= 1 + 2(1x)
2
2(1x)4
3 Consider the obtuse-angled triangle 4ABC and its side lengths a, b, c. Prove that a3 cos A+
b3 cos B + c3 cos C < abc.
4 Find the maximum number of disjoint pairs (xi , yi ) such that xi , yi {1, 2, ..., 2013}, xi + yi
2013 and for i 6= j xi + yi 6= xj + yj .

111 This file was downloaded from the AoPS Math Olympiad Resources Page
http://www.artofproblemsolving.com/

Page 3

Moldova
Team Selection Test
2013

Day 4
1 Consider real numbers x, y, z such that x, y, z > 0. Prove that


1
1
1
5
(xy + yz + xz)
+ 2
+ 2
> .
2
2
2
2
x +y
x +z
y +z
2
2 Consider a board on 2013 2013 squares, what is the maximum number of chess knights that
can be placed so that no 2 attack each other?
3 Consider the triangle 4ABC with AB 6= AC. Let point O be the circumcenter of 4ABC.
Let the angle bisector of BAC intersect BC at point D. Let E be the reflection of point
D across the midpoint of the segment BC. The lines perpendicular to BC in points D, E
intersect the lines AO, AD at the points X, Y respectively. Prove that the quadrilateral
B, X, C, Y is cyclic.
4 Let y be a positive integer. Prove that there are infinitely many prime numbers p such that
4 | p + 1 and p | 2n y + 1 for some positive integer n.

112 This file was downloaded from the AoPS Math Olympiad Resources Page
http://www.artofproblemsolving.com/

Page 4

Netherlands
IMO Team Selection Tests
2013

1 Show that

P2013

4026!
n=0 (n!(2013n)!)2

is a perfect square.

2 Let P be the point of intersection of the diagonals of a convex quadrilateral ABCD.Let X, Y, Z


AX
CZ
be points on the interior of AB, BC, CD respectively such that XB
= BY
Y C = ZD = 2.Suppose
that Y Z is tangent to the circumcircle of 4BXY .Show that AP D = XY Z. Edit:XY
changed to Y Z.
3 Fix a sequence a1 , a2 , a3 . . . of integers satisfying the following condition:for all prime numbers
p and all positive integers k,we have apk+1 = pak 3ap + 13.Determine all possible values of
a2013 .


4 Determine all positive integers n 2 satisfying i + j ni + nj (mod 2) for all i and j with
0 i j n.
5 Let ABCDEF be a cyclic hexagon satisfying AB BD and BC = EF .Let P be the
intersection of lines BC and AD and let Q be the intersection of lines EF and AD.Assume
that P and Q are on the same side of D and A is on the opposite side.Let S be the midpoint of
AD.Let K and L be the incentres of 4BP S and 4EQS respectively.Prove that KDL = 900 .

This file was downloaded from the AoPS Math Olympiad Resources Page
http://www.artofproblemsolving.com/

113

Page 1

Poland
Finals
2013

Day 1
1 Find all solutions of the following equation in integers x, y : x4 + y = x3 + y 2
2 There are given integers a and b such that a is different from 0 and the number 3 + a + b2 is
divisible by 6a. Prove that a is negative.
3 Given is a quadrilateral ABCD in which we can inscribe circle. The segments AB, BC, CD
and DA are the diameters of the circles o1, o2, o3 and o4, respectively. Prove that there exists
a circle tangent to all of the circles o1, o2, o3 and o4.

This file was downloaded from the AoPS Math Olympiad Resources Page
http://www.artofproblemsolving.com/

114

Page 1

Poland
Finals
2013

Day 2
4 Given is a tetrahedron ABCD in which AB = CD and the sum of measures of the angles
BAD and BCD equals 180 degrees. Prove that the measure of the angle BAD is larger than
the measure of the angle ADC.
5 Let k,m and n be three different positive integers. Prove that




1
1
1
k
m
n
kmn (k + m + n).
k
m
n
6 For each positive integer n determine the maximum number of points in space creating the
set A which has the following properties: 1) the coordinates of every point from the set A
are integers from the range [0, n] 2) for each pair of different points (x1 , x2 , x3 ), (y1 , y2 , y3 )
belonging to the set A it is satisfied at least one of the following inequalities x1 < y1 , x2 <
y2 , x3 < y3 and at least one of the following inequalities x1 > y1 , x2 > y2 , x3 > y3 .

115 This file was downloaded from the AoPS Math Olympiad Resources Page
http://www.artofproblemsolving.com/

Page 2

Romania
District Olympiad
2013

Grade 9

1 a) Prove that, whatever the real number x would be, the following inequality takes place
x4 x3 x + 1 0. b) Solve the following system in the set of real numbers: x1 + x2 + x3 =
3, x31 + x32 + x33 = x41 + x42 + x43 . The Mathematical Gazette
2 Given triangle ABC and the pointsD, E (BC), F, G (CA), H, I (AB) so that BD =
CE, CF = AG and AH = BI. Note with M, N, P the midpoints of [GH], [DI] and [EF ]
0
AG AB
and with M 0 the intersection of the segments AM and BC. a) Prove that BM
CM 0 = AH AC .
b) Prove that the segmentsAM , BN and CP are concurrent.
3 Let n N and a1 , a2 , ..., an R so a1 + a2 + ... + ak k, () k {1, 2, ..., n} .Prove that
a2
a1
an
1
1
1
1 + 2 + ... + n 1 + 2 + ... + n
4 At the top of a piece of paper is written a list of distinctive natural numbers. To continue the
list you must choose 2 numbers from the existent ones and write in the list the least common
multiple of them, on the condition that it isnt written yet. We can say that the list is closed
if there are no other solutions left (for example, the list 2, 3, 4, 6 closes right after we add
12). Which is the maximum numbers which can be written on a list that had closed, if the
list had at the beginning 10 numbers?

This file was downloaded from the AoPS Math Olympiad Resources Page
http://www.artofproblemsolving.com/

116

Page 1

Romania
District Olympiad
2013

Grade 10

1 Let a, b R and z C\R so that |a b| = |a + b 2z|. a) Prove that the equation |z a|x +
|
z b|x = |a b|x , with the unknown number x R, has a unique solution. b) Solve the
following inequation |z a|x + |
z b|x |a b|x , with the unknown number x R. The
Mathematical Gazette
2 Let a, b C. Prove that |az + b
z | 1, for every z C, with |z| = 1, if and only if |a|+|b| 1.
3 Take the function f : R R, f (x) = ax, x Q, f (x) = bx, x R\Q, where a and b are two
real numbers different from 0. Prove that f is injective if and only if f is surjective.



4 Let n N . Prove that 2 2n cos n arccos 42 is an odd integer.

117 This file was downloaded from the AoPS Math Olympiad Resources Page
http://www.artofproblemsolving.com/

Page 2

Romania
District Olympiad
2013

Grade 11

1 Let (an )n1 an increasing sequence and bounded.Calculate lim (2an a1 a2 ) (2an a2 a3 ) ... (2an an
n

2 Let the matrices of order 2 with the real elements A and B so that AB = A2 B 2 (AB)2
and det (B) = 2. a) Prove that the matrix A is not invertible. b) Calculate det (A + 2B)
det (B + 2A).
3 Let A be an non-invertible of order n, n > 1, with the elements in the set of complex numbers,
with all the elements having the module equal with 1
a)Prove that, for n = 3, two rows or two columns of the A matrix are proportional b)Does
the conclusion from the previous exercise remains true for n = 4?
4 Letf : R Rbe a monotone function. a) Prove thatf have side limits in each point x0 R.
b) We define the function g : R R, g (x) = lim f (t)( g (x) with limit at at left in x). Prove
t%x

that if the g function is continuous, than the function f is continuous.

118 This file was downloaded from the AoPS Math Olympiad Resources Page
http://www.artofproblemsolving.com/

Page 3

Romania
District Olympiad
2013

Grade 12

1 Calculate: lim

R1
0

ex dx

2 Problem 2. A group (G, ) has the propriety(P ), if, for any automorphism f for G,there are
two automorphisms g and h in G, so that f (x) = g (x)h (x), whatever x Gwould be. Prove
that: (a) Every group which the property (P ) is comutative. (b) Every commutative finite
group of odd order doesnt have the (P ) property. (c) No finite group of order 4n + 2, n N,
doesnt have the (P )property. (The order of a finite group is the number of elements of that
group).


R 2
.Prove
that:
(a)
f (x) f
3 Problem 3. Let f : 0,2 [0, )R an increasing function
0
Ra
a
0. (b) Exist a 4 , 2 such that 0 f (x) sin x dx = 0 f (x) cos x dx.



(sin x cos x) dx

4 Problem 4. Let(A, +, ) be a ring


property
that x = 0 is the only solution of the
 with the

2
2
x = 0, x Aecuation. Let B = a A|a = 1 . Prove that: (a) ab ba = bab a, whateer
would be a A and b B. (b) (B, ) is a group

119 This file was downloaded from the AoPS Math Olympiad Resources Page
http://www.artofproblemsolving.com/

Page 4

Romania
National Olympiad
2013

IX

1 A series of numbers is called complete if it has non-zero natural terms and any nonzero integer
has at least one among multiple series. Show that the arithmetic progression is a complete
sequence if and only if it divides the first term relationship.
2 Given f : R R an arbitrary function and g : R R a function of the second degree, with
the property: for any real numbers m and n equation f (x) = mx + n has solutions if and
only if the equation g (x) = mx + n has solutions Show that the functions f and g are equal.
3 Given P a point m inside a triangle acute-angled ABC and DEF intersections of lines with
that AP , BP , CP with[BC] , [CA] ,respective [AB] a) Show that the area of the triangle DEF
is at most a quarter of the area of the triangle ABC b) Show that the radius of the circle
inscribed in the triangle DEF is at most a quarter of the radius of the circle circumscribed
on triangle 4ABC.
4 Consider a nonzero integer number n and the function f : N N by
(
x
if x is even
2
f (x) = x1
.
n1
if x is odd
2 +2
Determine the set:
A = {x N | (f f .... f ) (x) = x}.
{z
}
|
n f s

This file was downloaded from the AoPS Math Olympiad Resources Page
http://www.artofproblemsolving.com/

120

Page 1

Romania
National Olympiad
2013

1 Solve the following equation 2sin

4 xcos2 x

4 xsin2 x

2cos

= cos 2x

2 To be considered the following complex and distinct a, b, c, d. Prove that the following affirmations are equivalent: i)For every z C the inequality takes place :|z a| + |z b|
|z c| + |z d|; ii)There is t (0, 1) so that c = ta + (1 t) b si d = (1 t) a + tb
3 Find all injective functionsf : Z Z that satisfy: |f (x) f (y)| |x y| ,for any x, y Z.
4 a)Prove that 12 + 13 + ... + 21m < m, for any m N . b)Let p1 , p2 , ..., pn be the prime numbers
less than 2100 . Prove that p11 + p12 + ... + p1n < 10

121 This file was downloaded from the AoPS Math Olympiad Resources Page
http://www.artofproblemsolving.com/

Page 2

Romania
National Olympiad
2013

XI

1 Given A, non-inverted matrices of order n with real elements, n 2 and given A adjoin
matrix A. Prove that tr(A ) 6= 1 if and only if the matrix In + A is invertible.
2 Whether m and n natural numbers, m, n 2. Consider matrices, A1 , A2 , ..., Am Mn (R)
not all nilpotent. Demonstrate that there is an integer number k > 0 such that Ak 1 + Ak 2 +
..... + Ak m 6= On
3 A function
f:(0,) (0,)
is called contract if, for every numbers x, y (0,) we have, lim (f n (x) f n (y)) = 0 where
n

f n = f f ... f a) Consider
|
{z
}
n f s

f : (0,) (0,)
a function contract, continue with the property that has a fixed point, that existing x0
(0,) there so that f (x0 ) = x0 . Show that f (x) > x, for every x (0,x0 ) and f (x) < x,
for every x (x0 ,) . b) Show that the given function
f :(0,) (0,)
given by f (x) = x +

1
x

is contracted but has no fix number.

4 a) Consider
f : [0,) [0,)
a differentiable and convex function .Show that f (x) x, for every x 0, than f 0 (x) 1
,for every x 0 b) Determine
f : [0,) [0,)
differentiable and convex functions which have the property that f (0) = 0 , and f 0 (x) f (f (x)) =
x, for every x 0

122 This file was downloaded from the AoPS Math Olympiad Resources Page
http://www.artofproblemsolving.com/

Page 3

Romania
National Olympiad
2013

XII

1 Determine continuous functions f : R R such that a2 + ab + b2

 Rb

Rb
f (x) dx = 3 x2 f (x) dx,

for every a, b R .

2 Given a ring (A, +, ) that meets both of the following conditions: (1) A is not a field, and
(2) For every non-invertible element x of A, there is an integer m > 1 (depending on x) such
m
that x = x2 + x3 + . . . + x2 . Show that (a) x + x = 0 for every x A, and (b) x2 = x for
every non-invertible x A.
3 Given a (0, 1) and C the set of increasing functions f : [0, 1] [0, ) such that
. Determine: (a)max

Ra

f C 0

f (x)dx (b)max

Ra

f C 0

R1

f (x)dx = 1

f 2 (x)dx

4 Given n 2 a natural number, (K, +, ) a body with commutative property that |1 +{z...+} 1 6=
m

0, m = 2, ..., n, f K[X] a polynomial of degree n and G a subgroup of the additive group


(K, +, ), G 6= K.Show that there is a K sof (a)
/ G.

123 This file was downloaded from the AoPS Math Olympiad Resources Page
http://www.artofproblemsolving.com/

Page 4

Romania
Stars Of Mathematics
2013

Juniors

1 Prove that for any integers a, b, the equation 2abx4 a2 x2 b2 1 = 0 has no integer roots.
(Dan Schwarz)
2 Three points inside a rectangle determine a triangle. A fourth point is taken inside the
triangle. Prove that at least one of the three concave quadrilaterals formed by these four
points has perimeter lesser than that of the rectangle.
(Dan Schwarz)
n

3 Consider the sequence (32 + 1)n1 .


i) Prove there exist infinitely many primes, none dividing any term of the sequence. ii) Prove
there exist infinitely many primes, each dividing some term of the sequence.
(Dan Schwarz)
4 A set S of unit cells of an n n array, n 2, is said full if each row and each column of the
array contain at least one element of S, but which has this property no more when any of
its elements is removed. A full set having maximum cardinality is said fat, while a full set of
minimum cardinality is said meagre.
i) Determine the cardinality m(n) of the meagre sets, describe all meagre sets and give their
count. ii) Determine the cardinality M (n) of the fat sets, describe all fat sets and give their
count.
(Dan Schwarz)

This file was downloaded from the AoPS Math Olympiad Resources Page
http://www.artofproblemsolving.com/

124

Page 1

Romania
Stars Of Mathematics
2013

Seniors

1 Let F be the family of bijective increasing functions f : [0, 1] [0, 1], and let a (0, 1).
Determine the best constants ma and Ma , such that for all f F we have
ma f (a) + f 1 (a) Ma .
(Dan Schwarz)
2 Three points inside a rectangle determine a triangle. A fourth point is taken inside the
triangle.
i) Prove at least one of the three concave quadrilaterals formed by these four points has
perimeter lesser than that of the rectangle. ii) Assuming the three points inside the rectangle
are three corners of it, prove at least two of the three concave quadrilaterals formed by these
four points have perimeters lesser than that of the rectangle.
(Dan Schwarz)
3 Consider the sequence (an + 1)n1 , with a > 1 a fixed integer.
i) Prove there exist infinitely many primes, each dividing some term of the sequence. ii) Prove
there exist infinitely many primes, none dividing any term of the sequence.
(Dan Schwarz)
4 Given a (fixed) positive integer N , solve the functional equation
f : Z R, f (2k) = 2f (k) and f (N k) = f (k), for all k Z.
(Dan Schwarz)

125 This file was downloaded from the AoPS Math Olympiad Resources Page
http://www.artofproblemsolving.com/

Page 2

Russia
All-Russian Olympiad
2013

Grade 9

Day 1
1 Given three distinct real numbers a, b, and c, show that at least two of the three following
equations
(x a)(x b) = x c
(x c)(x b) = x a
(x c)(x a) = x b
have real solutions.
2 Acute-angled triangle ABC is inscribed into circle . Lines tangent to at B and C intersect
at P . Points D and E are on AB and AC such that P D and P E are perpendicular to AB
and AC respectively. Prove that the orthocentre of triangle ADE is the midpoint of BC.
3 100 distinct natural numbers a1 , a2 , a3 , . . . , a100 are written on the board. Then, under each
number ai , someone wrote a number bi , such that bi is the sum of ai and the greatest common
factor of the other 99 numbers. What is the least possible number of distinct natural numbers
that can be among b1 , b2 , b3 , . . . , b100 ?
4 N lines lie on a plane, no two of which are parallel and no three of which are concurrent.
Prove that there exists a non-self-intersecting broken line A0 A1 A2 A3 ...AN with N parts, such
that on each of the N lines lies exactly one of the N segments of the line.

Day 2
1 2n real numbers with a positive sum are aligned in a circle. For each of the numbers, we can
see there are two sets of n numbers such that this number is on the end. Prove that at least
one of the numbers has a positive sum for both of these two sets.
2 Peter and Basil together thought of ten quadratic trinomials. Then, Basil began calling
consecutive natural numbers starting with some natural number. After each called number,
Peter chose one of the ten polynomials at random and plugged in the called number. The
results were recorded on the board. They eventually form a sequence. After they finished,
their sequence was arithmetic. What is the greatest number of numbers that Basil could have
called out?

This file was downloaded from the AoPS Math Olympiad Resources Page
http://www.artofproblemsolving.com/

126

Page 1

Russia
All-Russian Olympiad
2013

3 Squares CAKL and CBM N are constructed on the sides of acute-angled triangle ABC,
outside of the triangle. Line CN intersects line segment AK at X, while line CL intersects line
segment BM at Y . Point P , lying inside triangle ABC, is an intersection of the circumcircles
of triangles KXN and LY M . Point S is the midpoint of AB. Prove that angle ACS =
BCP .
4 On a 55 55 square grid, 500 unit squares were cut out as well as 400 L-shaped pieces
consisting of 3 unit squares (each piece can be oriented in any way) [refer to the figure].
Prove that at least two of the cut out pieces bordered each other before they were cut out.

127 This file was downloaded from the AoPS Math Olympiad Resources Page
http://www.artofproblemsolving.com/

Page 2

Russia
All-Russian Olympiad
2013

Grade 10

Day 1
1 Given three distinct real numbers a, b, and c, show that at least two of the three following
equations
(x a)(x b) = x c
(x c)(x b) = x a
(x c)(x a) = x b
have real solutions.
2 Circle is divided into n arcs by n marked points on the circle. After that circle rotate an
angle 2k/n (for some positive integer k), marked points moved to n new points , dividing
the circle into n new arcs. Prove that there is a new arc that lies entirely in the one of the
old ars. (It is believed that the endpoints of arcs belong to it.)
I. Mitrophanov
3 Find all positive integers k such that for the first k prime numbers 2, 3, . . . , pk there exist
positive integers a and n > 1, such that 2 3 . . . pk 1 = an .
V. Senderov
4 Inside the inscribed quadrilateral ABCD are marked points P and Q, such that P DC +
P CB, P AB + P BC, QCD + QDA and QBA + QAD are all equal to 90 . Prove
that the line P Q has equal angles with lines AD and BC.
A. Pastor

Day 2
1 Does exist natural n, such that for any non-zero digits a and b
ab | anb ?
(Here by x . . . y denotes the number obtained by concatenation decimal digits x, . . . , y.)
V. Senderov

128 This file was downloaded from the AoPS Math Olympiad Resources Page
http://www.artofproblemsolving.com/

Page 3

Russia
All-Russian Olympiad
2013

2 Peter and Vasil together thought of ten 5-degree polynomials. Then, Vasil began calling
consecutive natural numbers starting with some natural number. After each called number,
Peter chose one of the ten polynomials at random and plugged in the called number. The
results were recorded on the board. They eventually form a sequence. After they finished,
their sequence was arithmetic. What is the greatest number of numbers that Vasil could have
called out?
A. Golovanov
3 The incircle of triangle ABC has centre I and touches the sides BC, CA, AB at points A1 ,
B1 , C1 , respectively. Let Ia , Ib , Ic be excentres of triangle ABC, touching the sides BC, CA,
AB respectively. The segments Ia B1 and Ib A1 intersect at C2 . Similarly, segments Ib C1 and
Ic B1 intersect at A2 , and the segments Ic A1 and Ia C1 at B2 . Prove that I is the center of
the circumcircle of the triangle A2 B2 C2 .
L. Emelyanov, A. Polyansky
4 A square with horizontal and vertical sides is drawn on the plane. It held several segments
parallel to the sides, and there are no two segments which lie on one line or intersect at an
interior point for both segments. It turned out that the segments cuts square into rectangles,
and any vertical line intersecting the square and not containing segments of the partition
intersects exactly k rectangles of the partition, and any horizontal line intersecting the square
and not containing segments of the partition intersects exactly ` rectangles. How much the
number of rectangles can be?
I. Bogdanov, D. Fon-Der-Flaass

129 This file was downloaded from the AoPS Math Olympiad Resources Page
http://www.artofproblemsolving.com/

Page 4

Russia
All-Russian Olympiad
2013

Grade 11
Day 1
1 Let P (x) and Q(x) be (monic) polynomials with real coefficients (the first coefficient being
equal to 1), and deg P (x) = deg Q(x) = 10. Prove that if the equation P (x) = Q(x) has no
real solutions, then P (x + 1) = Q(x 1) has a real solution.
2 The inscribed and exscribed sphere of a triangular pyramid ABCD touch her face BCD at
different points X and Y . Prove that the triangle AXY is obtuse triangle.
3 Find all positive k such that product of the first k odd prime numbers, reduced by 1 is exactly
degree of natural number (which more than one).
4 On each of the cards written in 2013 by number, all of these 2013 numbers are different. The
cards are turned down by numbers. In a single move is allowed to point out the ten cards
and in return will report one of the numbers written on them (do not know what). For what
most w guaranteed to be able to find w cards for which we know what numbers are written
on each of them?

Day 2
1 101 distinct numbers are chosen among the integers between 0 and 1000. Prove that, among
the absolute values of their pairwise differences, there are ten different numbers not exceeding
100.
2 Let a, b, c, d be positive real numbers such that 2(a + b + c + d) abcd. Prove that
a2 + b2 + c2 + d2 abcd.
3 The head of the Mint wants to release 12 coins denominations (each - a natural number
rubles) so that any amount from 1 to 6543 rubles could be paid without having to pass, using
no more than 8 coins. Can he do it? (If the payment amount you can use a few coins of the
same denomination.)
4 Let be the incircle of the triangle ABC and with centre I. Let be the circumcircle of the
triangle AIB. Circles and intersect at the point X and Y . Let Z be the intersection of
the common tangents of the circles and . Show that the circumcircle of the triangle XY Z
is tangent to the circumcircle of the triangle ABC.

130 This file was downloaded from the AoPS Math Olympiad Resources Page
http://www.artofproblemsolving.com/

Page 5

Russia
Sharygin Geometry Olympiad
2013

First Round

1 Let ABC be an isosceles triangle with AB = BC. Point E lies on the side AB, and ED is
the perpendicular from E to BC. It is known that AE = DE. Find DAC.
2 Let ABC be an isosceles triangle (AC = BC) with C = 20 . The bisectors of angles A and
B meet the opposite sides at points A1 and B1 respectively. Prove that the triangle A1 OB1
(where O is the circumcenter of ABC) is regular.
3 Let ABC be a right-angled triangle (B = 90 ). The excircle inscribed into the angle A
touches the extensions of the sides AB, AC at points A1 , A2 respectively; points C1 , C2 are
defined similarly. Prove that the perpendiculars from A, B, C to C1 C2 , A1 C1 , A1 A2 respectively, concur.
4 Let ABC be a non isosceles triangle. Point O is its circumcenter, and the point K is the
center of the circumcircle of triangle BCO. The altitude of ABC from A meets at a
point P . The line P K intersects the circumcircle of ABC at points E and F . Prove that one
of the segments EP and F P is equal to the segment P A.
5 Four segments drawn from a given point inside a convex quadrilateral to its vertices, split the
quadrilateral into four equal triangles. Can we assert that this quadrilateral is a rhombus?
6 Diagonals AC and BD of a trapezoid ABCD meet at P . The circumcircles of triangles ABP
and CDP intersect the line AD for the second time at points X and Y respectively. Let M
be the midpoint of segment XY . Prove that BM = CM .
7 Let BD be a bisector of triangle ABC. Points Ia , Ic are the incenters of triangles ABD,
CBD respectively. The line Ia Ic meets AC in point Q. Prove that DBQ = 90 .
8 Let X be an arbitrary point inside the circumcircle of a triangle ABC. The lines BX and
CX meet the circumcircle in points K and L respectively. The line LK intersects BA and
AC at points E and F respectively. Find the locus of points X such that the circumcircles
of triangles AF K and AEL touch.
9 Let T1 and T2 be the points of tangency of the excircles of a triangle ABC with its sides
BC and AC respectively. It is known that the reflection of the incenter of ABC across the
midpoint of AB lies on the circumcircle of triangle CT1 T2 . Find BCA.
10 The incircle of triangle ABC touches the side AB at point C 0 ; the incircle of triangle ACC 0
touches the sides AB and AC at points C1 , B1 ; the incircle of triangle BCC 0 touches the sides
AB and BC at points C2 , A2 . Prove that the lines B1 C1 , A2 C2 , and CC 0 concur.

This file was downloaded from the AoPS Math Olympiad Resources Page
http://www.artofproblemsolving.com/

131

Page 1

Russia
Sharygin Geometry Olympiad
2013

11 a) Let ABCD be a convex quadrilateral and r1 r2 r3 r4 be the radii of the incircles of


triangles ABC, BCD, CDA, DAB. Can the inequality r4 > 2r3 hold?
b) The diagonals of a convex quadrilateral ABCD meet in point E. Let r1 r2 r3 r4 be
the radii of the incircles of triangles ABE, BCE, CDE, DAE. Can the inequality r2 > 2r1
hold?
12 On each side of triangle ABC, two distinct points are marked. It is known that these points
are the feet of the altitudes and of the bisectors.
a) Using only a ruler determine which points are the feet of the altitudes and which points
are the feet of the bisectors.
b) Solve p.a) drawing only three lines.
13 Let A1 and C1 be the tangency points of the incircle of triangle ABC with BC and AB
respectively, A0 and C 0 be the tangency points of the excircle inscribed into the angle B with
the extensions of BC and AB respectively. Prove that the orthocenter H of triangle ABC
lies on A1 C1 if and only if the lines A0 C1 and BA are orthogonal.
14 Let M , N be the midpoints of diagonals AC, BD of a right-angled trapezoid ABCD (]A =
]D = 90 ). The circumcircles of triangles ABN , CDM meet the line BC in the points Q,
R. Prove that the distances from Q, R to the midpoint of M N are equal.
15 (a) Triangles A1 B1 C1 and A2 B2 C2 are inscribed into triangle ABC so that C1 A1 BC,
A1 B1 CA, B1 C1 AB, B2 A2 BC, C2 B2 CA, A2 C2 AB. Prove that these triangles
are equal.
(b) Points A1 , B1 , C1 , A2 , B2 , C2 lie inside a triangle ABC so that A1 is on segment AB1 ,
B1 is on segment BC1 , C1 is on segment CA1 , A2 is on segment AC2 , B2 is on segment BA2 ,
C2 is on segment CB2 , and the angles BAA1 , CBB2 , ACC1 , CAA2 , ABB2 , BCC2 are equal.
Prove that the triangles A1 B1 C1 and A2 B2 C2 are equal.
16 The incircle of triangle ABC touches BC, CA, AB at points A1 , B1 , C1 , respectively. The
perpendicular from the incenter I to the median from vertex C meets the line A1 B1 in point
K. Prove that CK is parallel to AB.
17 An acute angle between the diagonals of a cyclic quadrilateral is equal to . Prove that an
acute angle between the diagonals of any other quadrilateral having the same sidelengths is
smaller than .
18 Let AD be a bisector of triangle ABC. Points M and N are projections of B and C respectively to AD. The circle with diameter M N intersects BC at points X and Y . Prove that
BAX = CAY .

132 This file was downloaded from the AoPS Math Olympiad Resources Page
http://www.artofproblemsolving.com/

Page 2

Russia
Sharygin Geometry Olympiad
2013

19 a) The incircle of a triangle ABC touches AC and AB at points B0 and C0 respectively. The
bisectors of angles B and C meet the perpendicular bisector to the bisector AL in points Q
and P respectively. Prove that the lines P C0 , QB0 and BC concur.
b) Let AL be the bisector of a triangle ABC. Points O1 and O2 are the circumcenters of
triangles ABL and ACL respectively. Points B1 and C1 are the projections of C and B to the
bisectors of angles B and C respectively. Prove that the lines O1 C1 , O2 B1 , and BC concur.
c) Prove that the two points obtained in pp. a) and b) coincide.
20 Let C1 be an arbitrary point on the side AB of triangle ABC. Points A1 and B1 on the rays
BC and AC are such that AC1 B1 = BC1 A1 = ACB. The lines AA1 and BB1 meet in
point C2 . Prove that all the lines C1 C2 have a common point.
21 Chords BC and DE of circle meet at point A. The line through D parallel to BC meets
again at F , and F A meets again at T . Let M = ET BC and let N be the reflection
of A over M . Show that (DEN ) passes through the midpoint of BC.
22 The common perpendiculars to the opposite sidelines of a nonplanar quadrilateral are mutually orthogonal. Prove that they intersect.
23 Two convex polytopes A and B do not intersect. The polytope A has exactly 2012 planes of
symmetry. What is the maximal number of symmetry planes of the union of A and B, if B
has a) 2012, b) 2013 symmetry planes?
c) What is the answer to the question of p.b), if the symmetry planes are replaced by the
symmetry axes?

133 This file was downloaded from the AoPS Math Olympiad Resources Page
http://www.artofproblemsolving.com/

Page 3

Russia
Sharygin Geometry Olympiad
2013

Grade 8

134 This file was downloaded from the AoPS Math Olympiad Resources Page
http://www.artofproblemsolving.com/

Page 4

Russia
Sharygin Geometry Olympiad
2013

Grade 9

135 This file was downloaded from the AoPS Math Olympiad Resources Page
http://www.artofproblemsolving.com/

Page 5

Russia
Sharygin Geometry Olympiad
2013

Grade 10

1 A circle k passes through the vertices B, C of a scalene triangle ABC. k meets the extensions
of AB, AC beyond B, C at P, Q respectively. Let A1 is the foot the altitude drop from A to
\
BC. Suppose A1 P = A1 Q. Prove that P\
A1 Q = 2BAC.
2 Let ABCD is a tangential quadrilateral such that AB = CD > BC. AC meets BD at L.
[ is acute.
Prove that ALB
According to the jury, they want to propose a more generalized problem is to prove (AB
CD)2 < (AD BC)2 , but this problem has appeared some time ago
3 Let X be a point inside triangle ABC such that XA.BC = XB.AC = XC.AC. Let I1 , I2 , I3
be the incenters of XBC, XCA, XAB. Prove that AI1 , BI2 , CI3 are concurrent.
Of course, the most natural way to solve this is the Ceva sin theorem, but there is an another approach that may surprise you;), try not to use the Ceva theorem ! s:) img
src=SMILIESP AT H/smile.gif alt = :)title = Smile/ ><! s :) >)
4 Given a square cardboard of area 14 , and a paper triangle of area 12 such that the square of
its sidelength is a positive integer. Prove that the triangle can be folded in some ways such
that the squace can be placed inside the folded figure so that both of its faces are completely
covered with paper.
Proposed by N.Beluhov, Bulgaria
It is interesting that number theory knowledge can be used to solve a geometry problem !
s;) img src=SMILIESP AT H/wink.gif alt = ; )title = W ink/ ><! s; ) >
5 Let ABCD is a cyclic quadrilateral inscribed in (O). E, F are the midpoints of arcs AB and
CD not containing the other vertices of the quadrilateral. The line passing through E, F and
parallel to the diagonals of ABCD meet at E, F, K, L. Prove that KL passes through O.
6 The altitudes AA1 , BB1 , CC1 of an acute triangle ABC concur at H. The perpendicular lines
from H to B1 C1 , A1 C1 meet rays CA, CB at P, Q respectively. Prove that the line from C
perpendicular to A1 B1 passes through the midpoint of P Q.
7 Given five fixed points in the space. It is known that these points are centers of five spheres,
four of which are pairwise externally tangent, and all these point are internally tangent to
the fifth one. It turns out that it is impossible to determine which of the marked points is
the center of the largest sphere. Find the ratio of the greatest and the smallest radii of the
spheres.

136 This file was downloaded from the AoPS Math Olympiad Resources Page
http://www.artofproblemsolving.com/

Page 6

Russia
Sharygin Geometry Olympiad
2013

8 Two fixed circles are given on the plane, one of them lies inside the other one. From a point
C moving arbitrarily on the external circle, draw two chords CA, CB of the larger circle such
that they tangent to the smalaler one. Find the locus of the incenter of triangle ABC.

137 This file was downloaded from the AoPS Math Olympiad Resources Page
http://www.artofproblemsolving.com/

Page 7

Serbia
National Math Olympiad
2013

1 Let k be a natural number. Bijection f : Z Z has the following property: for any integers
i and j, |i j| k implies |f (i) f (j)| k. Prove that for every i, j Z it stands:
|f (i) f (j)| = |i j|.
2 For a natural number n, set Sn is defined as:
     
 2 
n
2n
3n
n
Sn =
,
,
, ...,
.
n
n
n
n
a) Prove that there are infinitely many composite numbers n, such that the set Sn is not
complete residue system mod n;
b) Prove that there are infinitely many composite numbers n, such that the set Sn is complete
residue system mod n.
3 Let M , N and P be midpoints of sides BC, AC and AB, respectively, and let O be circumcenter of acute-angled triangle ABC. Circumcircles of triangles BOC and M N P intersect at
two different points X and Y inside of triangle ABC. Prove that
BAX = CAY.
4 Determine all natural numbers n for which there is a partition of {1, 2, ..., 3n} in n pairwise
disjoint subsets of the form {a, b, c}, such that numbers b a and c b are different numbers
from the set {n 1, n, n + 1}.
5 Let A0 and B 0 be feet of altitudes from A and B, respectively, in acute-angled triangle ABC
(AC 6= BC). Circle k contains points A0 and B 0 and touches segment AB in D. If triangles
ADA0 and BDB 0 have the same area, prove that
A0 DB 0 = ACB.
6 Find the largest constant K R with the following property: if a1 , a2 , a3 , a4 > 0 are numbers
satisfying a2i + a2j + a2k 2(ai aj + aj ak + ak ai ), for every 1 i < j < k 4, then
a21 + a22 + a23 + a24 K(a1 a2 + a1 a3 + a1 a4 + a2 a3 + a2 a4 + a3 a4 ).

This file was downloaded from the AoPS Math Olympiad Resources Page
http://www.artofproblemsolving.com/

138

Page 1

South africa
National Olympiad
2013

1 2013 is the first year since the Middle Ages that consists of four consecutive digits. How many
such years are there still to come after 2013 (and before 10000)?
2 A is a two-digit number and B is a three-digit number such that A increased by B
3 Let ABC be an acute-angled triangle and AD one of its altitudes (D on BC). The line through
D parallel to AB is denoted by l, and t is the tangent to the circumcircle of ABC at A. Finally,
let E be the intersection of l and t. Show that CE and t are perpendicular to each other.
4 Determine all pairs of polynomials f and g with real coefficients such that
x2 g(x) = f (g(x)).
5 Some coins are placed on a 20 13 board. Two coins are called neighbours if they are in
the same row or column and no other coins lie between them. What is the largest number of
coins that can be placed on the board if no coin is allowed to have more than two neighbours?
6 Let ABC be an acute-angled triangle with AC 6= BC, and let O be the circumcentre and F
the foot of the altitude through C. Furthermore, let X and Y be the feet of the perpendiculars
dropped from A and B respectively to (the extension of) CO. The line F O intersects the
circumcircle of F XY a second time at P . Prove that OP < OF .

This file was downloaded from the AoPS Math Olympiad Resources Page
http://www.artofproblemsolving.com/

139

Page 1

Turkey
JBMO TST - Turkey
2013

Day 1 - 25 May 2013


1 Let D be a point on the side BC of an equilateral triangle ABC where D is different than
the vertices. Let I be the excenter of the triangle ABD opposite to the side AB and J be
the excenter of the triangle ACD opposite to the side AC. Let E be the second intersection
point of the circumcircles of triangles AIB and AJC. Prove that A is the incenter of the
triangle IEJ.
2 a) Find all prime numbers p, q, r satisfying 3 - p + q + r and p + q + r and pq + qr + rp + 3
are both perfect squares.
b) Do there exist prime numbers p, q, r such that 3 | p + q + r and p + q + r and pq + qr + rp + 3
are both perfect squares?
3 Two players A and B play a game with a ball and n boxes placed onto the vertices of a
regular n-gon where n is a positive integer. Initially, the ball is hidden in a box by player A.
At each step, B chooses a box, then player A says the distance of the ball to the selected box
to player B and moves the ball to an adjacent box. If B finds the ball, then B wins. Find
the least number of steps for which B can guarantee to win.
4 For all positive real numbers a, b, c satisfying a + b + c = 1, prove that
b4 + 5c4
c4 + 5a4
a4 + 5b4
+
+
1 ab bc ca
a(a + 2b) b(b + 2c) c(c + 2a)

This file was downloaded from the AoPS Math Olympiad Resources Page
http://www.artofproblemsolving.com/

140

Page 1

Turkey
JBMO TST - Turkey
2013

Day 2 - 26 May 2013


5 Let a, b, c, d be real numbers greater than 1 and x, y be real numbers such that
ax + by = (a2 + b2 )x

and cx + dy = 2y (cd)y/2

Prove that x < y.


6 Find all positive integers n satisfying 2n + 7 | n! 1.

7 In a convex quadrilateral ABCD diagonals intersect at E and BE = 2 ED, BEC = 45 .


Let F be the foot of the perpendicular from A to BC and P be the second intersection point
of the circumcircle of triangle BF D and line segment DC. Find AP D.
8 In a directed graph with 2013 vertices, there is exactly one edge between any two vertices
and for every vertex there exists an edge outwards this vertex. We know that whatever the
arrangement of the edges, from every vertex we can reach k vertices using at most two edges.
Find the maximum value of k.

141 This file was downloaded from the AoPS Math Olympiad Resources Page
http://www.artofproblemsolving.com/

Page 2

Turkey
Junior National Olympiad
2013

1 Let x, y, z be real numbers satisfying x + y + z = 0 and x2 + y 2 + z 2 = 6. Find the maximum


value of
|(x y)(y z)(z x)|
2 Find all prime numbers p, q, r satisfying the equation
p4 + 2p + q 4 + q 2 = r2 + 4q 3 + 1
3 Let ABC be a triangle such that AC > AB. A circle tangent to the sides AB and AC at D
and E respectively, intersects the circumcircle of ABC at K and L. Let X and Y be points
on the sides AB and AC respectively, satisfying
AX
CE
=
AB
BD + CE

and

AY
BD
=
AC
BD + CE

Show that the lines XY, BC and KL are concurrent.


4 Player A places an odd number of boxes around a circle and distributes 2013 balls into some
of these boxes. Then the player B chooses one of these boxes and takes the balls in it. After
that the player A chooses half of the remaining boxes such that none of two are consecutive
and take the balls in them. If player A guarantees to take k balls, find the maximum possible
value of k.

This file was downloaded from the AoPS Math Olympiad Resources Page
http://www.artofproblemsolving.com/

142

Page 1

Turkey
National Olympiad Second Round
2013

Day 1 - 23 November 2013


1 The circle 1 with diameter [AB] and the circle 2 with center A intersects at points C and
D. Let E be a point on the circle 2 , which is outside 1 and at the same side as C with
respect to the line AB. Let the second point of intersection of the line BE with 2 be F . For
a point K on the circle 1 which is on the same side as A with respect to the diameter of 1
passing through C we have 2 CK AC = CE AB. Let the second point of intersection of
the line KF with 1 be L. Show that the symmetric of the point D with respect to the line
BE is on the circumcircle of the triangle LF C.
2 Let m be a positive integer. a. Show that there exist infinitely many positive integers k such
that 1+km3 is a perfect cube and 1 +kn3 is not a perfect cube for all positive integers n < m.
b. Let m = pr where p 2 (mod 3) is a prime number and r is a positive integer. Find all
numbers k satisfying the condition in part a.
3 Let G be a simple, undirected, connected graph with 100 vertices and 2013 edges. It is given
that there exist two vertices A and B such that it is not possible to reach A from B using
one or two edges. We color all edges using n colors, such that for all pairs of vertices, there
exists a way connecting them with a single color. Find the maximum value of n.

This file was downloaded from the AoPS Math Olympiad Resources Page
http://www.artofproblemsolving.com/

143

Page 1

Turkey
National Olympiad Second Round
2013

Day 2 - 24 November 2013


1 Find all positive integers m and n satisfying 2n + n = m!.
2 Find the maximum value of M for which for all positive real numbers a, b, c we have
a3 + b3 + c3 3abc M (ab2 + bc2 + ca2 3abc)
3 Let n be a positive integer and P1 , P2 , . . . , Pn be different points on the plane such that distances between them are all integers. Furthermore, we know that the distances Pi P1 , Pi P2 , . . . , Pi Pn
forms the same sequence for all i = 1, 2, . . . , n when these numbers are arranged in a nondecreasing order. Find all possible values of n.

144 This file was downloaded from the AoPS Math Olympiad Resources Page
http://www.artofproblemsolving.com/

Page 2

Turkey
Team Selection Test
2013

Day 1 - 30 March 2013


1 Let (n) be the number of positive integers less than n that are relatively prime to n, where
n is a positive integer. Find all pairs of positive integers (m, n) such that
2n + (n (n) 1)! = nm + 1.
2 We put pebbles on some unit squares of a 2013 2013 chessboard such that every unit square
contains at most one pebble. Determine the minimum number of pebbles on the chessboard,
if each 19 19 square formed by unit squares contains at least 21 pebbles.
b 6= m(C).
b
3 Let O be the circumcenter and I be the incenter of an acute triangle ABC with m(B)
Let D, E, F be the midpoints of the sides [BC], [CA], [AB], respectively. Let T be the foot
of perpendicular from I to [AB]. Let P be the circumcenter of the triangle DEF and Q be
the midpoint of [OI]. If A, P , Q are collinear, prove that
|AO| |BC|

= 4.
|OD|
|AT |

This file was downloaded from the AoPS Math Olympiad Resources Page
http://www.artofproblemsolving.com/

145

Page 1

Turkey
Team Selection Test
2013

Day 2 - 31 March 2013


1 Find all pairs of integers (m, n) such that m6 = nn+1 + n 1.
2 Let the incircle of the triangle ABC touch [BC] at D and I be the incenter of the triangle.
Let T be midpoint of [ID]. Let the perpendicular from I to AD meet AB and AC at K
and L, respectively. Let the perpendicular from T to AD meet AB and AC at M and N ,
respectively. Show that |KM | |LN | = |BM | |CN |.
3 For all real numbers x, y, z such that 2 x, y, z 2 and x2 + y 2 + z 2 + xyz = 4, determine
the least real number K satisfying
z(xz + yz + y)
K.
xy + y 2 + z 2 + 1

146 This file was downloaded from the AoPS Math Olympiad Resources Page
http://www.artofproblemsolving.com/

Page 2

Turkey
Team Selection Test
2013

Day 3 - 01 April 2013


1 Let E be intersection of the diagonals of convex quadrilateral ABCD. It is given that
\ = m(DEC)
\ = m(BAD).
\ If F is a point on [BC] such that m(BAF
\) + m(EBF
\) =
m(EDC)
\
m(BF
E), show that A, B, F , D are concyclic.
2 Determine all functions f : R R+ such that for all real numbers x, y the following conditions hold:
i. f (x2 ) = f (x)2 2xf (x)
ii. f (x) = f (x 1)
iii. 1 < x < y = f (x) < f (y).
3 Some cities of a country consisting of n cities are connected by round trip flights so that there
are at least k flights from any city and any city is reachable from any city. Prove that for any
such flight organization these flights can be distributed among n k air companies so that
one can reach any city from any city by using of at most one flight of each air company.

147 This file was downloaded from the AoPS Math Olympiad Resources Page
http://www.artofproblemsolving.com/

Page 3

Undergraduate Competitions
IMC
2013

Day 1 - 28 July 2013


1 Let A and B be real symmetric matrixes with all eigenvalues strictly greater than 1. Let
be a real eigenvalue of matrix AB. Prove that || > 1.
Proposed by Pavel Kozhevnikov, MIPT, Moscow.
2 Let f : R R be a twice differentiable function. Suppose f (0) = 0. Prove there exists

,
such that
2 2

f 00 () = f () 1 + 2tan2 .
Proposed by Karen Keryan, Yerevan State University, Yerevan, Armenia.
3 There are 2n students in a school (n N, n > 2). Each week n students go on a trip. After
several trips the following condition was fulfiled: every two students were together on at least
one trip. What is the minimum number of trips needed for this to happen?
Proposed by Oleksandr Rybak, Kiev, Ukraine.
4 Let n > 3 and let x1 , x2 , ..., xn be nonnegative real numbers. Define A =

n
X
i=1

xi , B =

n
X

x2i , C =

i=1

Prove that:
(n + 1) A2 B + (n 2) B 2 > A4 + (2n 2) AC.
Proposed by Gza Ks, Etvs University, Budapest.
5 Does there exist a sequence (an ) of complex numbers such that for every positive integer p
+
X
we have that
apn converges if and only if p is not a prime?
n=1

Proposed by Tom Brta, Charles University, Prague.

This file was downloaded from the AoPS Math Olympiad Resources Page
http://www.artofproblemsolving.com/

148

Page 1

n
X
i=1

x3i .

Undergraduate Competitions
IMC
2013

Day 2 - 09 August 2013




1 Let z be a complex number with |z + 1| > 2. Prove that z 3 + 1 > 1.
Proposed by Walther Janous and Gerhard Kirchner, Innsbruck.
2 Let p, q be relatively prime positive integers. Prove that
pq1
X

j k j k
k
+ kq
p

(1)

k=0

(
0
=
1

if pq is even
if pq odd

Proposed by Alexander Bolbot, State University, Novosibirsk.


3 Suppose that v1 , v2 , ..., vd are unit vectors in Rd . Prove that there exists a unitary vector u
1
such that |u vi | for i = 1, 2, ..., d.
d
Note. Here denotes the usual scalar product on Rd .
Proposed by Tomasz Tkocz, University of Warwick.
4 Does there exist an infinite set M consisting of positive integers such that for any a, b M
with a < b the sum a + b is square-free? Note. A positive integer is called square-free if no
perfect square greater than 1 divides it.
Proposed by Fedor Petrov, St. Petersburg State University.
5 Consider a circular necklace with 2013 beads. Each bead can be paintes either green or white.
A painting of the necklace is called good if among any 21 successive beads there is at least
one green bead. Prove that the number of good paintings of the necklace is odd.
Note. Two paintings that differ on some beads, but can be obtained from each other by
rotating or flipping the necklace, are counted as different paintings.
Proposed by Vsevolod Bykov and Oleksandr Rybak, Kiev.

149 This file was downloaded from the AoPS Math Olympiad Resources Page
http://www.artofproblemsolving.com/

Page 2

Undergraduate Competitions
Mikls Schweitzer
2013

1 Let q be a positive integer. Prove there exists a constant Cq such that the following inequality
holds for any finite set A of integers:
|A + qA| (q + 1)|A| Cq .
Proposed by Antal Balog.
2 Prove there exists a constant k0 such that for any k k0 , the equation
a2n + b4n + 2013 = kan b2n
has no positive integer solutions a, b, n.
Proposed by Istvn Pink.
3 Find for which positive integers n the An alternating group has a permutation which is
contained in exactly one 2-Sylow subgroup of An .
Proposed by Pter Pl Plfy
4 Let A be an Abelian group with n elements. Prove that there are two subgroups in GL(n, C),
isomorphic to Sn , whose intersection is isomorphic to the automorphism group of A.
Proposed by Zoltn Halasi
5 A subalgebra h of a Lie algebra g is said to have the property with respect to a scalar product
h, i given on g if X h implies h[X, Y ], Xi = 0 for all Y g. Prove that the maximum
dimension of -property subalgebras of a given 2 step nilpotent Lie algebra with respect to a
scalar product is independent of the selection of the scalar product.
Proposed by Pter Nagy Tibor
6 Let A be a C algebra with a unit element and let A+ be the cone of the positive elements
of A (this is the set of such self adjoint elements in A whose spectrum is in [0, ). Consider
the operation

x y = xy x, x, y A+
Prove that if for all x, y A+ we have
(x y) y = x (y y),
then A is commutative.
Proposed by Lajos Molnr

This file was downloaded from the AoPS Math Olympiad Resources Page
http://www.artofproblemsolving.com/

150

Page 1

Undergraduate Competitions
Mikls Schweitzer
2013

7 Suppose that f : R
R is an additive function (that is f (x + y) = f (x) + f (y) for all x, y R)
for which x 7 f (x)f ( 1 x2 ) is bounded of some nonempty subinterval of (0, 1). Prove that
f is continuous.
Proposed by Zoltn Boros
8 Let f : R R be a continuous and strictly increasing function for which




x+y
1 f (x) + f (y)
f
(f (x) + f (y)) = (x + y)f
2
2
for all x, y R(f 1 denotes the inverse of f ). Prove that there exist real constants a 6= 0 and
b such that f (x) = ax + b for all x R.
Proposed by Zoltn Darczy
9 Prove that there is a function f : (0, ) (0, ) which is nowhere continuous and for all
x, y (0, ) and any rational we have

1! 
1
x + y
f (x) + f (y)
f

.
2
2
Is there such a function if instead the above relation holds for every x, y (0, ) and for
every irrational ?
Proposed by Maksa Gyula and Zsolt Ples
10 Consider a Riemannian metric on the vector space Rn which satisfies the property that for
each two points a, b there is a single distance minimising geodesic segment g(a, b). Suppose
that for all a Rn , the Riemannian distance with respect to a, a : Rn R is convex and
differentiable outside of a. Prove that if for a point x 6= a, b we have
i a (x) = i b (x), i = 1, , n
then x is a point on g(a, b) and conversely.
Proposed by Lajos Tamssy and Dvid Kertsz
11 (a) Consider an ellipse in the plane. Prove that there exists a Riemannian metric which is
defined on the whole plane, and with respect to which the ellipse is a geodesic. Prove that the
Gaussian curvature of any such Riemannian metric takes a positive value. (b) Consider two
nonintersecting, simple closed smooth curves in the plane. Prove that if there is a Riemmanian
metric defined on the whole plane and the two curves are geodesics of that metric, then the
Gaussian curvature of the metric vanishes somewhere.
Proposed by Tran Quoc Binh

151 This file was downloaded from the AoPS Math Olympiad Resources Page
http://www.artofproblemsolving.com/

Page 2

Undergraduate Competitions
Mikls Schweitzer
2013

12 There are n tokens in a pack. Some of them (at least one, but not all) are white and the
rest are black. All tokens are extracted randomly from the pack, one by one, without putting
them back. Let Xi be the ratio of white tokens in the pack before the ith extraction and let
T = max{|Xi Xj | : 1 i j n}.
Prove that E(T ) H(E(X1 )), where H(x) = x ln x (1 x) ln(1 x).
Proposed by Tams Mri

152 This file was downloaded from the AoPS Math Olympiad Resources Page
http://www.artofproblemsolving.com/

Page 3

Undergraduate Competitions
Nanyang Technological University/Euler Math Contest
2013

1 Let f : [1, ) [1, ) be such that for x 1, f (x) = y where y 1 is the unique solution
to the equation y y = x. Then compute
Z e
f (ex ) dx
0

Proposed by Jane Street


2 Let A1 , A2 , . . . , A4` R3 be in generic position (no four points are coplanar). Prove that one
can partition A1 , A2 , . . . , A4` R3 into vertices of mutually non-intersecting tetrahedra.
Proposed by Fedor Duzhin
3 What is the last digit of



5+ 21
2

2013 

?
Proposed by Fedor Duzhin

4 Find the number of permutations ((1), (2), . . . , (20)) of the integers (1, 2, . . . , 20) satisfying
1(1) 2(2) . . . 20(20).
Proposed by Jane Street
5 Find all continuous functions f : R R satisfying


2x
f (x) 2013f
= 89 for all x 6= 1, 1.
1 x2
Proposed by Chan Swee Hong

This file was downloaded from the AoPS Math Olympiad Resources Page
http://www.artofproblemsolving.com/

153

Page 1

Undergraduate Competitions
Putnam
2013

1 Recall that a regular icosahedron is a convex polyhedron having 12 vertices and 20 faces; the
faces are congruent equilateral triangles. On each face of a regular icosahedron is written a
nonnegative integer such that the sum of all 20 integers is 39. Show that there are two faces
that share a vertex and have the same integer written on them.
2 Let S be the set of all positive integers that are not perfect squares. For n in S, consider
choices of integers a1 , a2 , . . . , ar such that n < a1 < a2 < < ar and n a1 a2 ar is a
perfect square, and let f (n) be the minimum of ar over all such choices. For example, 2 3 6
is a perfect square, while 2 3, 2 4, 2 5, 2 3 4, 2 3 5, 2 4 5, and 2 3 4 5 are not, and so
f (2) = 6. Show that the function f from S to the integers is one-to-one.
3 Suppose that the real numbers a0 , a1 , . . . , an and x, with 0 < x < 1, satisfy
a0
a1
an
+
+ +
= 0.
2
1x 1x
1 xn+1
Prove that there exists a real number y with 0 < y < 1 such that
a0 + a1 y + + an y n = 0.
4 A finite collection of digits 0 and 1 is written around a circle. An arc of length L 0 consists
of L consecutive digits around the circle. For each arc w, let Z(w) and N (w) denote the
number of 0s in w and the number of 1s in w, respectively. Assume that |Z(w) Z(w0 )| 1
for any two arcs w, w0 of the same length. Suppose that some arcs w1 , . . . , wk have the
property that
k
k
1X
1X
Z=
Z(wj ) and N =
N (wj )
k
k
j=1

j=1

are both integers. Prove that there exists an arc w with Z(w) = Z and N (w) = N.

5 For m 3, a list of m
3 real numbers aijk (1 i < j < k m) is said to be area definite for
Rn if the inequality
X
aijk Area(4Ai Aj Ak ) 0
1i<j<km

holds for every choice of m points A1 , . . . , Am in Rn . For example, the list of four numbers
a123 = a124 = a134 = 1, a234 = 1 is area definite for R2 . Prove that if a list of m
3 numbers
is area definite for R2 , then it is area definite for R3 .

This file was downloaded from the AoPS Math Olympiad Resources Page
http://www.artofproblemsolving.com/

154

Page 1

Undergraduate Competitions
Putnam
2013

6 Define a function w : Z Z Z as follows. For |a|, |b| 2, let w(a, b) be as in the table
shown; otherwise, let w(a, b) = 0.

w(a, b)
2
1
a
0
1
2

2
1
2
2
2
1

1
2
4
4
4
2

b
0
1
2
2 2 1
4
4 2
12 4
2
4
4 2
2 2 1

For every finite subset S of Z Z, define


A(S) =

w(s s0 ).

(s,s0 )SS

Prove that if S is any finite nonempty subset of Z Z, then A(S) > 0. (For example, if S =
{(0, 1), (0, 2), (2, 0), (3, 1)}, then the terms in A(S) are 12, 12, 12, 12, 4, 4, 0, 0, 0, 0, 1, 1, 2, 2, 4, 4.)

155 This file was downloaded from the AoPS Math Olympiad Resources Page
http://www.artofproblemsolving.com/

Page 2

Undergraduate Competitions
Putnam
2013

1 For positive integers n, let the numbers c(n) be determined by the rules c(1) = 1, c(2n) = c(n),
and c(2n + 1) = (1)n c(n). Find the value of
2013
X

c(n)c(n + 2).

n=1

2 Let C =

N =1 CN ,

where CN denotes the set of cosine polynomials of the form


f (x) = 1 +

N
X

an cos(2nx)

n=1

for which:
(i) f (x) 0 for all real x, and (ii) an = 0 whenever n is a multiple of 3.
Determine the maximum value of f (0) as f ranges through C, and prove that this maximum
is attained.
3 Let P be a nonempty collection of subsets of {1, . . . , n} such that:
(i) if S, S 0 P, then S S 0 P and S S 0 P, and (ii) if S P and S 6= , then there is a
subset T S such that T P and T contains exactly one fewer element than S.
Suppose that f : P R is a function such that f () = 0 and
f (S S 0 ) = f (S) + f (S 0 ) f (S S 0 ) for all S, S 0 P.
Must there exist real numbers f1 , . . . , fn such that
X
f (S) =
fi
iS

for every S P ?
4 For any continuous real-valued function f defined on the interval [0, 1], let
Z 1
Z 1
(f ) =
f (x) dx, Var(f ) =
(f (x) (f ))2 dx, M (f ) = max |f (x)|.
0

0x1

Show that if f and g are continuous real-valued functions defined on the interval [0, 1], then
Var(f g) 2Var(f )M (g)2 + 2Var(g)M (f )2 .

156 This file was downloaded from the AoPS Math Olympiad Resources Page
http://www.artofproblemsolving.com/

Page 3

Undergraduate Competitions
Putnam
2013

5 Let X = {1, 2, . . . , n}, and let k X. Show that there are exactly knn1 functions f : X X
such that for every x X there is a j 0 such that f (j) (x) k.

[Here f (j) denotes the jth iterate of f, so that f (0) (x) = x and f (j+1) (x) = f f (j) (x) .]
6 Let n 1 be an odd integer. Alice and Bob play the following game, taking alternating turns,
with Alice playing first. The playing area consists of n spaces, arranged in a line. Initially all
spaces are empty. At each turn, a player either
places a stone in an empty space, or removes a stone from a nonempty space s, places a
stone in the nearest empty space to the left of s (if such a space exists), and places a stone
in the nearest empty space to the right of s (if such a space exists).
Furthermore, a move is permitted only if the resulting position has not occurred previously
in the game. A player loses if he or she is unable to move. Assuming that both players play
optimally throughout the game, what moves may Alice make on her first turn?

157 This file was downloaded from the AoPS Math Olympiad Resources Page
http://www.artofproblemsolving.com/

Page 4

USA
TSTST
2013

Day 1 - 21 June 2013


1 Let ABC be a triangle and D, E, F be the midpoints of arcs BC, CA, AB on the circumcircle.
Line `a passes through the feet of the perpendiculars from A to DB and DC. Line ma passes
through the feet of the perpendiculars from D to AB and AC. Let A1 denote the intersection
of lines `a and ma . Define points B1 and C1 similarly. Prove that triangle DEF and A1 B1 C1
are similar to each other.
2 A finite sequence of integers a1 , a2 , . . . , an is called regular if there exists a real number x
satisfying
bkxc = ak for 1 k n.
Given a regular sequence a1 , a2 , . . . , an , for 1 k n we say that the term ak is forced if the
following condition is satisfied: the sequence
a1 , a2 , . . . , ak1 , b
is regular if and only if b = ak . Find the maximum possible number of forced terms in a
regular sequence with 1000 terms.
3 Divide the plane into an infinite square grid by drawing all the lines x = m and y = n for
m, n Z. Next, if a squares upper-right corner has both coordinates even, color it black;
otherwise, color it white (in this way, exactly 1/4 of the squares are black and no two black
squares are adjacent). Let r and s be odd integers, and let (x, y) be a point in the interior
of any white square such that rx sy is irrational. Shoot a laser out of this point with slope
r/s; lasers pass through white squares and reflect off black squares. Prove that the path of
this laser will form a closed loop.

This file was downloaded from the AoPS Math Olympiad Resources Page
http://www.artofproblemsolving.com/

158

Page 1

USA
TSTST
2013

Day 2 - 23 June 2013


4 Circle , centered at X, is internally tangent to circle , centered at Y , at T . Let P and S be
variable points on and , respectively, such that line P S is tangent to (at S). Determine
the locus of O the circumcenter of triangle P ST .
5 Let p be a prime. Prove that any complete graph with 1000p vertices, whose edges are labelled
with integers, has a cycle whose sum of labels is divisible by p.
6 Let N be the set of positive integers. Find all functions f : N N that satisfy the equation
f abca (abc) + f abcb (abc) + f abcc (abc) = a + b + c
for all a, b, c 2. (Here f 1 (n) = f (n) and f k (n) = f (f k1 (n)) for every integer k greater
than 1.)

159 This file was downloaded from the AoPS Math Olympiad Resources Page
http://www.artofproblemsolving.com/

Page 2

USA
TSTST
2013

Day 3 - 25 June 2013


7 A country has n cities, labelled 1, 2, 3, . . . , n. It wants to build exactly n 1 roads between
certain pairs of cities so that every city is reachable from every other city via some sequence
of roads. However, it is not permitted to put roads between pairs of cities that have labels
differing by exactly 1, and it is also not permitted to put a road between cities 1 and n. Let
Tn be the total number of possible ways to build these roads. (a) For all odd n, prove that
Tn is divisible by n. (b) For all even n, prove that Tn is divisible by n/2.
8 Define a function f : N N by f (1) = 1, f (n + 1) = f (n) + 2f (n) for every positive integer
n. Prove that f (1), f (2), . . . , f (32013 ) leave distinct remainders when divided by 32013 .
9 Let r be a rational number in the interval [1, 1] and let = cos1 r. Call a subset S of the
plane good if S is unchanged upon rotation by around any point of S (in both clockwise
and counterclockwise directions). Determine all values of r satisfying the following property:
The midpoint of any two points in a good set also lies in the set.

The problems on this page are copyrighted by the Mathematical Association of


Americas American Mathematics Competitions.

160 This file was downloaded from the AoPS Math Olympiad Resources Page
http://www.artofproblemsolving.com/

Page 3

USA
USAMO
2013

Day 1 - 30 April 2013


1 In triangle ABC, points P , Q, R lie on sides BC, CA, AB respectively. Let A , B , C denote
the circumcircles of triangles AQR, BRP , CP Q, respectively. Given the fact that segment
AP intersects A , B , C again at X, Y , Z, respectively, prove that Y X/XZ = BP/P C.
2 For a positive integer n 3 plot n equally spaced points around a circle. Label one of them A,
and place a marker at A. One may move the marker forward in a clockwise direction to either
the next point or the point after that. Hence there are a total of 2n distinct moves available;
two from each point. Let an count the number of ways to advance around the circle exactly
twice, beginning and ending at A, without repeating a move. Prove that an1 + an = 2n for
all n 4.
3 Let n be a positive integer. There are n(n+1)
marks, each with a black side and a white
2
side, arranged into an equilateral triangle, with the biggest row containing n marks. Initially,
each mark has the black side up. An operation is to choose a line parallel to the sides of the
triangle, and flipping all the marks on that line. A configuration is called admissible if it can
be obtained from the initial configuration by performing a finite number of operations. For
each admissible configuration C, let f (C) denote the smallest number of operations required
to obtain C from the initial configuration. Find the maximum value of f (C), where C varies
over all admissible configurations.

This file was downloaded from the AoPS Math Olympiad Resources Page
http://www.artofproblemsolving.com/

161

Page 1

USA
USAMO
2013

Day 2 - 01 May 2013


4 Find all real numbers x, y, z 1 satisfying
p

min( x + xyz, y + xyz, z + xyz) = x 1 + y 1 + z 1.


5 Given postive integers m and n, prove that there is a positive integer c such that the numbers
cm and cn have the same number of occurrences of each non-zero digit when written in base
ten.
6 Let ABC be a triangle. Find all points P on segment BC satisfying the following property:
If X and Y are the intersections of line P A with the common external tangent lines of the
circumcircles of triangles P AB and P AC, then


PA
XY

2
+

PB PC
= 1.
AB AC

The problems on this page are copyrighted by the Mathematical Association of


Americas American Mathematics Competitions.

162 This file was downloaded from the AoPS Math Olympiad Resources Page
http://www.artofproblemsolving.com/

Page 2

Uzbekistan
National Olympiad
2013

1 Let real numbers a, b such that a b 0. Prove that


p
p
p
3
4
a2 + b2 + a3 + b3 + a4 + b4 3a + b.
2 Let x and y are real numbers such that x2 y 2 + 2yx2 + 1 = 0. If S =
find (a)maxS and (b) minS.

2
x2

+ 1 + x1 + y(y + 2 + x1 ),

3 Find all functions f : Q Q such that


f (x + y) + f (y + z) + f (z + t) + f (t + x) + f (x + z) + f (y + t) 6f (x 3y + 5z + 7t)
for all x, y, z, t Q.
4 Let circles and are circumcircle and incircle of the triangle ABC, the incircle touches
sides BC, CA, AB at the points A1 , B1 , C1 . Let A2 and B2 lies the lines A1 I and B1 I (A1 and
A2 lies different sides from I, B1 and B2 lies different sides from I) such that IA2 = IB2 = R.
Prove that : (a) AA2 = BB2 = IO; (b) The lines AA2 and BB2 intersect on the circle ;
5 Let SABC is pyramid, such that SA 4, SB 7, SC 9, AB = 5, BC 6 and AC 8.
Find max value capacity(volume) of the pyramid SABC.

This file was downloaded from the AoPS Math Olympiad Resources Page
http://www.artofproblemsolving.com/

163

Page 1

Vietnam
National Olympiad
2013

Day 1
1 Solve with full solution:
q
q
(sin x)2 + 1 2 + (cos y)2 +
(sin x)
q
q
(sin y)2 + 1 2 + (cos x)2 +
(sin y)

2 Define a sequence {an } as:

1
(cos y)2

1
(cos x)2

20y

q x+y
20x
x+y

a1 = 1

an+1 = 3 an + 2 for n 1.
2an
Prove that this sequence has a finite limit as n + . Also determine the limit.

3 Let ABC be a triangle such that ABC isnt a isosceles triangle. (I) is incircle of triangle
touches BC, CA, AB at D, E, F respectively. The line through E perpendicular to BI cuts
(I) again at K. The line through F perpendicular to CI cuts (I) again at L.J is midpoint
of KL. a) Prove that D, I, J collinear. b) B, C are fixed points,A is moved point such that
AB
AC = k with k is constant.IE, IF cut (I) again at M, N respectively.M N cuts IB, IC at
P, Q respectively. Prove that bisector perpendicular of P Q through a fixed point.
4 Write down some numbers a1 , a2 , . . . , an from left to right on a line. Step 1, we write a1 + a2
between a1 , a2 ; a2 + a3 between a2 , a3 , , an1 + an between an1 , an , and then we have new
sequence b = (a1 , a1 + a2 , a2 , a2 + a3 , a3 , . . . , an1 , an1 + an , an ). Step 2, we do the same
thing with sequence b to have the new sequence c again. And so on. If we do 2013 steps,
count the number of the number 2013 appear on the line if a) n = 2, a1 = 1, a2 = 1000 b)
n = 1000, ai = i, i = 1, 2 . . . , 1000

Sorry for my bad English Moderator says: alternate phrasing here: ! m a class=postlink
href=https://www.artofproblemsolving.com/Forum/viewtopic.php?f=42t=516134https://www.artofprob
... 2t=516134/a! m

This file was downloaded from the AoPS Math Olympiad Resources Page
http://www.artofproblemsolving.com/

164

Page 1

Vietnam
National Olympiad
2013

Day 2
1 Find all f : R R that satisfies f (0) = 0, f (1) = 2013 and
(x y)(f (f 2 (x)) f (f 2 (y))) = (f (x) f (y))(f 2 (x) f 2 (y))
Note: f 2 (x) = (f (x))2
2 Let ABC be a cute triangle.(O) is circumcircle of 4ABC.D is on arc BC not containing
A.Line 4 moved through H(H is orthocenter of 4ABC cuts circumcircle of 4ABH,circumcircle
4ACH again at M, N respectively. a.Find 4 satisfy SAM N max b.d1 , d2 are the line through
M perpendicular to DB,the line through N perpendicular to DC respectively. d1 cuts d2 at
P .Prove that P move on a fixed circle.
3 Find all ordered 6-tuples satisfy following system of modular equation: ab + a0 b0 1(mod 15)
bc + b0 c0 1(mod 15) ca + c0 a0 1(mod 15) Given that a, b, c, a0 , b0 , c0 (0; 1; 2; ...; 14)

165 This file was downloaded from the AoPS Math Olympiad Resources Page
http://www.artofproblemsolving.com/

Page 2

Вам также может понравиться